You are on page 1of 121

MANAGEMENT ADVISORY SERVICES CAPITAL BUDGETING

BASIC CONCEPTS Capital Budgeting vs. Strategic Planning


Capital Budgeting Defined 3. Which of the following statements regarding capital budgeting and
1
. The capital budget is a(n) strategic planning is true?
a. Plan to insure that there are sufficient funds available for the operating A. Capital budgeting and strategic planning are bottom-up processes
needs of the company. B. Capital budgeting and strategic planning are top down processes B & M
b. Exercise that sets the long-range goals of the company including the C. Capital budgeting is a top-down process, while strategic planning is a
consideration of external influences. bottom-up process
c. Plan that coordinates and communicates a company’s plan for the D. Capital budgeting is a bottom-up process, while strategic planning is a
coming year to all departments and divisions. top-down process
d. Plan that assesses the long-term needs of the company for plant and
equipment purchases. CMA 0695 3-17 Agency Problem in Capital Budgeting
15. The following are agency problems in capital budgeting except:
2
. In planning and controlling capital expenditures, the most logical sequence A. Reduced effort C. Empire building
is to begin with B. Need for good information D. Perks B&M
a. Analyzing capital addition proposals.
b. Making capital and expenditure decisions. 16. The following are agency problems in capital budgeting except:
c. Analyzing and evaluating all promising alternatives. A. Empire building C. Avoiding risks
d. Identifying capital addition projects and other capital needs. CMA 0696 B. Entrenching investment D. Reducing forecast bias B&M
3-11
17. The following are agency problems in capital budgeting except:
21. Which of the following involves significant financial investments in projects A. Avoiding risks
to develop new products, expand production capacity, or remodel current B. The free-rider problem
production facilities? (E) C. Compensation
a. Capital budgeting c. Master budgeting D. All of the above are agency problems B&M
b. Working capital d. Project-cost budgeting Horngren
18. The following are agency problems in capital budgeting except:
*. The detailed plan for the acquisition and replacement of major portions of A. Monitoring C. Avoiding risks
property, plant, and equipment is known as the B. Compensation D. Economic value added B&M
a. capital budget. c. commitments budget.
b. purchases budget. d. treasury budget. Barfield Uses
107. A capital budget is used by management to determine
2. _____________ decisions are concerned with the process of planning, setting Barfield a. b. c. d.
goals and priorities, arranging financing, and identifying criteria for making In what to invest No No Yes Yes
long-term investments. How much to No Yes No Yes
a. Limited resources c. Capital investment invest
b. Sell now or process further d. Make or buy H&M
3
. Capital budgeting is concerned with
A. Decisions affecting only capital intensive industries.
CMA EXAMINATION QUESTIONS Page 1 of 121
MANAGEMENT ADVISORY SERVICES CAPITAL BUDGETING

B. Analysis of short-range decisions. Life Cycle Costing


C. Analysis of long-range decisions. 22. The accounting system that corresponds to the project dimension in capital
D. Scheduling office personnel in office buildings. Gleim budgeting is the (E)
a. net present value method. c. accrual accounting rate of return.
4
. Capital budgeting is used for the decision analysis of b. internal rate of return. d. life-cycle costing. Horngren
A. Adding product lines or facilities. C. Lease-or-buy decisions.
B. Multiple profitable alternatives. D. All of the answers are correct. Stages
Gleim Identification Stage
23. The stage of the capital budgeting process which distinguishes which types
5
. Capital budgeting techniques are least likely to be used in evaluating the of capital expenditure projects are necessary to accomplish organization
(E) objectives is the (E)
a. Acquisition of new aircraft by a cargo company. a. identification stage. c. information-acquisition stage.
b. Design and implementation of a major advertising program. b. search stage. d. selection stage. Horngren
c. Trade for a star quarterback by a football team. CMA 0693 4-19
d. Adoption of a new method of allocating non-traceable costs to product Information-Acquisition Stage
lines. 25. The stage of the capital budgeting process which considers the expected
costs and the expected benefits of alternative capital investments is the (E)
Application a. identification stage. c. information-acquisition stage.
2. A company can replace the machinery currently used to manufacture its b. search stage. d. selection stage. Horngren
product with more efficient machinery. The new machinery will reduce
labor and also will reduce the percentage of spoiled units. It is expected to Search Stage
have a useful life of 5 years. The most appropriate technique for 24. The stage of the capital budgeting process which explores alternative
determining whether or not the company should replace its machinery with capital investments that will achieve organization objectives is the (E)
the new, more efficient machinery is: a. identification stage. c. information-acquisition stage.
A. cost-volume-profit analysis C. regression analysis b. search stage. d. selection stage. Horngren
B. capital-budgeting analysis D. linear programming CIA adapted
Selection Stage
Not-for-Profit Entities 26. The stage of the capital budgeting process which chooses projects for
10. Not-for-profit entities implementation is the (E)
a. Cannot use capital budgeting techniques because profitability is a. selection stage. c. identification stage.
irrelevant to them. b. search stage. d. management-control stage.
b. Cannot use discounted cash flow techniques because the time value of Horngren
money is irrelevant to them.
c. Might have serious problems in quantifying the benefits expected from Appropriation Requests
an investment. 8. Which of the following statements regarding appropriation requests is true?
d. Should use the IRR method to make investment decisions. L&H A. Usually submitted by head office staff
B. Represents the first step in the capital budgeting process
C. Authorization tends to be reserved for senior management
CMA EXAMINATION QUESTIONS Page 2 of 121
MANAGEMENT ADVISORY SERVICES CAPITAL BUDGETING

D. None of the above B&M d. Require forecasts of cash flows expected from the project. L&H
6
9. Which of the following statements regarding appropriation requests is true? . High-Tech Industries is considering the acquisition of a new state-of-the-art
A. Usually submitted by head office staff manufacturing machine to replace a less efficient machine. Hi-Tech has
B. Usually requires a detailed analysis using more than one investment completed a net present value analysis and found it to be favorable. Which
criterion one of the following factors should not be of concern to Hi-Tech in its
C. Usually submitted to a single review at the head office acquisition considerations?
D. None of the above B&M A. The availability of any necessary financing.
B. The probability of near-term technological changes to the
Implementation & Control Stage manufacturing process.
27. The stage of the capital-budgeting process in which projects get underway C. The investment tax credit. CMA 1290 4-18
and performance is monitored is the (E) D. Maintenance requirements, warranties, and availability of service
a. implementation and control stage. c. identification stage. arrangements.
b. search stage. d. management-control stage.
Horngren 8. A number of evaluations of a single capital expenditure proposal may be
necessary because of:
Acquisition Considerations A. circumstances that change during the time span from the origin of the
6. Effective planning and control is important for the effective administration project idea to its completion
of a capital expenditure program because: B. alternative solutions of the problem for which the project is designed
A. the long-term commitment increases financial risk C. assumptions that vary as to the amount and timing of cash flows
B. the magnitude of expenditures is substantial and the economic D. all of the above Carter & Usry
penalties for unwise decisions are usually severe
C. decisions made in this area provide the structure for operation of the 23. Which of the following events is most likely to increase the number of
firm investments that meet a company’s acceptance criteria? (M)
D. all of the above Carter & Usry a. Top management raises the target rate of return.
b. The interest rate on long-term debt rises.
7. A company manual used for detailing policies and procedures required for c. The income tax rate rises.
administering the capital expenditure program should: d. The IRS allows companies to expense purchases of fixed assets, instead
A. encourage people to work on and submit new ideas of depreciating them over their lives. L&H
B. focus attention on useful analytical tasks
C. facilitate rapid project development and expeditious review Qualitative Factors
D. all of the above Carter & Usry 37. Qualitative issues could increase the acceptability of a project under which
of the following conditions?
14. The normal methods of analyzing investments a. The IRR is less than the company’s cutoff rate.
a. Cannot be used by not-for-profit entities. b. The project has a negative NPV.
b. Do not apply if the project will not produce revenues. c. The payback period is longer than the company’s cutoff period.
c. Cannot be used if the company plans to finance the project with funds d. All of the above. L&H
already available internally.
CMA EXAMINATION QUESTIONS Page 3 of 121
MANAGEMENT ADVISORY SERVICES CAPITAL BUDGETING

40. Qualitative factors can influence managers to C. Shortly after a project has started operating B&M
a. Accept an investment project having a negative NPV. D. Long after the project has been completed and the salvage has been
b. Reject an investment project having an IRR greater than the company’s realized
cutoff rate.
c. Raise the “ranking” of an investment project. 77. A post audit compares
d. Take any of the above courses of action. L&H a. estimated benefits and costs with budgeted benefits and cost
b. estimated benefits with estimated costs
76. Which of the following statements is (are) true about automation? c. actual benefits with actual costs
a. Automation is inexpensive. d. actual benefits and costs with estimated benefits and costs H&M
b. Automation should be adopted as soon as new technology is available.
c. Automation should be adopted after a company makes the most 27 The post-audit is used to(E)
efficient use of existing technology. a. Improve cash flow forecasts.
d. All of the above are true H&M b. Stimulate management to improve operations and bring results into line
with forecasts.
Ethical Consideration c. Eliminate potentially profitable but risky projects.
4. Common problems related to ethical considerations in the capital d. Statements a and b are correct.
budgeting include all of the following, except: e. All of the statements above are correct. Brigham
A. superiors and associates sometimes apply pressure to circumvent the
approval process 11. A post audit will:
B. pressure may exist to write-off or devalue assets below their true value A. Identify the problem that needs to be fixed
to justify replacement B. Check the accuracy of the cash flow forecasts
C. the economic benefit of capital projects may be exaggerated to C. Suggest questions that should have been asked before
increase the likelihood of approval D. All of the above B&M
D. the accountant may mistakenly go to the individuals involved in the
ethical conflict first, rather than first discussing it with the accounting Type of Capital Expenditure
supervisor Replacement Expenditures
E. all of the above are ethical problems related to capital budgeting AICPA 9. The following capital expenditures that compare the future costs of the old
adapted assets with the future costs of the new assets as a basis for making a
decision are:
Post- Investment Audit A. replacement expenditures C. improvement expenditures
69. Comparison of the actual results for a project to the costs and benefits B. expansion expenditures D. allowance expenditures Carter &
expected at the time the project was selected is referred to as (E) Usry
a. the audit trail. c. a postinvestment audit.
b. management control. d. a cost-benefit analysis. Horngren Expansion Expenditures
10. In which of the following types of capital expenditure decisions does the
10. Post audit is conducted: basis for a decision most markedly shift from cost savings to increased
A. Before starting a project profits and cash flow?
B. Before authorizing a project A. replacement expenditures C. improvement expenditures
CMA EXAMINATION QUESTIONS Page 4 of 121
MANAGEMENT ADVISORY SERVICES CAPITAL BUDGETING

B. expansion expenditures D. allowance expenditures Carter &


Usry Types of Projects
Independent Projects
Improvement Expenditures 1. ______________ are projects that when accepted or rejected will NOT affect
11. The capital expenditures in which the benefits are most difficult to quantify the cash flows of another project.
are: a. Independent projects c. Dependent projects
A. replacement expenditures C. improvement expenditures b. Mutually exclusive projects d. Both b and c H&M
B. expansion expenditures D. allowance expenditures Carter &
Usry Mutually Exclusive Projects
4. ______________are projects that when accepted preclude the acceptance of
Not a Capital Expenditure competing projects.
6. The following capital expenditures may not appear in the capital budget a. Independent projects c. Dependent projects
except: b. Mutually exclusive projects d. Both b and c H&M
A. Investment in information technology
B. Investment in research and development Inflation Element
C. Investment in training and personal development *. The “inflation element” refers to the
D. Investment in a new office building B&M a. Impact that future price increases will have on the original cost of a
capital expenditure.
7. The following capital expenditures may not appear in the capital budget b. Fact that the real purchasing power of a monetary unit usually
except: increases over time.
A. Marketing c. Future deterioration of the general purchasing power of the monetary
B. Training and personnel development unit.
C. Investment in a new machine d. Future increases in the general purchasing power of the monetary unit.
D. Investment in research and development B&M RPCPA 0597

4. Which of the following capital expenditure may not appear in capital Relevant and Irrelevant Costs
budget? 28. Capital budgeting emphasizes two factors (E)
A. Investment in a new plant a. qualitative and nonfinancial. c. quantitative and financial
B. Investment in a new machine b. quantitative and nonfinancial. d. qualitative and financial.
C. Investment in information technology Horngren
D. All of the above are included in capital budget B&M
29. Which of the following are NOT included in the formal financial analysis of a
5. Which of the following capital expenditures may not appear in capital capital budgeting program? (E)
budget? a. Quality of the output c. Cash flow
A. Investment in a new building b. Safety of employees d. Neither (a) nor (b) are included
B. Investment in a new machine Horngren
C. Investment in research and development
D. All of the above are included in capital budget B&M 56. The focus in capital budgeting should be on (E)
CMA EXAMINATION QUESTIONS Page 5 of 121
MANAGEMENT ADVISORY SERVICES CAPITAL BUDGETING

a. the tax consequences of different investment strategies.


b. the internal rate of return of different strategies. *. You are the treasurer of the Hibang Corp. The company is considering a
c. expected future cash flows that differ between alternatives. proposed project which has an expected economic life of seven years. Net
d. none of the above. Horngren present value is the capital budgeting technique the president wants you to
use. Salvage value of the project would be (M)
32. The only future costs that are relevant to deciding whether to accept an a. Treated as cash inflow at estimated salvage value.
investment are those that will b. Treated as cash flow at its present value.
a. Be different if the project is accepted rather than rejected. c. Irrelevant cash flow item.
b. Be saved if the project is accepted rather than rejected. d. Treated as cash inflow at the future value. RPCPA 1096
c. Be deductible for tax purposes.
d. Affect net income in the period that they are incurred. L&H Working Capital
66. In the analysis of a capital budgeting proposal, for which of the following
7
. Which of the following rules are essential to successful cash flow estimates, items are there no after-tax consequences? (E)
and ultimately, to successful capital budgeting? (M) a. Cash flow from operations
a. The return on invested capital is the only relevant cash flow. b. Gain or loss on the disposal of the asset
b. Only incremental cash flows are relevant to the accept/reject decision. c. Reduction of working capital balances at the end of the useful life of the
c. Total cash flows are relevant to capital budgeting analysis and the capital asset
accept/reject decision. d. There are no after-tax consequences of any of the above. Horngren
d. Statements a and b are correct.
e. All of the statements above are correct. Brigham 11. A major difference between an investment in working capital and one in
depreciable assets is that (M)
Salvage Value a. An investment in working capital is never returned, while most
60. The relevant terminal disposal price of a machine equals (M) depreciable assets have some residual value.
a. the difference between the salvage value of the old machine and the b. An investment in working capital is returned in full at the end of the
ultimate salvage value of the new machine. project’s life, while an investment in depreciable assets has no residual
b. the total of the salvage values of the old machine and the new value.
machine. c. An investment in working capital is not tax-deductible when made, not
c. the salvage value of the old machine. taxable when returned, while an investment in depreciable assets does
d. the salvage value of the new machine. Horngren allow tax deductions.
d. Because an investment in working capital is usually returned in full at
*. Karen Company is considering replacing an old machine with a new the end of the project’s life, it is ignored in computing the amount of
machine. Which of the following items is economically relevant to Karen’s the investment required for the project. L&H
decisions? (M)
RPCPA 0598 a. b. c. d. *. Mahlin Movers, Inc. is planning to purchase equipment to make its
Carrying amount of old Yes Yes No No operations more efficient. This equipment has an estimated useful life of
machine six years. As part of this acquisition, a P150,000 investment in working
Disposal value of new Yes No Yes No capital is required. In a discounted cash flow analysis, this investment in
machine working capital should be (E)
CMA EXAMINATION QUESTIONS Page 6 of 121
MANAGEMENT ADVISORY SERVICES CAPITAL BUDGETING

a. Amortized over the useful life of the equipment. d. Not real. L & H, RPCPA 1001
b. Disregarded because no cash is involved. RPCPA 1095
c. Treated as a recurring annual cash flow that is recovered at the end of 29. XYZ Co. is adopting just-in-time principles. When evaluating an investment
six years. project that would reduce inventory, how should XYZ treat the reduction?
d. Treated as an immediate cash outflow that is recovered at the end of a. Ignore it.
six years. b. Decrease the cost of the investment and decrease cash flows at the end
of the project’s life.
8
. Fast Freight, Inc. is planning to purchase equipment to make its operations c. Decrease the cost of the investment.
more efficient. This equipment has an estimated life of 6 years. As part of d. Decrease the cost of the investment and increase the cash flow at the
this acquisition, a $75,000 investment in working capital is anticipated. In end of the project’s life. L&H
a discounted cash flow analysis, the investment in working capital (E)
a. Should be amortized over the useful life of the equipment. 4. Net Working Capital should be considered in project cash flows because:
b. Should be treated as a recurring cash outflow over the life of the A. They are sunk costs
equipment. B. Firms must invest cash in short-term assets to produce finished goods
c. Should be treated as an immediate cash outflow. CMA 0691 4-20 C. Firms need positive NPV projects for investment
d. Should be treated as an immediate cash outflow recovered at the end D. None of the above B&M
of 6 years.
Opportunity Costs
32. In connection with a capital budgeting project, an investment in working 10. The value of a previously purchased machine expected to be used by a
capital is normally recovered proposed project is an example of:
a. At the end of the project’s life. c. Evenly through the project’s life. L A. Sunk costs C. Fixed costs
&H B. Opportunity costs D. None of the above B&M
b. In the first year of the project’s life. d. When the company goes out
of business. Cash Outflows
14. All of the following are common cash outflows from capital expenditure
12. The proper treatment of an investment in receivables and inventory is to programs, except:
a. Ignore it. A. equipment installation D. increased working capital
b. Add it to the required investment in fixed assets. requirements
c. Add it to the required investment in fixed assets and subtract it from B. employee training E. salvage value at the end of the
the annual cash flows. project
d. Add it to the investment in fixed assets and add the present value of C. computer programming and fine tuning Carter & Usry
the recovery to the present value of the annual cash flows. L&H
33. For investments that have only costs (no revenues or cost savings), an
34. The cash inflow from the return on an investment in working capital is appropriate decision rule is to accept the project that has the
a. Adjusted for taxes due. a. Longest payback period.
b. Discounted to present value. b. Lower present value of cash outflows.
c. Ignored if any depreciable assets also involved in the project have no c. Higher present value of future cash outflows.
expected residual value. d. Lowest internal rate of return. L&H
CMA EXAMINATION QUESTIONS Page 7 of 121
MANAGEMENT ADVISORY SERVICES CAPITAL BUDGETING

d. depreciation usually has no effect on the disposal price of the machine.


Cash Inflows Horngren
15. As to a capital investment, net cash inflow is equal to the
a. cost savings resulting from the investment. 25. Which of these could occur in practice where the capital expenditure
b. sum of all future revenues from the investment. relates to the production of an established product or service, the demand
c. net increase in cash receipts over cash payments. for which is expected to vary in response to temporary changes in
d. net increase in cash payments over cash receipts. Barfield consumer taste?
A. perfectly correlated cash flows C. independent cash flows
8. Which of the following describes the annual returns that are discounted in B. negative cash flows D. mixed cash flows Carter & Usry
determining the NPV of an investment?
a. Net incomes expected to be earned by the project. 3. Which of the following is NOT relevant in calculating annual net cash flows
b. Pre-tax cash flows expected from the project. for an investment?
c. After-tax cash flows expected from the project. a. Interest payments on funds borrowed to finance the project.
d. After-tax cash flows adjusted for the time value of money. L&H b. Depreciation on fixed assets purchased for the project.
c. The income tax rate.
57. Annual after-tax corporate net income can be converted to annual after-tax d. Lost contribution margin if sales of the product invested in will reduce
cash flow by (E) sales of other products. L&H
a. adding back the depreciation amount.
b. deducting the depreciation amount. Barfield 39. Which of the following is NOT relevant in calculating net cash flows for
c. adding back the quantity (t x depreciation deduction), where t is the Project N?
corporate tax rate. a. Interest payments on funds that would be borrowed to finance Project
d. deducting the quantity [(1- t) x depreciation deduction], where t is the N.
corporate tax rate. b. Depreciation on assets purchased for Project N.
c. The contribution margin the company would lose if sales of the product
. To approximate annual cash inflow, depreciation is introduced by Project N will reduce sales of other products.
a. Added back to net income because it is an inflow of cash. d. The income tax rate applicable to the entity. L&H
b. Subtracted from net income because it is an outflow of cash.
c. Subtracted from net income because it is an expense. 13. All of the following are common cash inflows related to capital expenditure
d. Added back to net income because it is not an outflow of cash. RPCPA proposals, except:
1001 A. additional revenues from increased sales
B. increased working capital requirements
59. Depreciation is usually not considered an operating cash flow in capital C. reduction in inventory carrying costs
budgeting because (E) D. salvage value at the end of the project Carter & Usry
a. depreciation is usually a constant amount each year over the life of the
capital investment. Tax Shield on Depreciation
b. deducting depreciation from operating cash flows would be counting 8. Which of the following cash flows should be treated as incremental flows
the lump-sum amount twice. when deciding whether to go ahead with an electric car?
c. depreciation usually does not result in an increase in working capital. A. The cost of research and development undertaken for developing the
CMA EXAMINATION QUESTIONS Page 8 of 121
MANAGEMENT ADVISORY SERVICES CAPITAL BUDGETING

electric car in the past three years c. Interest expenses should be included in project cash flows.
B. The annual depreciation charge d. Statements a and b are correct. Brigham
C. The reduction in taxes resulting from the depreciation charges
10
D. Dividend payments B&M . When evaluating potential projects, which of the following factors should be
incorporated as part of a project’s estimated cash flows? (E)
Irrelevant Costs a. Any sunk costs that were incurred in the past prior to considering the
9. Money that a firm has already spent or committed to spend regardless of proposed project.
whether a project is taken is called: b. Any opportunity costs that are incurred if the project is undertaken.
A. Sunk costs C. Fixed costs c. Any externalities (both positive and negative) that are incurred if the
B. Opportunity costs D. None of the above B&M project is undertaken.
d. Statements b and c are correct. Brigham
*. In capital expenditures decisions, the following are relevant in estimating
11
operating costs except (E) . Which one of the following statements concerning cash flow determination
a. Future costs. c. Differential costs. for capital budgeting purposes is not correct?
b. Cash costs. d. Historical costs. RPCPA 1094 a. Tax depreciation must be considered since it affects cash payments for
taxes.
58. An example of a sunk cost in a capital budgeting decision for new b. Book depreciation is relevant since it affects net income.
equipment is (E) d. Net working capital changes should be included in cash flow forecasts.
a. increase in working capital required by a particular investment choice. c. Sunk costs are not incremental flows and should not be included. CMA
b. the book value of the old equipment. 1295 4-11
c. the necessary transportation costs on the new equipment.
12
d. all of the above are examples of sunk costs. Horngren . A company is considering a new project. The company’s CFO plans to
calculate the project’s NPV by discounting the relevant cash flows (which
7. The following cash flows should be treated as incremental flows when include the initial up-front costs, the operating cash flows, and the terminal
deciding whether to go ahead with an electric car except: (M) cash flows) at the company’s cost of capital (WACC). Which of the following
A. The consequent deduction in sales of the company's existing gasoline factors should the CFO include when estimating the relevant cash flows?
models (E)
B. The expenditure on new plants and equipment a. Any sunk costs associated with the project.
C. The value of tools that can be transferred from the company's existing b. Any interest expenses associated with the project.
plants c. Any opportunity costs associated with the project.
D. Interest payment on debt B&M d. Statements b and c are correct. Brigham
13
Comprehensive . Which of the following statements is correct? (M)
9
. Which of the following statements is most correct? (E) a. An asset that is sold for less than book value at the end of a project’s
a. When evaluating corporate projects it is important to include all sunk life will generate a loss for the firm and will cause an actual cash
costs in the estimated cash flows. outflow attributable to the project.
b. When evaluating corporate projects it is important to include all
relevant externalities in the estimated cash flows.
CMA EXAMINATION QUESTIONS Page 9 of 121
MANAGEMENT ADVISORY SERVICES CAPITAL BUDGETING

b. Only incremental cash flows are relevant in project analysis and the project, which of the following items should Laurier explicitly include in its
proper incremental cash flows are the reported accounting profits cash flow analysis? (M)
because they form the true basis for investor and managerial decisions. a. The company will produce the detergent in a vacant facility that they
c. It is unrealistic to expect that increases in net operating working capital renovated five years ago at a cost of $700,000.
that are required at the start of an expansion project are simply b. The company will need to use some equipment that it could have
recovered at the project’s completion. Thus, these cash flows are leased to another company. This equipment lease could have
included only at the start of a project. generated $200,000 per year in after-tax income.
d. Equipment sold for more than its book value at the end of a project’s c. The new detergent is likely to significantly reduce the sales of the other
life will increase income and, despite increasing taxes, will generate a detergent products the company currently sells.
greater cash flow than if the same asset is sold at book value. Brigham d. Statements b and c are correct. Brigham
14 18
. Which of the following statements is most correct? (E) . Sanford & Son Inc. is thinking about expanding their business by opening
a. The rate of depreciation will often affect operating cash flows, even another shop on property they purchased 10 years ago. Which of the
though depreciation is not a cash expense. following items should be included in the analysis of this endeavor? (M)
b. Corporations should fully account for sunk costs when making a. The property was cleared of trees and brush 5 years ago at a cost of
investment decisions. $5,000.
c. Corporations should fully account for opportunity costs when making b. The new shop is expected to affect the profitability of the existing shop
investment decisions. since some current customers will transfer their business to the new
d. Statements a and c are correct. Brigham shop. Sanford and Son estimate that profits at the existing shop will
decrease by 10 percent.
15
. Which of the following is not a cash flow that results from the decision to c. Sanford & Son can lease the entire property to another company (that
accept a project? (E) wants to grow flowers on the lot) for $5,000 per year.
a. Changes in net operating working capital. d. Opportunity costs. d. Both statements b and c should be included in the analysis. Brigham
b. Shipping and installation costs. e. Externalities.
19
c. Sunk costs. Brigham . Pickles Corp. is a company that sells bottled iced tea. The company is
thinking about expanding its operations into the bottled lemonade
16
. Adams Audio is considering whether to make an investment in a new type business. Which of the following factors should the company incorporate
of technology. Which of the following factors should the company consider into its capital budgeting decision as it decides whether or not to enter the
when it decides whether to undertake the investment? (M) lemonade business? (M)
a. The company has already spent $3 million researching the technology. a. If the company enters the lemonade business, its iced tea sales are
b. The new technology will affect the cash flows produced by its other expected to fall 5 percent as some consumers switch from iced tea to
operations. lemonade.
c. If the investment is not made, then the company will be able to sell one b. Two years ago the company spent $3 million to renovate a building for
of its laboratories for $2 million. a proposed project that was never undertaken. If the project is adopted,
d. Statements b and c should be considered. Brigham the plan is to have the lemonade produced in this building.
c. If the company doesn’t produce lemonade, it can lease the building to
17
. Laurier Inc. is a household products firm that is considering developing a another company and receive after-tax cash flows of $500,000 a year.
new detergent. In evaluating whether to go ahead with the new detergent d. Statements a and c are correct. Brigham
CMA EXAMINATION QUESTIONS Page 10 of 121
MANAGEMENT ADVISORY SERVICES CAPITAL BUDGETING

d. The cost of a product analysis completed in the previous tax year and
20
. Which of the following statements is correct? (M) specific to the new product. Brigham
a. In a capital budgeting analysis where part of the funds used to finance
the project are raised as debt, failure to include interest expense as a Uncertainty
cost in the cash flow statement when determining the project’s cash 1. Which of the following best identifies the reason for using probabilities in
flows will lead to an upward bias in the NPV. capital budgeting decisions?
b. The preceding statement would be true if “upward” were replaced with A. uncertainty C. time value of money
“downward.” B. cost of capital D. projects with unequal lives AICPA
c. The existence of “externalities” reduces the NPV to a level below the adapted
value that would exist in the absence of externalities.
d. If one of the assets that would be used by a potential project is already *. Which of the following best identifies the reason for using probabilities in
owned by the firm, and if that asset could be leased to another firm if capital budgeting is (E)
the project is not undertaken, then the net rent that could be obtained a. Different life of projects. c. Uncertainty.RPCPA 0577, 0588,
should be charged as a cost to the project under consideration. 1093
e. The rent referred to in statement d is a sunk cost, and as such it should b. Cost of capital. d. Time value of money.
be ignored. Brigham
26. In capital expenditure analysis, which of the following can be constructed
21
. Which of the following constitutes an example of a cost that is not to evaluate alternative levels of investment?
incremental, and therefore, not relevant in an accept/reject decision? (M) A. normal distribution D. pie chart
a. A firm has a parcel of land that can be used for a new plant site, or B. bar graph E. payoff table
alternatively, can be used to grow watermelons. C. nonnormal distribution Carter & Usry
b. A firm can produce a new cleaning product that will generate new sales,
but some of the new sales will be from customers who switch from 28. The standard deviation of the expected net present value is determined by
another product the company currently produces. summing the discounted standard deviations for each period over the life
c. A firm orders and receives a piece of new equipment that is shipped of the project when the cash flows in each of the periods are:
across the country and requires $25,000 in installation and set-up A. independent D. negative
costs. B. positive E. perfectly correlated
d. Statements a, b, and c are examples of incremental cash flows, and C. mixed Carter & Usry
therefore, relevant cash flows. Brigham
12. Cinzano Inc. wants to use discounted cash flow techniques when analyzing
22
. Which of the following is not considered a relevant concern in deter- mining its capital investment projects. The company is aware of the uncertainty
incremental cash flows for a new product? (M) involved in estimating future cash flows. A simple method some
a. The use of factory floor space that is currently unused but available for companies employ to adjust for the uncertainty inherent in their estimates
production of any product. is to:
b. Revenues from the existing product that would be lost as a result of A. ignore salvage values
some customers switching to the new product. B. average the expectations of several different managers
c. Shipping and installation costs associated with preparing the machine C. use accelerated depreciation
to be used to produce the new product. D. adjust the minimum desired rate of return
CMA EXAMINATION QUESTIONS Page 11 of 121
MANAGEMENT ADVISORY SERVICES CAPITAL BUDGETING

E. increase the estimates of the cash flows CMA adapted No Income Tax Situation
21. If there were no income taxes,
Tax Factor a. Depreciation would be ignored in capital budgeting.
In general b. The NPV method would not work.
58. Income taxes are levied on c. Income would be discounted instead of cash flow.
a. net cash flow. d. All potential investments would be desirable. L&H
b. income as measured by accounting rules.
c. net cash flow plus depreciation. 35. If a company is NOT subject to income tax, which of the following is true of
d. income as measured by tax rules. Barfield a proposed investment?
a. The project’s IRR equals the entity’s cost of capital.
18. Which of the following statements is true? b. The project’s NPV is zero.
a. All revenue is taxed. c. Depreciation on assets required for the project is irrelevant to the
b. All expenses are tax-deductible. evaluation.
c. Some revenues and expenses have no tax effects. d. The expected annual increase in future cash flows equals the
d. Income taxes are based solely on revenues and expenses. L&H investment required to undertake the project. L&H

*. In capital budgeting decisions, the following items are considered among 53. The pre-tax and after-tax cash flows would be the same for all of the
others: (M) following items except (D)
1. Cash outflow for the investment. a. the liquidation of working capital at the end of a project's life.
2. Increase in working capital requirements. b. the initial (outlay) cost of an investment.
3. Profit on sale of old asset c. the sale of an asset at its book value.
4. Loss on write-off of old asset. d. a cash payment for salaries and wages. Barfield
For which of the above items would taxes be relevant? (D)
a. Items 1 and 3 only. c. All items. Depreciation Tax Shield
b. Items 3 and 4 only. d. Items 1, 3 and 4 only. RPCPA 0594 *. The accounting area in which the only objective of depreciation accounting
relates to the effect of depreciation charges upon tax payments is (E)
6. The government could encourage increases in investment by a. Income determination. c. Cost/volume/profit analysis.
a. Increasing tax rates. b. Financial reporting. d. Capital budgeting. RPCPA 0587
b. Lengthening the MACRS period.
c. Letting a company expense fixed assets in the year acquired instead of 29. Which statement describes the relevance of depreciation in calculating
through annual depreciation charges. cash flows?
d. Taking actions that would increase interest rates. L&H a. Depreciation is relevant only when income taxes exist.
b. Depreciation is always relevant.
61. Which of the following are tax deductible under U.S. tax law? (M) c. Depreciation is never relevant.
a. interest payments to bondholders c. common stock dividends d. Depreciation is relevant only with discounted cash flow methods. L & H
b. preferred stock dividends d. all of the above Barfield
56. Multiplying the depreciation deduction by the tax rate yields a measure of
the depreciation tax (D)
CMA EXAMINATION QUESTIONS Page 12 of 121
MANAGEMENT ADVISORY SERVICES CAPITAL BUDGETING

a. shield. c. payable. Changes in Tax Rates


b. benefit. d. loss. Barfield *. Your company is purchasing a transport equipment as part of its territorial
expansion strategy. The technical services department indicated that this
3. Depreciation is incorporated explicitly in the cash flow analysis of an equipment needs overhauling in year 4 or year 5 of its useful life. The
investment proposal because it: overhauling cost will be expected during the year the overhauling is done.
A. is a cost of operations that cannot be avoided The finance officer insists that the overhauling be done in year 4, not in
B. results in an annual cash outflow year 5. The most likely reason is (M) RPCPA 0594
C. is a cash inflow a. There is lower tax rate in year 5. c. The time value of money is
D. reduces the cash outlay for income taxes considered.
E. represents the initial cash outflow spread over the life of the investment b. There is higher tax rate in year 5 d. Due statements A and C above.
CMA adapted
Optimal Capital Budget
23 26
. A depreciation tax shield is . An optimal capital budget is determined by the point where the marginal
a. An after-tax cash outflow. cost of capital is (D)
b. A reduction in income taxes. A. Minimized.
c. The cash provided by recording depreciation. B. Equal to the average cost of capital.
d. The expense caused by depreciation. CMA 1293 4-14, RPCPA 0596 C. Equal to the rate of return on total assets.
D. Equal to the marginal rate of return on investment. CIA 1187 IV-43
24
. The annual tax depreciation expense on an asset reduces income taxes by
27
an amount equal to . A firm seeking to optimize its capital budget has calculated its marginal
a. The firm’s average tax rate times the depreciation amount. cost of capital and projected rates of return on several potential projects.
b. One minus the firm’s average tax rate times the depreciation amount. The optimal capital budget is determined by
c. The firm’s marginal tax rate times the depreciation amount. A. Calculating the point at which marginal cost of capital meets the
d. One minus the firm’s marginal tax rate times the depreciation amount. projected rate of return, assuming that the most profitable projects are
CMA 1293 4-20 accepted first.
B. Calculating the point at which average marginal cost meets average
Accelerated Method vs. Straight-line Method projected rate of return, assuming the largest projects are accepted
25
. The use of an accelerated method instead of the straight-line method of first.
depreciation in computing the net present value of a project has the effect C. Accepting all potential projects with projected rates of return exceeding
of the lowest marginal cost of capital.
a. Raising the hurdle rate necessary to justify the project. D. Accepting all potential projects with projected rates of return lower than
b. Lowering the net present value of the project. the highest marginal cost of capital. CIA 1191 IV-57
c. Increasing the present value of the depreciation tax shield.
28
d. Increasing the cash outflows at the initial point of the project. CMA 0695 . Shanahan Inc. has two divisions: Division A makes up 50 percent of the
4-3 company, while Division B makes up the other 50 percent. Shanahan’s beta
is 1.2. Looking at stand-alone competitors, Shanahan’s CFO estimates that
Division A’s beta is 1.5, while Division B’s beta is 0.9. The risk-free rate is 5

CMA EXAMINATION QUESTIONS Page 13 of 121


MANAGEMENT ADVISORY SERVICES CAPITAL BUDGETING
30
percent and the market risk premium is 5 percent. The company is 100 . Commodore Corporation is deciding whether it makes sense to invest in a
percent equity-financed. (WACC = ks, the cost of equity). project today, or to postpone this decision for one year. Which of the
Division B is considering the following projects given below. Each of the following statements best describes the issues that Commodore faces
projects has the same risk and all have the same risk as a “typical” Division when considering this investment timing option? (E)
B project. a. The investment timing option does not affect the expected cash flows
Project Capital required IRR and should therefore have no impact on the project’s risk.
1 $400 million 14.0% b. The more uncertainty about the project’s future cash flows the more
2 300 million 10.7 likely it is that Commodore will go ahead with the project today.
3 250 million 10.5 c. If the project has a positive expected NPV today, this means that its
4 320 million 10.0 expected NPV will be even higher if it chooses to wait a year.
5 230 million 9.0 d. All of the above statements are correct.
The company is debating which cost of capital they should use to evaluate e. None of the above statements is correct. Brigham
Division B’s projects. John Green argues that Shanahan should use the
same cost of capital for each of its divisions, and believes it should base the Real options
31
cost of equity on Shanahan’s overall beta. Becky White argues that the cost . Which of the following is an example of a flexibility option? (E)
of capital should vary for each division, and that Division B’s beta should be a. A company has the option to invest in a project today or to wait
used to estimate the cost of equity for Division B’s projects. a year.
If the company uses White’s approach, how much larger will the capital b. A company has the option to back out of a project that turns out
budget be than if it uses Green’s approach? (E) to be unproductive.
a. Capital budget is $320 million larger using White’s approach. c. A company pays a higher cost today in order to be able to
b. Capital budget is $550 million larger using White’s approach. reconfigure the project’s input or outputs at a later date.
c. Capital budget is $870 million larger using White’s approach. d. A company invests in a project today that may lead to enhanced
d. Capital budget is $1,200 million larger using White’s approach. technological improvements that allow it to expand into different
e. The capital budget is the same using the two approaches. Brigham markets at a later date.
e. All of the statements above are correct. Brigham
INVESTMENT OPTIONS
Abandonment option 4. Which of the following are not real options? (M)
29
. Which of the following statements best describes the likely impact that an a. The option to expand production if the product is successful.
abandonment option will have on a project’s expected cash flow and risk? b. The option to buy additional shares of stock if the stock price goes up.
(E) c. The option to expand into a new geographic region.
a. No impact on expected cash flow, but risk will increase. d. The option to abandon a project.
b. Expected cash flow increases and risk decreases. e. The option to switch sources of fuel used in an industrial furnace.
c. Expected cash flow increases and risk increases. Brigham
d. Expected cash flow decreases and risk decreases.
e. Expected cash flow decreases and risk increases. Brigham 5. Which of the following will not increase the value of a real option? (M)
a. An increase in the time remaining until the real option must be
Investment timing option exercised.
b. An increase in the volatility of the underlying source of risk.
CMA EXAMINATION QUESTIONS Page 14 of 121
MANAGEMENT ADVISORY SERVICES CAPITAL BUDGETING

c. An increase in the risk-free rate. a. Accrual accounting rate of return. c. Future value cash flow.
d. An increase in the cost of exercising the real option. b. Payback period. d. Discounted cash flow rate of
e. Statements b and d. Brigham return.

Abandonment and growth options 1. The following measures are used by firms when making capital budgeting
32
. Clueless Corporation never considers abandonment options or growth decisions except:
options when estimating its optimal capital budget. What impact does this A. Payback period C. Net present value
policy have on the company’s optimal capital budget? (M) B. Internal rate of return D. P/E ratio B&M
a. Its estimated capital budget is too small because it fails to consider
abandonment and growth options. Use of Net Income
b. Its estimated capital budget is too large because it fails to consider *. A number of techniques are commonly used in the analysis of capital
abandonment and growth options. budgeting decisions. Each method involves the measurement of cash
c. Failing to consider abandonment options makes the optimal capital flows, except the (E)
budget too large, but failing to consider growth options makes the a. Internal rate of return. c. Average rate of return method.
optimal capital budget too small, so it is unclear what impact this policy b. Payback period method. d. Net present value. RPCPA 1097
has on the overall capital budget.
d. Failing to consider abandonment options makes the optimal capital 20. The technique that does NOT use cash flows is
budget too small, but failing to consider growth options makes the a. Payback. c. IRR.
optimal capital budget too large, so it is unclear what impact this policy b. NPV. d. Book rate of return. L&H
has on the overall capital budget.
e. Neither abandonment nor growth options should have an effect on the 9. Which of the following capital budgeting methods does NOT consider the
company’s optimal capital budget. Brigham time value of money?
a. IRR. c. Time-adjusted rate of return.
CAPITAL BUDGET EVALUATION METHODS b. Book rate of return. d. NPV. L&H
In general
33
*. “Net present value” is an example of which concept? (E) . Which one of the following capital investment evaluation methods does not
a. Capital budgeting. c. Managerial control. take the time value of money into consideration?
b. Project feasibility. d. Management by exception. RPCPA a. Net present value. c. Internal rate of return.
0580 b. Discounted payback. d. Accounting rate of return. CMA
0696 4-26
*. “Net present value” is an example of which concept? (E)
a. Capital budgeting c. Management control Use of Cash Flows
b. Project feasibility d. Management by objectives RPCPA In general
0577 *. In capital budgeting, these techniques are applied: payback (PB) method,
net present value (NPV) method and time-adjusted rate of return (TARR)
*. All of the following are methods that aid management in analyzing the method. PB method has this in common with NPV and TARR methods. (M)
expected results of capital budgeting decisions, except (E) Horngren, RPCPA a. Use of cash flows.
1095, 1096 b. Consideration of the time value of money.
CMA EXAMINATION QUESTIONS Page 15 of 121
MANAGEMENT ADVISORY SERVICES CAPITAL BUDGETING

c. Use of discounting. d. internal rate of return and net present value H&M
d. Use of accrual method of accounting. RPCPA 1094
*. Which of the following capital investment rating procedures recognize(s)
57. All of the following are major categories of cash flows in capital investment the time value of money? (E)
decisions EXCEPT (E) RPCPA 0590 a. b. c. d.
a. the initial investment in machines and working capital. Profitability index Yes No Yes No
b. recurring operating cash flows. Discounted rate of return Yes Yes No No
c. the initial working capital investment
d. depreciation expense reported on the income statement. Horngren 1. Which of the following groups of capital budgeting techniques uses the
time value of money? (E)
Without Time Value of Money a. Book rate of return, payback, and profitability index.
1. Which of the following capital budgeting techniques ignores the time value b. IRR, payback, and NPV.
of money? c. IRR, NPV, and profitability index.
a. payback period c. internal rate of return d. IRR, book rate of return, and profitability index. L&H
b. net present value d. profitability index Barfield
30. Which of the following combinations of capital budgeting techniques
With Time Value of Method includes only discounted cash flow techniques?
26. A dollar now is worth more than a dollar to be received in the future a. Book rate of return, payback, and profitability index.
because of b. NPV, IRR and profitability index.
a. Inflation. c. The opportunity cost of waiting. c. IRR, payback, and NPV.
b. Uncertainty. d. None of the above. L&H d. Profitability index, NPV, and payback. L&H

28. Which of the following is a discounted cash flow method? Use of Time Value of Money vs. No Time Value of Money
a. NPV. c. Book rate of return. 27. In contrast to the payback period and book rate of return methods, the NPV
b. Payback. d. All of the above. L&H and IRR methods
a. Consider the time value of money. c. Use after-tax cash flows.
8. The method of project selection that considers the time value of money in a b. Ignore depreciation. d. All of the above. L&H
capital budgeting decision computes the:
A. accounting rate of return on average investment 18. The net present value and internal rate of return methods of capital
B. internal rate of return budgeting are superior to the payback method in that they: (M)
C. payback period a. are easier to implement.
D. return on investment b. consider the time value of money.
E. accounting rate of return on initial investment AICPA adapted c. require less input.
d. reflect the effects of depreciation and income taxes. AICPA adapted
52. Which of the following methods consider the time value of money?
a. payback and accounting rate of return
b. payback and internal rate of return
c. internal rate of return and accounting rate of return
CMA EXAMINATION QUESTIONS Page 16 of 121
MANAGEMENT ADVISORY SERVICES CAPITAL BUDGETING

ACCOUNTING RATE OF RETURN a. payback c. internal rate of return


Definition b. accounting rate of return d. net present value H&M
*. A capital budgeting method that provides a rough approximation of an
37
investment’s profitability as measured with net income from the income . The capital budgeting technique known as accounting rate of return uses
statement is known as: (E) a. b. c. d.
a. Average rate of return method. c. Payback period. RPCPA 1097 Revenue over life of No No Yes Yes
b. Net present value method. d. Internal rate of return method. project
Depreciation expense Yes No No Yes
34
. The technique that measures the estimated performance of a capital
investment by dividing the project's annual after-tax net income by the
average investment cost is called the (E)
A. Bail-out payback method. C. Profitability index method. CMA
0692 4-21
B. Internal rate of return method. D. Accounting rate of return method.
35
. The technique that measures the estimated performance of a capital
investment by dividing the project's annual after-tax net income by the
average investment cost is called the
A. Average rate of return method. C. Capital asset pricing model. CMA
1290 4-15
B. Internal rate of return method. D. Accounting rate of return method.

31. The capital budgeting method that divides a project's annual incremental
net income by the initial investment is the: (M)
a. internal rate of return method.
b. the simple ( or accounting) rate of return method.
c. the payback method.
d. the net present value method. CMA adapted

Characteristics
36
. The accounting rate of return
A. Is synonymous with the internal rate of return.
B. Focuses on income as opposed to cash flows.
C. Is inconsistent with the divisional performance measure known as
return on investment.
D. Recognizes the time value of money. CMA 0691 4-18

11. Which of the following methods uses income instead of cash flows?
CMA EXAMINATION QUESTIONS Page 17 of 121
MANAGEMENT ADVISORY SERVICES CAPITAL BUDGETING

99. The capital budgeting technique known as accounting rate of return uses From the above statements, which are considered limitations of the ARR
(D) concept? (M)
Barfield a. b. c. d. a. Statements 2 and 3 only. c. All the 3 statements.
Salvage value No No Yes Yes b. Statements 3 and 1 only. d. Statements 1 and 2 only. RPCPA
Time value of No Yes Yes No 1094
money
65. For capital budgeting decisions, the use of the accrual accounting rate of
Formula return for evaluating performance is often a stumbling block to the
38
. The method that divides a project’s annual after-tax net income by the implementation of the (E)
average investment cost to measure the estimated performance of a a. net cash flow.
capital investment is the b. most effective goal-congruence choice.
a. Internal rate of return method. c. Payback method. CMA 1294 4-24 c. discounted cash flow method for capital budgeting.
b. Accounting rate of return method. d. Net present value (NPV) method. d. most effective tax strategy. Horngren

100. In computing the accounting rate of return, the ______ level of 19. The disadvantages of the book rate of return method is/are:
investment should be used as the denominator. A. It uses net income instead of cash flows
a. Average c. Residual B. The pattern of income has no impact on the book rate of return
b. Initial d. Cumulative Barfield C. There is no clear cut decision rule
D. All of the above B&M
16. The accounting rate of return on original investment is calculated as
a. original investment/net income c. net income/original investment PAYBACK PERIOD
b. net income/debt d. assets/debt H&M Definition
10. The payback period is the
64. The approach to capital budgeting which divides an accounting measure of a. length of time over which the investment will provide cash inflows.
income by an accounting measure of investment is (E) b. length of time over which the initial investment is recovered.
a. net present value. c. payback method. Horngren c. shortest length of time over which an investment may be depreciated.
b. internal rate of return. d. accrual accounting rate of return. d. shortest length of time over which the net present value will be
positive. Barfield
18. The book rate of return on a project is calculated as:
39
A. Book Cash Flow/book assets C. Book assets/book income . The length of time required to recover the initial cash outlay of a capital
B. Book income/book assets D. None of the above B&M project is determined by using the CMA 1294 4-20
a. Discounted cash flow method. c. Weighted net present value
Limitation method.
*. The following statements refer to the accounting rate of return (ARR) b. Payback method. d. Net present value method.
1. The ARR is based on the accrual basis, not cash basis.
2. The ARR does not consider the time value of money. *. An investment rating approach which measures the length of time required
3. The profitability of the project is considered. to recover the initial outlay for a particular investment proposal is the (E)
a. Accounting rate of return c. Net present value
CMA EXAMINATION QUESTIONS Page 18 of 121
MANAGEMENT ADVISORY SERVICES CAPITAL BUDGETING

b. Payback period d. Present value index RPCPA 0590


*. The method that measures how quickly investment pesos may be
40
. The technique that measures the number of years required for the after-tax recovered is the (E)
cash flows to recover the initial investment in a project is called the a. Payback method c. Simple rate of return method
A. Net present value method. C. Profitability index method. CMA b. Time adjusted rate of return d. Least squares method RPCPA
1290 4-17 1074, 10/77
B. Payback method. D. Accounting rate of return method.
*. The payback method measures (E)
a. Profitability of an investment c. Time to recover investmentRPCPA
1093
b. Economic life of the investment d. Cash flow from an investment

27. The payback method measures: (E)


a. how quickly investment dollars may be recovered.
b. the cash flow from an investment.
c. the economic life of an investment.
d. the profitability of an investment. CMA adapted

61. The method that measures the time it will take to recoup, in the form of
future cash inflows, the total dollars invested in a project is called (E)
a. the accrued accounting rate-of-return method.
b. payback method.
c. internal rate-of-return method.
d. the book-value method. Horngren

63. The payback method of capital budgeting approach to the investment


decision highlights (E)
a. cash flow over the life of the investment.
b. the liquidity of the investment.
c. the tax savings of the depreciation amounts.
d. having as lengthy payback time as possible. Horngren

Characteristics
19. The technique most concerned with liquidity is (M)
a. Payback. c. IRR.
b. NPV. d. Book rate of return. L&H

2. The payback criterion for capital investment decisions


CMA EXAMINATION QUESTIONS Page 19 of 121
MANAGEMENT ADVISORY SERVICES CAPITAL BUDGETING

a. Is conceptually superior to the IRR criterion. a. Payback period. c. NPV.


b. Takes into consideration the time value of money. b. IRR. d. PI. L&H
c. Gives priority to rapid recovery of cash.
d. Emphasizes the most profitable projects. L&H 30. The evaluation of an investment having uneven cash flows using the
payback method: (M)
8. The payback period rule: a. cannot be done.
A. Varies the cut-off point with the interest rate b. can be done only by matching cash inflows and investment outflows on
B. Determines a cut-off point so that all projects accepted by the NPV rule a year-by-year basis.
will be accepted by the payback period rule. c. will product essentially the same results as those obtained through the
C. Requires an arbitrary choice of a cut-off point use of discounted cash flow techniques.
D. Both A and C B&M d. requires the use of a sophisticated calculator or computer software. G &
N 9e
2. Which of the following capital budgeting techniques may potentially ignore
part of a project's relevant cash flows? (M) 10. Which of the following investment rules may not use all possible cash flows
a. net present value c. payback period in its calculations?
b. internal rate of return d. profitability index Barfield A. Payback period. C. IRR
B. NPV D. All of the above B&M
41
. A characteristic of the payback method (before taxes) is that it (E)
a. Incorporates the time value of money. Advantage
b. Neglects total project profitability. *. An advantage of the payback method is (E)
c. Uses accrual accounting inflows in the numerator of the calculation.CMA a. Not based on cash flow data. c. Precise in estimate of profits.
0694 4-17 b. Insensitive of the life of the project. d. Easy to apply.
d. Uses the estimated expected life of the asset in the denominator of the RPCPA 1093
calculation.
*. An advantage of using the payback method of evaluating capital budgeting
4. Which of the following capital budgeting techniques does not routinely rely alternatives is that payback is
on the assumption that all cash flows occur at the end of the period? a. Insensitive to the life of the project considered.
a. internal rate of return c. profitability index b. Precise estimate of profitability.
b. net present value d. payback period Barfield c. Based on cash flow data.
d. Easy to apply. RPCPA 0577, 5/80, 5/98
7. The payback method assumes that all cash inflows are reinvested to yield a
return equal to 12. The advantage of the payback rule is:
a. the discount rate. c. the internal rate of return. A. Adjustment for uncertainty of early cash flows
b. the hurdle rate. d. zero. Barfield B. It is simple to use
C. Does not discount cash flows
39. Assuming that a project has already been evaluated using the following D. Both A and C B&M
techniques, the evaluation under which technique is least likely to be
affected by an increase in the estimated residual value of the project?
CMA EXAMINATION QUESTIONS Page 20 of 121
MANAGEMENT ADVISORY SERVICES CAPITAL BUDGETING

Disadvantage
*. This technique is criticised because it fails to consider investment 1. A major disadvantage of the payback period method is that it (E)
profitability (E) a. Is useless as a risk indicator.
a. Time adjusted ROI c. Average return on investment b. Ignores cash flows beyond the payback period.
b. Payback method d. Present value method RPCPA c. Does not directly account for the time value of money.
1093 d. Statements b and c are correct. Brigham

11. Which of the following capital budgeting techniques has been criticized 15. The following are disadvantages of using the payback rule except:
because it fails to consider investment profitability? (E) A. The payback rule ignores all cash flow after the cutoff date
a. payback method c. net present value method B. The payback rule does not use the time value of money
b. accounting rate of return d. internal rate of return Barfield C. The payback period is easy to calculate and use
D. The payback rule does not have the value additive property B&M
42
. Which one of the following statements about the payback method of
investment analysis is correct? The payback method Formula
a. Does not consider the time value of money. . The cash payback formula is:
b. Consider cash flows after the payback has been reached. a. Cost of Capital Investment / Net Income.
c. Uses discounted cash flow techniques. CMA 1295 4-1 b. Average Investment / Net Annual Cash Inflow.
d. Generally leads to the same decision as other methods for long-term c. Cost of Capital Investment / Net Annual Cash Inflow.
projects. d. Average Investment / Net Income. RPCPA 1001

34. Which of the following methods FAILS to distinguish between return of 5. Assume that a project consists of an initial cash outlay of $100,000
investment and return on investment. followed by equal annual cash inflows of $40,000 for 4 years. In the formula
a. NPV. c. Payback. X = $100,000/$40,000, X represents the (E)
b. IRR. d. Book rate of return. L&H a. payback period for the project. c. internal rate of return for the
project.
15. Which of the following is NOT a defect of the payback method? b. profitability index of the project. d. project's discount rate. Barfield
a. It ignores cash flow because it uses net income.
b. It ignores profitability. Decision Criteria
c. It ignores the present values of cash flows. 9. The payback period rule accepts all projects for which the payback period
d. It ignores the pattern of cash flows beyond the payback period. L&H is:
A. Greater than the cut-off value C. Is positive
14. Deficiencies associated with using the payback method to evaluate B. Less than the cut-off value D. An integer B&M
investment alternatives include all of the following, except that:
A. the present value of cash inflows is ignored Payback Reciprocal
43
B. inflows of different time periods are treated equally . The payback reciprocal can be used to approximate a project’s
C. it may be used to select those investments yielding a quick return of a. Profitability index
cash b. Net present value.
D. cash flows after the payback period are ignored CIA adapted c. Accounting rate of return if the cash flow pattern is relatively stable.
CMA EXAMINATION QUESTIONS Page 21 of 121
MANAGEMENT ADVISORY SERVICES CAPITAL BUDGETING

d. Internal rate of return if the cash flow pattern is relatively stable. CMA
45
0693 4-27 . The bailout payback method (E)
A. Is used by firms with federally insured loans.
Relevant Items B. Calculates the payback period using the sum of the net cash flows and
1. In order to calculate the payback period for a project, it is necessary to the salvage value.
know the: C. Calculates the payback period using the difference between net cash
A. salvage value D. net present value inflow and the salvage value.
B. useful life E. annual cash flow D. Estimates short-term profitability. Gleim
C. minimum desired rate of return Carter & Usry
Use
46
1. Calculating the payback period for a capital project requires knowing which . The bailout payback method
of the following? a. Incorporates the time value of money.
a. Useful life of the project. b. Equals the recovery period from normal operations.
b. The company’s minimum required rate of return. c. Eliminates the disposal value from the payback calculation.
c. The project’s NPV. d. Measures the risk if a project is terminated. CMA 1292 4-11
d. The project’s annual cash flow. L&H

Irrelevant Items
*. As a capital budgeting technique, the payback period considers
depreciation expenses (DE) and time value of money (TVM) as follows: (M)
RPCPA a. b. c. d.
1095
DE relevant irrelevant Irrelevant relevant
TVM relevant irrelevant Relevant irrelevant
44
. The capital budgeting technique known as payback period uses
a. b. c. d.
Depreciation expense Yes Yes No No
Time value of money Yes No No Yes

BAILOUT PAYBACK
Definition
*. The bailout payback period is (E)
a. The payback period used by firms with government insured loans.
b. The length of time for payback using cash flows plus the salvage value
to recover the original investment
c. (a) and (b)
d. None of the above. RPCPA 1090
CMA EXAMINATION QUESTIONS Page 22 of 121
MANAGEMENT ADVISORY SERVICES CAPITAL BUDGETING

*. The method of project selection which considers the time value of money in
DISCOUNTED CASH FLOW ANALYSIS a capital budgeting decision is accomplished by computing the (E)
37. The consumption opportunity increases when an investment with positive a. Accounting rate of return on initial investment
NPV is available because: b. Payback period.
A. The investment is better than what is available in the market c. Accounting rate of return on average investment.
B. The project rate of return is less than market rate d. Discounted cash flow. RPCPA 0598
C. The market is irrelevant to the investment criteria
D. All of the above are reasons for increase in consumption B&M 2. The component of the capital investment decision that would most likely
concern an accountant is the:
Factors A. social responsibility factors D. imponderables
14. When using one of the discounted cash flow methods to evaluate the B. competition E. legal restrictions
desirability of a capital budgeting project, which of the following factors is C. time value of money Carter & Usry
generally not important? (E)
a. method of financing the project under consideration *. The fact that an amount of money that is to be received in the future is not
b. timing of cash flows relating to the project equivalent to the same amount of money to be received now is referred to
c. impact of the project on income taxes to be paid as: (E)
d. amounts of cash flows relating to the project Barfield a. Present value of money. c. Future value of money.
b. Time value of money. d. Discounted value of money.
Assumptions RPCPA 0587
52. In a typical (conservative assumptions) after-tax discounted cash flow
47
analysis, depreciation expense is assumed to accrue at . What is the time value of money?
a. the beginning of the period. A. Interest. C. Future value.
b. the middle of the period. B. Present value. D. Annuity. Gleim
c. the end of the period.
d. irregular intervals over the life of the investment. Barfield 12. The time value of money is explicitly recognized through the process of
a. interpolating. c. annuitizing.
Time Value Of Money b. discounting. d. budgeting. Barfield
53. There are two reasons for discounting future cash flow. They are:
A. A dollar today is worth more than a dollar tomorrow 13. The time value of money is considered in long-range investment decisions
B. A safe dollar is worth more than a risky one by (M)
C. The value of a dollar is changing all the time a. assuming equal annual cash flow patterns.
D. A and B above B&M b. investing only in short-term projects.
c. assigning greater value to more immediate cash flows.
29. The line that connects the maximum that one can consume this year (now) d. ignoring depreciation and tax implications of the investment. Barfield
and the maximum one can consume next year:
48
A. Has a slope of (1+r) C. Has a slope of r . Basic time value of money concepts concern
B. Has a slope of -(1+r) D. Has a slope of 1/r B&M Gleim A. B. C. D.
Interest Yes Yes No No
CMA EXAMINATION QUESTIONS Page 23 of 121
MANAGEMENT ADVISORY SERVICES CAPITAL BUDGETING

Factors b. Having useful lives shorter than one year.


Risk Yes No Yes No c. That are essential to the business.
Cost of No Yes No Yes d. Involving replacement of existing assets. L&H
Capital
Definition of Discounted Cash Flow
49
. The present value may be calculated for discounted cash *. Which of the following methods measures the cash flows and outflows of a
Gleim A. B. C. D. project as if they occurred at a single point in time? (M)
Inflows Yes Yes No No a. Cash flow based payback period. c. Payback method.
Outflows Yes No Yes No b. Capital budgeting. d. Discounted cash flow. RPCPA 0595
Annuities Yes Yes No Yes
*. The method of project selection which considers the value of money in a
9. The net present value and the internal rate of return methods of decision capital budgeting decision is accomplished by computing the
making in capital budgeting are superior to the payback method in that a. Payback period.
they: b. Accounting rate of return on initial investment.
A. consider the time value of money c. Accounting rate of return on average investment.
B. are easier to implement d. Discounted cash flow. RPCPA 0577, 10/79, 10/93
C. consider accrual-based accounting income
D. require less input *. There are several evaluation techniques for determining the acceptability
E. reflect the effects of depreciation and income taxes AICPA adapted of an investment. The method that considers the time value of money in
an investment budgeting decision is accomplished by determining the (E)
Application a. Cash-flow payback method. c. Average rate of return.
11. A company is considering the purchase of a new conveyor belt system for b. Accounting rate of return. d. Discounted cash flow. RPCPA 1081
carrying parts and subassemblies from building to building within its plant
complex. It is expected that the system will have a useful life of at least ten *. The method of project selection which considers the time value of money in
years and that it will substantially reduce labor and waiting-time costs. If a capital budgeting decision is accomplished by computing the (E)
the company's average cost of capital is about 15% and if some evaluation a. Accounting rate of return on initial investment.
must be made of cost/benefit relationships (including the effects of b. Accounting rate of return on average investment.
interest) to determine the desirability of the purchase, the most relevant c. Discounted cash flow.
quantitative technique for evaluating the investment is: d. Payback period. RPCPA 0580
A. present value (or internal rate of return) analysis
B. Program Evaluation and Review Technique (PERT) *. The modern approach used by decision-makers in evaluating investment
C. accounting rate of return analysis opportunities by comparing the original investment with the cash
D. cost-volume-profit analysis generation for the entire life of the project to come up with the rate of
E. payback analysis AICPA adapted return on investment is (E)
a. Discounted cash flow method d. Profitability index
2. Discounted cash flow techniques for analyzing capital budgeting decisions b. Accounting rate of return approach e. Cash payoff method
are NOT normally applied to projects c. Cash flow projections RPCPA 1077
a. Requiring no investment after the first year of life.
CMA EXAMINATION QUESTIONS Page 24 of 121
MANAGEMENT ADVISORY SERVICES CAPITAL BUDGETING
50 51
. All of the following items are included in discounted cash flow analysis . The capital budgeting model that is ordinarily considered the best model
except for long-range decision making is the CMA 1294 4-25
a. Future operating cash savings. a. Payback model. c. Unadjusted rate of return model.
b. The disposal prices of the current assets b. Accounting rate of return model. d. Discounted cash flow model.
c. The future asset depreciation expense.
d. The tax effects of future asset depreciation. CMA 0694 4-18 *. When using one of the discounted-cash-flow methods to evaluate the
feasibility of a capital budgeting project, which of the following factors
generally is not important? (M)
a. The method of financing the project under consideration.
b. The impact of the project on income taxes to be paid.
c. The timing of cash flows relating to the project.
d. The amount of cash flows relating to the project. RPCPA 0598
52
. Discounted cash flow concepts concern
Gleim a. b. c. d.
Interest Yes Yes No No
Factors
Risk Yes No Yes No
53
. Depreciation is incorporated explicitly in the discounted cash flow analysis
of an investment proposal because it
A. Is a cost of operations that cannot be avoided.
B. Is a cash inflow.
C. Reduces the cash outlay for income taxes.
D. Represents the initial cash outflow spread over the life of the
investment. CMA 1277 5-14

31. Discounted cash flow methods for capital budgeting focus on (E)
a. cash inflows. c. cash outflows.
b. operating income. d. both (a) and (c). Horngren

Rate of Return
Discount Rate
18. The discount rate for safe projects is the:
A. Market rate of return C. Market risk premium
B. Risk-free rate D. None of the above B&M

CMA EXAMINATION QUESTIONS Page 25 of 121


MANAGEMENT ADVISORY SERVICES CAPITAL BUDGETING

19. The discount rate for a project with a risk the same as the market risk is c. The cost of equity financing.
the: d. The weighted-average cost of capital. CIA 0597 IV-42
A. Market rate of return C. Market risk premium
B. Risk-free rate D. None of the above B&M Opportunity Cost of Capital
32. The ___________________ is the highest rate of return that can be earned
Cost of Capital from the most attractive, alternative capital project available to the firm.
27. Hurdle rate for capital budgeting decisions is: (D)
A. The cost of capital C. The cost of equity a. accounting rate of return c. hurdle rate
B. The cost of debt D. All of the above B&M b. internal rate of return d. opportunity cost of capital Barfield

*. In an investment in plant the return that should keep the market price of 21. The opportunity cost of capital for a risky project is
the firm stock unchanged is (M) RPCPA 0577, 1077, 0588, 1093 A. The expected rate of return on a government security having the same
a. Payback c. Net present value maturity as the project
b. Discounted rate of return d. Cost of capital B. The expected rate of return on a well diversified portfolio of common
stocks
25. For a project such as plant investment, the return that should leave the C. The expected rate of return on a portfolio of securities of similar risks as
market price of the firm's stock unchanged is known as the the project
a. cost of capital. c. payback rate. D. None of the above B&M
b. net present value. d. internal rate of return. Barfield
Comprehensive
16. A company with cost of capital of 15% plans to finance an investment with . All of the following refer to the discount rate used by a firm in capital
debt that bears 10% interest. The rate it should use to discount the cash budgeting except (M)
flows is a. Hurdle rate. c. Opportunity cost.
a. 10%. c. 25%. b. Required rate of return. d. Opportunity cost of capital. RPCPA
b. 15%. d. Some other rate. L&H 1096

1. The term cost of capital for a project depends on: Present Value Formula
A. The use to which the capital is put, i.e. the project 94. If r is the discount rate, the formula [1/(1 + r)] refers to the
B. The company's cost of capital a. future value interest factor associated with r for one period.
C. The industry cost of capital b. present value of some future cash flow.
D. All of the above B&M c. present value interest factor associated with r for one period.
d. future value interest factor for an annuity with a duration of r periods.
Weighted-Average Cost of Capital Barfield
54
. A company had made the decision to finance next year’s capital projects
through debt rather than additional equity. The benchmark cost of capital 97. Which of the following indicates that the first cash flow is at the end of a
for these projects should be (M) period? (M)
a. The before-tax cost of new-debt financing. Barfield a. b. c. d.
b. The after-tax cost of new-debt financing. Ordinary Yes Yes No No
CMA EXAMINATION QUESTIONS Page 26 of 121
MANAGEMENT ADVISORY SERVICES CAPITAL BUDGETING
55
annuity . The discount rate (hurdle rate of return) must be determined in advance for
Annuity due No Yes Yes No the (E)
a. Payback period method. c. Net present value method.
Future Value Formula b. Time adjusted rate of return method. d. Internal rate of return method.
95. Future value is the
56
a. sum of dollars-in discounted to time zero. . Amster Corporation has not yet decided on its hurdle rate for use in the
b. sum of dollars-out discounted to time zero. evaluation of capital budgeting projects. This lack of information will
c. difference of dollars-in and dollars-out. prohibit Amster from calculating a project’s (M)
d. value of dollars-in minus dollars-out for future periods adjusted for any CMA 0693 4-20 a. b. c. d.
interest-compounding factor. Barfield Accounting Rate of No Yes No No
Return
98. Assume that X represents a sum of money that Bill has available to invest Net Present Value No Yes Yes Yes
in a project that will yield a return of r. In the formula Y = X(1 + r), Y Internal Rate of No Yes Yes No
represents the Return
a. future value of X in one period.
b. future value interest factor associated with r. 30. Which capital budgeting technique(s) measure all expected future cash
c. present value of X. inflows and outflows as if they occurred at a single point in time? (E)
d. present value interest factor associated with r. Barfield a. Net present value c. Payback
b. Internal rate of return d. Both (a) and (b). Horngren
Discounted Cash Flow Methods
*. A mechanized system of handling parts from one assembly line to another 2. Which of the following investment rules does not use the time value of the
is being contemplated by the Moonbeam Co. The technical evaluation money concept?
indicated that the system will reduce labor and waiting time costs A. The payback period
substantially. An assessment has to be made of cost/benefit relationship B. Internal rate of return
including the effects of interest. The most relevant quantitative technique C. Net present value
to evaluate the project is (M) D. All of the above use the time value concept B&M
a. Regression analysis. c. Time adjusted rate of return
analysis. BREAKEVEN TIME
b. PERT-CPM. d. Payback period analysis. RPCPA 14. Which of the following statements regarding the discounted payback period
0594 rule is true?
A. The discounted payback rule uses the time value of money concept.
B. The discounted payback rule is better than the NPV rule
C. The discounted payback rule considers all cash flows
D. The discounted payback rule exhibits the value additive property B & M
57
. When evaluating projects, breakeven time is best described as (M)
a. Annual fixed costs  monthly contribution margin.

CMA EXAMINATION QUESTIONS Page 27 of 121


MANAGEMENT ADVISORY SERVICES CAPITAL BUDGETING
59
b. Project investment  annual net cash inflows. . The profitability index (present value index)
c. The point at which cumulative cash inflows on a project equal total cash a. Represents the ratio of the discounted net cash outflows to cash
outflows. inflows.
d. The point at which discounted cumulative cash inflows on a project b. Is the relationship between the net discounted cash inflows less the
equal discounted total cash outflows. CMA 0693 4-28 discounted cash outflows.
c. Is calculated by dividing the discounted profits by the cash outflows.
PROFITABILITY INDEX d. Is the ratio of the discounted net cash inflows to discounted cash
Definition outflows. CMA 0695 4-4
3. The profitability index
a. Does not use present values of cash flows. 19. The profitability index is the ratio of
b. Is generally preferable to any other approach for evaluating mutually a. Total cash inflows to the cost of the investment.
exclusive investment alternatives. b. The present value of cash inflows to the cost of the investment.
c. Produces the same ranking of investment alternatives as does the IRR c. The NPV of the investment to the cost of the investment.
criterion. d. The IRR to the company’s cost of capital. L&H
d. Is a discounted cash flow method. L&H
Assumption
*. Which of the following capital budgeting techniques is computed by 42. Which method of evaluating capital projects assumes that cash inflows can
dividing present value of future inflows by the initial investment? (M) be reinvested at the discount rate?
a. Accounting rate of return c. Payback reciprocal a. internal rate of return c. profitability index
b. Time-adjusted rate of return d. Profitability index RPCPA 0589 b. payback period d. accounting rate of return Barfield

41. The profitability index is (E) Formula


a. the ratio of net cash flows to the original investment. 33. Profitability index is the ratio of:
b. the ratio of the present value of cash flows to the original investment. A. Present value of cash flow to initial investment
c. a capital budgeting evaluation technique that doesn't use discounted B. Net present value cash flow to initial investment
values. C. Net present value of cash flow to IRR
d. a mandatory technique when capital rationing is used. Barfield D. Present value of cash flow to IRR B&M

58
. The technique that reflects the time value of money and is calculated by Benefit Cost Ratio
dividing the present value of the future net after-tax cash inflows that have 34. Benefit cost ratio is defined as:
been discounted at the desired cost of capital by the initial cash outlay for A. Present value of cash flow to initial investment
the investment is the B. Net present value cash flow to initial investment
a. Capital rationing method. c. Profitability index method. C. Net present value of cash flow to IRR
b. Average rate of return method. d. Accounting rate of return. CMA D. Present value of cash flow to IRR B&M
1290 4-14

CMA EXAMINATION QUESTIONS Page 28 of 121


MANAGEMENT ADVISORY SERVICES CAPITAL BUDGETING

NET PRESENT VALUE METHOD


Definition 70. A capital budgeting tool management can use to summarize the difference
60
. The method that recognizes the time value of money by discounting the in the future net cash inflows from an intangible asset at two different
after-tax cash flows over the life of a project, using the company's points in time is referred to as (E)
minimum desired rate of return is the a. the accrual accounting rate-of-return method.
A. Accounting rate of return method. C. Internal rate of return b. the net present value method.
method. c. sensitivity analysis
B. Net present value method. D. Payback method. CMA 1294 4-23 d. the payback method. Horngren
61
. The technique that recognizes the time value of money by discounting the Underlying Theory
after-tax cash flows for a project over its life to time period zero using the *. The excess present value method is anchored on the theory that the future
company’s minimum desired rate of return is the CMA 1290 4-13 returns, expressed in terms of present value, must at least be (E)
a. Net present value method. c. Average rate of return method. a. Equal to the amount of investment c. More than the amount of
b. Payback method. d. Accounting rate of return method. investment
b. Less than the amount of investment RPCPA 1074
22. Probabilistic estimates are most frequently used with which of the following
62
methods of capital expenditure evaluation? . If a firm identifies (or creates) an investment opportunity with a present
A. payback C. internal rate of return value <List A> its cost, the value of the firm and the price of its common
B. present value D. accounting rate of return Carter & stock will <List B> (M)
Usry CIA 1195 IV- a. b. c. d.
44
3. The net present value of a proposed project represents the: List A Greater than Greater Equal to Equal to
A. cash flows less the original investment than
B. present value of the cash flows plus the present value of the original List B Increase Decrease Increase Decrease
investment less the original investment
C. present value of the cash flows less the original investment Characteristics
D. present value of the cash flows less the cost of the old machine being 35. The net present value method of evaluating proposed investments
replaced a. measures a project's internal rate of return.
E. cash flows less the present value of the cash flows Carter & Usry b. ignores cash flows beyond the payback period.
c. applies only to mutually exclusive investment proposals.
34. The capital budgeting method which calculates the expected monetary d. discounts cash flows at a minimum desired rate of return. Barfield
gain or loss from a project by discounting all expected future cash inflows
and outflows to the present point in time using the required rate of return is 63
. In evaluating a capital budget project, the use of the net present value
the (E) (NPV) model is ordinarily not affected by the (E)
a. payback method. a. Method of funding the project.
b. accrual accounting rate-of-return method. b. Initial cost of the project.
c. sensitivity method. c. Amount of added working capital needed for operations during the term
d. net present value method. Horngren of the project.
CMA EXAMINATION QUESTIONS Page 29 of 121
MANAGEMENT ADVISORY SERVICES CAPITAL BUDGETING

d. Project’s salvage value. CMA 1294 4-21 c. Will result in inconsistent errors being made on estimating NPVs such
e. Amount of the project’s associated depreciation tax allowance. that project cannot be evaluated reliably.
d. Results in higher estimate for the IRR on the investment. RPCPA 1095
64
. The capital budgeting technique known as net present value uses
65
AICPA 1180 T-48 a. b. c. d. . The accountant of Ronier, Inc. has prepared an analysis of a proposed
Cash flow over life of No No Yes Yes capital project using discounted cash flow techniques. One manager has
project questioned the accuracy of the results because the discount factors
Time value of money Yes No No Yes employed in the analysis have assumed the cash flows occurred at the end
of the year when the cash flows actually occurred uniformly throughout
42. The net present value method focuses on (E) each year. The net present value calculated by the accountant will
a. cash inflows. c. cash outflows. A. Not be in error.
b. accrual-accounting net income. d. both (a) and (c). Horngren B. Be slightly overstated.
C. Be unusable for actual decision making.
41. The net present value rule is valid for: D. Be slightly understated but usable. CMA 1278 5-10
A. Two period certain cash flows
B. Two period uncertain cash flows 21. The net present value method assumes that all cash inflows can be
C. Uncertain cash flows that extend far into the future immediately reinvested at the
D. All of the above B&M a. cost of capital. c. internal rate of return. Barfield
b. discount rate. d. rate on the corporation's short-
16. Which of the following capital budgeting methods has the value additive term debt.
property?
A. NPV C. Payback period 21. The net present value method of capital budgeting assumes that cash flows
B. IRR D. Discounted payback period B & M are reinvested at: (E)
a. the internal rate of return on the project.
Assumption b. the rate of return on the company's debt.
33. If an analyst desires a conservative net present value estimate, he/she will c. the discount rate used in the analysis.
assume that all cash inflows occur at (M) d. a zero rate of return. CMA adapted
a. mid year. c. year end.
66
b. the beginning of the year. d. irregular intervals. Barfield . The net present value (NPV) method of investment project analysis
assumes that the project’s cash flows are reinvested at the CMA 0692 4-16,
*. The common assumption in capital budgeting analysis is that cash inflows RPCPA 0596
occur in lump sums at the end of individual years during the life of an a. Computed internal rate of return. c. Discount rate used in the NPV
investment project when in fact they flow more or less continuously during calculation.
those years (M) b. Risk-free interest rate. d. Firm’s accounting rate of return.
a. Results in understated estimates of NPV.
67
b. Is done because present value tables for continuous flows cannot be . The net present value method of capital budgeting assumes that cash flows
constructed. are reinvested at

CMA EXAMINATION QUESTIONS Page 30 of 121


MANAGEMENT ADVISORY SERVICES CAPITAL BUDGETING

a. The risk-free rate. c. The internal rate of return of the Advantage


68
project. . An advantage of the net present value method over the internal rate of
b. The cost of debt. d. The discount rate used in the return model in discounted cash flow analysis is that the net present value
analysis. method
CMA 1295 4-9 a. Computes a desired rate of return for capital projects. CMA 0695 4-1
b. Can be used when there is no constant rate of return required for each
36. Which of the following statements is true regarding capital budgeting year of the project.
methods? (D) c. Uses a discount rate that equates the discounted cash inflows with the
a. The Fisher rate can never exceed a company's cost of capital. outflows.
b. The internal rate of return measure used for capital project evaluation d. Uses discounted cash flows whereas the internal rate of return model
has more conservative assumptions than the net present value method, does not.
especially for projects that generate a positive net present value.
c. The net present value method of project evaluation will always provide 28. The net present value capital budgeting technique can be used when cash
the same ranking of projects as the profitability index method. flows from period to period are:
d. The net present value method assumes that all cash inflows can be AICPA A. B. C. D.
reinvested at the project's cost of capital. Barfield adapted
Uniform No No Yes Yes
10. The advantage of the Net Present Value method over the Internal Rate of Uneven Yes No No Yes
Return method for screening investment projects is that it:
a. does not consider the time value of money 53. In situations where the required rate of return is not constant for each year
b. implicitly assumes that the company is able to reinvest cash flows from of the project, it is advantageous to use (E)
the project at the company’s discount rate a. the adjusted rate-of-return method. c. the net present value
c. implicitly assumes that the company is able to reinvest cash flows from method.
the project at the internal rate of return b. the internal rate-of-return method. d. sensitivity analysis.
d. fails to consider the timing of cash flows Pol Bobadilla Horngren

16. The capital budgeting method that assumes that funds are reinvested at Disadvantage
the company's cost of capital is: 69
. A disadvantage of the net present value method of capital expenditure
A. accounting rate of return D. return on investment evaluations is that it (M)
B. net present value E. payback a. Is calculated using sensitivity analysis.
C. internal rate of return AICPA adapted b. Computes the true interest rate.
c. Does not provide the true rate of return on investment.
36. The net present value rule assumes that: d. Is difficult to apply because it uses a trial-and-error approach. CMA 1295
A. Borrowing and lending rate are equal C. Both A and B are true 4-16 RPCPA 0597
B. Financial markets are well functioning D.
None of the above are true B&M Application
35. NPV is appropriate to use to analyze which decision relating to a joint-
products company?
CMA EXAMINATION QUESTIONS Page 31 of 121
MANAGEMENT ADVISORY SERVICES CAPITAL BUDGETING

a. Whether or not to sell facilities now used for additional processing of D. None of the above B&M
one of the joint products.
70
b. Whether or not to acquire facilities needed for additional processing of . The discount rate ordinarily used in present value calculation is the
one of the joint products. a. Federal Reserve rate.
c. Whether or not to sell facilities now used to operate the joint process. b. Treasury bill rate.
d. All of the above. L&H c. Minimum desired rate of return set by the firm.
d. Prime rate. Gleim
Variables
Required Rate of Return 19. A firm's discount rate is typically based on (M)
32. Net present value is calculated using (E) a. the interest rates related to the firm's bonds.
a. the internal rate of return. b. a project's internal rate of return.
b. the required rate of return. c. its cost of capital.
c. the rate of return required by the investment bankers. d. the corporate Aa bond yield. Barfield
d. none of the above. Horngren
20. In capital budgeting, a firm's cost of capital is frequently used as the (M)
13. The rate of return is also called: a. internal rate of return. c. discount rate.
A. Discount rate C. Opportunity cost of capital b. accounting rate of return. d. profitability index. Barfield
B. Hurdle rate D. All of the above. B&M
71
. When using the net present value method for capital budgeting analysis,
39. Which of the following is NOT an appropriate term for the required rate of the required rate of return is called all of the following except the
return? (E) A. Risk-free rate. C. Discount rate.
a. Discount rate c. Cost of capital Horngren B. Cost of capital. D. Cutoff rate. CMA 1292 4-16
b. Hurdle rate d. All of the above are appropriate
72
terms . All of the following are the rates used in net present value analysis except
for the
18. The interest rate used to find the present value of a future cash flow is the a. Cost of capital. c. Discount rate.
a. prime rate. c. cutoff rate. b. Hurdle rate. d. Accounting rate of return. CMA
b. discount rate. d. internal rate of return. Barfield 0694 4-15

54. By using the required rate of return of an equivalent security traded in the Net Investment
73
financial markets as a discount rate in the NPV calculations, we are: . A project’s net present value, ignoring income tax considerations, is
A. Discounting for time C. A and B above normally affected by the
B. Discounting for risk D. None of the above B&M a. Proceeds from the sale of the asset to be replaced.
b. Carrying amount of the asset to be replaced by the project.
52. The discount rate is used for calculating the NPV is: c. Amount of annual depreciation on the asset to be replaced. AICPA 0593
A. Determined by the financial market T-47
B. Found by the government d. Amount of annual depreciation on fixed assets used directly on the
C. Found by the CEO project.
CMA EXAMINATION QUESTIONS Page 32 of 121
MANAGEMENT ADVISORY SERVICES CAPITAL BUDGETING

Formula
Working Capital 50. The present value formula for one period cash flow is:
22. Some investment projects require that a company expand its working A. PV = C1(1 + r) C. PV = C1/(1 + r)
capital to service the greater volume of business that will be generated. B. PV = C1/r D. None of the above B&M
Under the net present value method, the investment of working capital
should be treated as: (M)
51. The net present value formula for one period is:
a. an initial cash outflow for which no discounting is necessary.
A. NPV = PV cash flows - initial investment C. NPV = C0/[C1(1 + r)]
b. a future cash inflow for which discounting is necessary.
c. both an initial cash outflow for which no discounting is necessary and a B. NPV = C0/C1 D. None of the above B&M
future cash inflow for which discounting is necessary.
d. irrelevant to the net present value analysis. G & N 9e Decision Criteria
43. The managers of a firm can maximize stockholder wealth by:
Salvage Value A. Taking all projects with positive NPVs
31. A proposed project has an expected economic life of eight years. In the B. Taking all projects with NPVs greater than the cost of investment
calculation of the net present value (NPV) of the project, salvage value C. Taking all projects with NPVs greater than present value of cash flow
would be: D. All of the above B&M
A. excluded from the calculation of the NPV
B. included as a cash inflow at the estimated salvage value AICPA adapted 43. If the net present value for a project is zero or positive, this means (E)
C. included as a cash inflow at the future amount of the estimated salvage a. the project should be accepted.
value b. the project should not be accepted.
D. included as a cash inflow at the present value of the estimated salvage c. the expected rate of return is below the required rate of return.
value d. both (a) and (c). Horngren

Comprehensive 32. According to the net present value rule, an investment in a project should
19. How are the following used in the calculation of the net present value of a be made if the:
proposed project? Ignore income tax considerations. (M) A. Net present value is greater than the cost of investment
AICPA adapted A. B. C. D. B. Net present value is greater than the present value of cash flows
Depreciation Include Include Exclude Exclude C. Net present value is positive
expense D. Net present value is negative B&M
Salvage value Include Exclude Include Exclude
35. Which of the following statements regarding the net present value rule and
20. The net present value method takes into account: (M) the rate of return rule is true?
AICPA adapted A. B. C. D. A. Accept a project if the rate of return is positive
B. Accept a project the rate of return on a risky project exceeds the risk-
Cash Flow Over Life of No No Yes Yes
free rate
Project
C. Accept a project if the net present value is positive
Time Value of Money Yes No No Yes
D. None of the above statements are true B&M

CMA EXAMINATION QUESTIONS Page 33 of 121


MANAGEMENT ADVISORY SERVICES CAPITAL BUDGETING

38. In using the net present value method, only projects with a zero or positive
74
net present value are acceptable because (E) . When determining net present value in an inflationary environment,
a. the return from these projects equals or exceeds the cost of capital. adjustments should be made to
b. a positive net present value on a particular project guarantees company a. Increase the discount rate, only.
profitability. b. Increase the estimated cash inflows and increase the discount rate.
c. the company will be able to pay the necessary payments on any loans c. Increase the estimated cash inflows but not the discount rate.
secured to finance the project. d. Decrease the estimated cash inflows and increase the discount rate.
d. of both (a) and (b). Horngren CMA 1293 4-21

33. Which of the following statements regarding the net present value rule and 18. Proper treatment of inflation in the NPV calculation involves:
the rate of return rule is not true? A. Discounting nominal cash flows using the nominal discount rate
A. Accept a project if NPV > cost of investment B. Discounting real cash flows using the real discount rate
B. Accept a project if NPV is positive C. Discounting nominal cash flows using the real discount rates
C. Accept a project if return on investment exceeds the rate of return on D. A and B B&M
an equivalent investment in the financial market
D. All of the above statements are true B&M 20. The NPV value obtained by discounting nominal cash flows using the
nominal discount rate is:
17. The following statements regarding the NPV rule and the rate of return rule A. The same as the NPV value obtained by discounting real cash flows
are true except: using the real discount rate
A. Accept a project if its NPV > 0 B. The same as the NPV value obtained by discounting real cash flows
B. Reject a project if its NPV < 0 using the nominal discount rate
C. Accept a project if its rate of return > 0 C. The same as the NPV value obtained by discounting nominal cash flows
D. Accept a project if its rate of return > opportunity cost of capital B & M using the real discount rate
D. None of the above B&M
34. According to the rate of return rule an investment in a risky project should
be made if: INTERNAL RATE OF RETURN
A. The return on investment exceeds the risk-free rate Definition
B. The return on investment is positive *. The discount rate that equates the present value of the expected cash
C. The return on investments exceeds the rate of return on an equivalent flows with the cost of the investment is the (E)
investment in the financial market a. Net present value c. Accounting rate of return
D. None of the above statements are true B&M b. Internal rate of return d. Payback period RPCPA 0593
75
NPV in an Inflationary Environment . The technique that incorporates the time value of money by determining
13. Which of the following statements is true? the compound interest rate of an investment such that the present value of
A. Nominal cash flows are discounted using nominal discount rate the after-tax cash inflows over the life of the investment is equal to the
B. Nominal cash flows are discounted using the real discount rate initial investment is called the
C. Real cash flows are discounted using the nominal discount rate A. Internal rate of return method. C. Profitability index method. CMA
D. None of the above statements are true B&M 1290 4-16
CMA EXAMINATION QUESTIONS Page 34 of 121
MANAGEMENT ADVISORY SERVICES CAPITAL BUDGETING
76
B. Capital asset pricing model. D. Accounting rate of return method. . The internal rate of return (IRR) is the
a. Hurdle rate.
b. Rate of interest for which the net present value is greater than 1.0.
c. Rate of interest for which the net present value is equal to zero.
d. Rate of return generated from the operational cash flows. CMA 0694 4-
16, RPCPA 1096

24. The IRR is defined as: (E)


A. The discount rate that makes the NPV equal to zero
B. The difference between the cost of capital and the present value of the
cash flows
C. The discount rate used in the NPV method
D. The discount rate used in the discounted payback period method B & M

46. The rate of interest that produces a zero net present value when a project's
discounted cash operating advantage is netted against its discounted net
investment is the
a. cost of capital. c. cutoff rate.
b. discount rate. d. internal rate of return. Barfield
77
. The internal rate of return for a project can be determined
A. If the internal rate of return is greater than the firm's cost of capital.
B. Only if the project cash flows are constant. CMA 1293 4-12
C. By finding the discount rate that yields a net present value of zero for
the project.
D. By subtracting the firm's cost of capital from the project's profitability
index.
78
. The internal rate of return is
a. The discount rate at which the NPV of the cash flows is zero.
b. The breakeven borrowing rate for the project in question.
c. The yield rate/effective rate of interest quoted on long-term debt and
other instruments.
d. All of the answers are correct. AICPA 1181 I-39
79
. The internal rate of return is
A. The breakeven borrowing rate for the project in question.

CMA EXAMINATION QUESTIONS Page 35 of 121


MANAGEMENT ADVISORY SERVICES CAPITAL BUDGETING

B. The yield rate/effective rate of interest quoted on long-term debt and Residual sales value of Exclude Include Exclude Include
other instruments. project
C. Favorable when it exceeds the hurdle rate. Depreciation expense Include Include Exclude Exclude
D. All of the answers are correct. Gleim
38. If Co. X wants to use IRR to evaluate long-term decisions and to establish a
17. When a project has uneven projected cash inflows over its life, an analyst cutoff rate of return, X must be sure the cutoff rate is (E)
may be forced to use _______________ to find the project's internal rate of a. At least equal to its cost of capital.
return. b. At least equal to the rate used by similar companies.
a. a screening decision c. a post investment audit c. Greater than the IRR on projects accepted in the past.
b. a trial-and-error approach d. a time line Barfield d. Greater than the current book rate of return. L&H

47. The capital budgeting method that calculates the discount rate at which Assumption/Disadvantage
the present value of expected cash inflows from a project equals the 48. Which of the following capital expenditure planning and control techniques
present value of expected cash outflows is the (E) has been criticized because it might mistakenly imply that earnings are
a. net present value method. reinvested at the rate of return earned by the investment? (M)
b. accrual accounting rate-of-return method. a. payback method c. net present value method
c. payback method. b. accounting rate of return d. internal rate of return Barfield
d. internal rate of return. Horngren
81
. The net present value (NPV) method and the internal rate of return (IRR)
21 The internal rate of return of a capital investment(M) method are used to analyze capital expenditures. The IRR method, as
a. Changes when the cost of capital changes. contrasted with the NPV method,
b. Is equal to the annual net cash flows divided by one half of the project’s A. Is considered inferior because it fails to calculate compounded interest
cost when the cash flows are an annuity. rates.
c. Must exceed the cost of capital in order for the firm to accept the B. Incorporates the time value of money whereas the NPV method does
investment. not.
d. Is similar to the yield to maturity on a bond. C. Assumes that the rate of return on the reinvestment of the cash
e. Statements c and d are correct. Brigham proceeds is at the indicated rate of return of the project analyzed rather
than at the discount rate used.
Variables D. Is preferred in practice because it is able to handle multiple desired
*. The capital budgeting technique known as internal rate of return uses (E) hurdle rates, which is impossible with the NPV method. CMA 1291 4-7
RPCPA 0598 a. b. c. d.
82
Cash flow over entire life of No Yes Yes No . A weakness of the internal rate of return (IRR) approach for determining
project the acceptability of investments is that it (E)
Time value of money Yes Yes No No a. Does not consider the time value of money.
b. Is not a straightforward decision criterion.
80
. How are the following used in the calculation of the internal rate of return c. Implicitly assumes that the firm is able to reinvest project cash flows at
of a proposed project? Ignore income tax considerations. the firm’s cost of capital.
a. b. c. d.
CMA EXAMINATION QUESTIONS Page 36 of 121
MANAGEMENT ADVISORY SERVICES CAPITAL BUDGETING

d. Implicitly assumes that the firm is able to reinvest project cash flows at b. I and II. d. I, II, and III. AICPA 1192 T-49
the project’s internal rate of return. CMA 1292 4-13
Decision Criteria
25. A weakness of the internal rate of return method for screening investment 48. In capital budgeting, a project is accepted only if the internal rate of return
projects is that it: (M) (E)
a. does not consider the time value of money. a. equals or exceeds the required rate of return.
b. implicitly assumes that the company is able to reinvest cash flows from b. equals or is less than the required rate of return.
the project at the company's discount rate. c. equals or exceeds the net present value.
c. implicitly assumes that the company is able to reinvest cash flows from d. equals or exceeds the accrual accounting rate of return. Horngren
the project at the internal rate of return.
d. does not take into account all of the cash flows from a project. CMA Comprehensive
adapted 23. Your company is comparing internal rate of return to net present value
computations as alternative criteria for evaluating potential capital
10. Which of the following capital expenditure planning and control techniques investments. Which of the following best describes these computations?
has been criticized because it might mistakenly imply that earnings are A. The internal rate of return method ignores the initial cost of the
reinvested at the rate of return earned by the investment? investment in its computations.
A. internal rate of return method B. The net present value method ignores the company's cost of capital.
B. accounting rate of return on initial investment method C. The net present value method is more appropriate to use during periods
C. payback method of inflation.
D. average return on investment method D. The two methods will give the same rankings because they both
E. present value method AICPA adapted consider the time value of money. CIA adapted
E. The internal rate of return method assumes that the positive cash flows
25. The following are some of the shortcomings of the IRR method except: (E) generated each year are reinvested at the computed rate of return for
A. IRR is conceptually easy to communicate the investment being evaluated.
B. Projects can have multiple IRRs B&M
C. IRR method cannot distinguish between a borrowing project and a 11. A company is considering the purchase of a new conveyor belt system for
lending project carrying parts and subassemblies from building to building within its plant
D. It is very cumbersome to evaluate mutually exclusive projects using the complex. It is expected that the system will have a useful life of at least ten
IRR method years and that it will substantially reduce labor and waiting-time costs. If
the company's average cost of capital is about 15% and if some evaluation
Advantage must be made of cost/benefit relationships (including the effects of
83
. Which of the following characteristics represent an advantage of the interest) to determine the desirability of the purchase, the most relevant
internal rate of return techniques over the accounting rate of return quantitative technique for evaluating the investment is:
technique in evaluating a project? (M) A. present value (or internal rate of return) analysis
I Recognition of the project’s salvage value. B. Program Evaluation and Review Technique (PERT)
II Emphasis on cash flows. C. accounting rate of return analysis
III Recognition of the time value of money. D. cost-volume-profit analysis
a. I only. c. II and III. E. payback analysis AICPA adapted
CMA EXAMINATION QUESTIONS Page 37 of 121
MANAGEMENT ADVISORY SERVICES CAPITAL BUDGETING

RELATIONSHIP among Payback, ARR, PI, NPV & IRR


*. Statement 1 The internal rate of return is the discount rate that equals ARR vs. IRR
the amount invested at a given date with the present value of 11. Which of the following is a basic difference between the IRR and the book
the expected cash inflows from the investment. rate of return (BRR) criteria for evaluating investments?
Statement 2 If the minimum desired rate of return exceeds the a. IRR emphasizes expenses and BRR emphasizes expenditures.
internal rate of return expected from a project, the project b. IRR emphasizes revenues and BRR emphasizes receipts.
should be rejected. c. IRR is used for internal investments and BRR is used for external
Statement 3 The internal rate of return can be more easily applied to investments.
situations with uneven periodic cash flows than can the net d. IRR concentrates on receipts and expenditures and BRR concentrates
present value. (M) on revenues and expenses. L&H
RPCPA 0592 a. b. c. d.
Statement 1 True False True False Payback & NPV
Statement 2 True False True False 12. If a project has a payback period shorter than its life,
Statement 3 True False False True a. Its NPV may be negative.
b. Its IRR is greater than cost of capital.
MODIFIED INTERNAL RATE OF RETURN c. It will have a positive NPV.
22 Which of the following statements is most correct? The modified IRR d. Its incremental cash flows may not cover its cost. L&H
(MIRR) method: (M)
a. Always leads to the same ranking decision as NPV for independent Payback & IRR
projects. 5. The relationship between payback period and IRR is that
b. Overcomes the problem of multiple rates of return. a. A payback period of less than one-half the life of a project will yield an
c. Compounds cash flows at the cost of capital. IRR lower than the target rate.
d. Overcomes the problems of cash flow timing and project size that lead b. The payback period is the present value factor for the IRR.
to criticism of the regular IRR method. c. A project whose payback period does not meet the company’s cutoff
e. Statements b and c are correct. Brigham rate for payback will not meet the company’s criterion for IRR.
d. None of the above. L&H
84
. Which of the following statements is most correct? (M)
a. The MIRR method will always arrive at the same conclusion as the NPV PI and NPV
method. 31. An investment whose profitability index is 1.00
b. The MIRR method can overcome the multiple IRR problem, while the a. Has an IRR equal tot eh prevailing interest rate.
NPV method cannot. b. Returns to the company only the cash outlay for the investment.
c. The MIRR method uses a more reasonable assumption about c. Has a payback period equal to its useful life.
reinvestment rates than the IRR method. d. Has an NPV of zero. L&H
d. Statements a and c are correct.
e. All of the statements above are correct. Brigham 38. If a project generates a net present value of zero, the profitability index for
the project will
a. equal zero. c. equal -1.
b. equal 1. d. be undefined. Barfield
CMA EXAMINATION QUESTIONS Page 38 of 121
MANAGEMENT ADVISORY SERVICES CAPITAL BUDGETING

d. Cannot be determined without more information. L&H


40. If a project's profitability index is less than 1, the project's (E)
a. discount rate is above its cost of capital. c. payback period is 47. A project has an IRR in excess of the cost of capital. The profitability index
infinite. for this project would be (M)
b. internal rate of return is less than zero. d. a. Less than zero.
net present value is negative. Barfield b. Between zero and one.
c. Greater than one.
39. If the profitability index for a project exceeds 1, then the project's d. Cannot be determined without more information. L&H
a. net present value is positive.
b. internal rate of return is less than the project's discount rate. NPV and IRR
86
c. payback period is less than 5 years. . Polo Co. requires higher rates of return for projects with a life span greater
d. accounting rate of return is greater than the project's internal rate of than 5 years. Projects extending beyond 5 years must earn a higher
return. Barfield specified rate of return. Which of the following capital budgeting
techniques can readily accommodate this requirement? (M)
15. An investment with a positive NPV also has AICPA 0590 T-48 a. b. c. d.
a. A positive profitability index. Internal Rate of Yes No No Yes
b. A profitability index of one. Return
c. A profitability index less than one. Net Present Value No Yes No Yes
d. A profitability index greater than one. L&H
49. If the discount rate that is used to evaluate a project is equal to the
85
. If an investment project has a profitability index of 1.15, the project's internal rate of return, the project's _________ is zero.
a. Project’s internal rate of return is 15%. a. profitability index c. present value of the investment
b. Project’s cost of capital is greater than its internal rate of return. b. internal rate of return d. net present value Barfield
c. Project’s internal rate of return exceeds its net present value.
d. Net present value of the project is positive. CMA 1293 4-11 22. If an investment project (normal project) has an IRR equal to the cost of
capital, the NPV for that project is: (E)
PI & IRR A. Positive C. Zero
21. If the profitability index is less than one, B. Negative D. Unable to be determined B&M
a. The IRR is less than cost of capital. c. The IRR is greater than cost
of capital. 26. If the net present value of a project is zero based on a discount rate of
b. The IRR is the same as cost of capital. d. sixteen percent, then the time-adjusted rate of return: (M)
None of the above is true. L&H a. is equal to sixteen percent.
b. is less than sixteen percent.
48. A project has an IRR less than the cost of capital. The profitability index for c. is greater than sixteen percent.
this project would be d. cannot be determined from the information given. G & N 9e
a. Less than zero.
b. Between zero and one. 4. If the present value of the future cash flows for an investment equals the
c. Greater than one. required investment, the IRR is (D)
CMA EXAMINATION QUESTIONS Page 39 of 121
MANAGEMENT ADVISORY SERVICES CAPITAL BUDGETING

a. Equal to the cutoff rate. 18. An investment has a positive NPV discounting the cash flows at a 14% cost
b. Equal to the cost of borrowed capital. of capital. Which statement is true?
c. Equal to zero. a. The IRR is lower than 14%. c. The payback period is less than
d. Lower than the company’s cutoff rate of return. L&H 14 years.
b. The IRR is higher than 14%. d. The book rate of return is 14%.L &
87
. If a prospective investment has a positive net present value at a company's H
cost of capital of 15%, it can be concluded that
89
A. The accounting rate of return of the project is greater than 15%. . The net present value of a proposed investment is negative; therefore, the
B. The internal rate of return of the project is equal to the accounting rate discount rate used must be
of return. A. Greater than the project's internal rate of return.
C. The payback period of the associated asset is shorter than its life. B. Less than the project's internal rate of return.
D. The internal rate of return of the project is greater than 15%. CIA C. Greater than the firm's cost of equity.
0R98 IV-37 D. Less than the risk-free rate. CMA 1295 4-14, RPCPA 0596

45. If an investment has a positive net present value, the 25. At a company's cost of capital of 15%, a prospective investment has a
a. internal rate of return is higher than the discount rate. negative net present value. Based on this information, it can be concluded
b. discount rate is higher than the hurdle rate of return. that:
c. internal rate of return is lower than the discount rate of return. A. the internal rate of return is greater than 15%
d. hurdle rate of return is higher than the discount rate. Barfield B. the payback period is shorter than the life of the asset
C. the accounting rate of return is less than 15%
7. If an investment has a positive NPV D. the accounting rate of return is greater than 15%
a. Its IRR is greater than the company’s cost of capital. E. the internal rate of return is less than 15% CIA adapted
b. Cost of capital exceeds the cutoff rate of return.
90
c. Its IRR is less than the company’s cutoff rate of return. . Which of the following statements is most correct? (E)
d. The cutoff rate of return exceeds cost of capital. L&H a. If a project’s internal rate of return (IRR) exceeds the cost of capital,
then the project’s net present value (NPV) must be positive.
*. Lenders Inc. is considering an investment that has a positive net present b. If Project A has a higher IRR than Project B, then Project A must also
value based on its 16% hurdle rate. The internal rate of return would be have a higher NPV.
(M) c. The IRR calculation implicitly assumes that all cash flows are reinvested
a. More than 16%. c. 16%. at a rate of return equal to the cost of capital.
b. Less than 16%. d. Zero. RPCPA 1095 d. Statements a and c are correct. Brigham
88 91
. Neu Co. is considering the purchase of an investment that has a positive . Which of the following statements is most correct? (M)
net present value based on Neu’s 12% hurdle rate. The internal rate of a. If a project with normal cash flows has an IRR which exceeds the cost of
return would be capital, then the project must have a positive NPV.
a. 0%. c. >12% b. If the IRR of Project A exceeds the IRR of Project B, then Project A must
b. 12%. d. < 12% also have a higher NPV.
c. The modified internal rate of return (MIRR) can never exceed the IRR.
CMA EXAMINATION QUESTIONS Page 40 of 121
MANAGEMENT ADVISORY SERVICES CAPITAL BUDGETING

d. Statements a and c are correct. Brigham NPV, IRR & WACC


95
. Assume a project has normal cash flows (that is, the initial cash flow is
92
. Project A has an internal rate of return (IRR) of 15 percent. Project B has negative, and all other cash flows are positive). Which of the following
an IRR of 14 percent. Both projects have a cost of capital of 12 percent. statements is most correct? (E)
Which of the following statements is most correct? (E) a. All else equal, a project’s IRR increases as the cost of capital declines.
a. Both projects have a positive net present value (NPV). b. All else equal, a project’s NPV increases as the cost of capital declines.
b. Project A must have a higher NPV than Project B. c. All else equal, a project’s MIRR is unaffected by changes in the cost of
c. If the cost of capital were less than 12 percent, Project B would have a capital.
higher IRR than Project A. d. Statements a and b are correct.
d. Statements a and c are correct. e. Statements b and c are correct. Brigham
e. All of the statements above are correct. Brigham
96
. Which of the following statements is incorrect? (M)
93
. Project A has an IRR of 15 percent. Project B has an IRR of 18 percent. a. Assuming a project has normal cash flows, the NPV will be positive if the
Both projects have the same risk. Which of the following statements is IRR is less than the cost of capital.
most correct? (E) b. If the multiple IRR problem does not exist, any independent project
a. If the WACC is 10 percent, both projects will have a positive NPV, and acceptable by the NPV method will also be acceptable by the IRR
the NPV of Project B will exceed the NPV of Project A. method.
b. If the WACC is 15 percent, the NPV of Project B will exceed the NPV of c. If IRR = k (the cost of capital), then NPV = 0.
Project A. d. NPV can be negative if the IRR is positive.
c. If the WACC is less than 18 percent, Project B will always have a shorter e. The NPV method is not affected by the multiple IRR problem. Brigham
payback than Project A.
97
d. If the WACC is greater than 18 percent, Project B will always have a . A project has an up-front cost of $100,000. The project’s WACC is 12
shorter payback than Project A. percent and its net present value is $10,000. Which of the following
e. If the WACC increases, the IRR of both projects will decline. Brigham statements is most correct? (E)
a. The project should be rejected since its return is less than the WACC.
94
. Project J has the same internal rate of return as Project K. Which of the b. The project’s internal rate of return is greater than 12 percent.
following statements is most correct? (M) c. The project’s modified internal rate of return is less than 12 percent.
a. If the projects have the same size (scale) they will have the same NPV, d. All of the statements above are correct. Brigham
even if the two projects have different levels of risk.
b. If the two projects have the same risk they will have the same NPV, NPV, IRR & Payback
even if the two projects are of different size. 37. A project that has a negative NPV.
c. If the two projects have the same size (scale) they will have the same a. Has a payback period longer than its life.
discounted payback, even if the two projects have different levels of b. Has a negative profitability index.
risk. c. Must be rejected.
d. All of the statements above are correct. d. Doesn’t necessarily fit any of the above descriptions. L&H
e. None of the statements above is correct. Brigham

CMA EXAMINATION QUESTIONS Page 41 of 121


MANAGEMENT ADVISORY SERVICES CAPITAL BUDGETING
98
. Project X has an internal rate of return of 20 percent. Project Y has an b. The project’s MIRR is greater than 10 percent but less than 12 percent.
internal rate of return of 15 percent. Both projects have a positive net c. The project’s payback period is greater than its discounted payback
present value. Which of the following statements is most correct? (M) period.
a. Project X must have a higher net present value than Project Y. d. Statements a and b are correct. Brigham
b. If the two projects have the same WACC, Project X must have a higher
net present value. COMPARISION among Payback, ARR, PI, NPV & IRR
c. Project X must have a shorter payback than Project Y. NPV vs.IRR
d. Statements b and c are correct. 52. An important advantage of the net present value method of capital
e. None of the statements above is correct. Brigham budgeting over the internal rate-of-return method is (E)
a. the net present value method is expressed as a percentage.
PI, NPV & IRR b. the net present values of individual projects can be added to determine
22. Which of the following combinations is possible? L&H the effects of accepting a combination of projects.
Profitability Index NPV IRR c. no advantage.
a. Greater than 1 Positive Equal cost of capital d. both (a) and (b). Horngren
b. Greater than 1 Negative Less that cost of
capital Depreciation
c. Less than 1 Negative Less than cost of 23. (Ignore income taxes in this problem.) How is depreciation handled by the
capital following capital budgeting techniques? (M)
d. Less than 1 Positive Less than cost of CMA adapted A. B. C. D.
capital Internal Rate of Excluded Included Excluded Included
Return
23. Which of the following combinations is NOT possible? (E) L&H Simple Rate of Included Excluded Excluded Included
Profitability Index NPV IRR Return
a. Greater than 1 Positive More than cost of Payback Excluded Included Excluded
capital
b. Equals 1 Zero Equals cost of 19. If income tax considerations are ignored, how is depreciation expense used
capital in the following capital budgeting techniques?
c. Less than 1 Negative Less than cost of AICPA adapted A. B. C. D.
capital Internal Rate of Excluded Included Included Excluded
d. Less than 1 Positive Less than cost of Return
capital Payback Included Excluded Included Excluded

NPV, IRR, MIRR, and Payback *. If income tax considerations are ignored, how is depreciation used in the
99
. A proposed project has normal cash flows. In other words, there is an up- following capital budgeting techniques? (E)
front cost followed over time by a series of positive cash flows. The RPCPA 0595 a. b. c. d.
project’s internal rate of return is 12 percent and its WACC is 10 percent. Internal rate of return Included Excluded Excluded Included
Which of the following statements is most correct? (E) Accounting rate of Excluded Included Excluded Included
a. The project’s NPV is positive.
CMA EXAMINATION QUESTIONS Page 42 of 121
MANAGEMENT ADVISORY SERVICES CAPITAL BUDGETING

return a. If the expected rate of return on a given capital project lies above the
SML, the project should be accepted even if its beta is above the beta of
30. If income tax considerations are ignored, how is depreciation expense used the firm’s average project.
in the following capital budgeting techniques? b. If a project’s return lies below the SML, it should be rejected if it has a
AICPA adapted A. B. C. D. beta greater than the firm’s existing beta but accepted if its beta is
Internal Rate of Excluded Excluded Included Included below the firm’s beta.
Return c. If two mutually exclusive projects’ expected returns are both above the
Net Present Value Excluded Included Excluded Included SML, the project with the lower risk should be accepted.
d. If a project’s expected rate of return is greater than the expected rate
100
. If income tax considerations are ignored, how is depreciation handled by of return on an average project, it should be accepted. Brigham
the following budgeting technique?
CMA 1293 4-17 a. b. c. d. 23. On a graph with common stock returns on the Y axis and market returns on
Internal Rate of Excluded Included Excluded Included the X-axis, the slope of the regression line represents the:
Return A. Alpha C. R-squared
Accounting Rate of Included Excluded Excluded Included B. Beta D. Adjusted beta B&M
Return
Payback Excluded Included Included Included 15. The historical returns data for the past three years for Company A's stock is
-6.0%, 15%, 15% and that of the market portfolio is 10%, 10% and 16%.
Cash Flow According to the SML, the Stock A is:
24. Which of the following capital budgeting techniques consider(s) cash flow A. over priced C. Correctly priced
over the entire life of the project? (E) B. Under priced D. Need more information B&M
AICPA adapted A. B. C. D.
Internal Rate of Yes Yes No No 22. The historical returns data for the past three years for Stock B and the
Return stock market portfolio are: Stock B:- 24%, 0%, 24%, Market Portfolios:-
Payback Yes No Yes No 10%, 12%, 20%. According to the SML the stock B is:
A. Overpriced C. Correctly priced
PROJECT SCREENING (Accept/Reject Decision for Independent Project) B. Underpriced B&M
59. Which of the following best represents a screening decision?
a. determining which project has the highest net present value Single Project
101
b. determining if a project's internal rate of return exceeds the firm's cost . The profitability index approach to investment analysis (M)
of capital A. Fails to consider the timing of project cash flows.
c. determining which projects are mutually exclusive B. Considers only the project's contribution to net income and does not
d. determining which are the best projects Barfield consider cash flow effects.
C. Always yields the same accept/reject decisions for independent projects
Security Market Line (SML) Concept as the net present value method.
. Using the Security Market Line concept in capital budgeting, which of the D. Always yields the same accept/reject decisions for mutually exclusive
following is correct? (M) projects as the net present value method. CMA 1292 4-14, RPCPA 0596

CMA EXAMINATION QUESTIONS Page 43 of 121


MANAGEMENT ADVISORY SERVICES CAPITAL BUDGETING

14. The NPV and IRR methods give 5 14 .5194 3.4331


a. The same decision (accept or reject) for any single investment. 5 15 .4972 3.3522
b. The same choice from among mutually exclusive investments. 6 10 .5645 4.3553
c. Different rankings of projects with unequal lives. 6 14 .4556 3.8887
d. The same rankings of projects with different required investments. L & H 6 15 .4323 3.7845
102
. According to the net present value criterion, which of the following is true?
29. Which of the following is always true of the net present value (NPV) A. Manager one will recommend that the project be accepted.
approach? B. Manager two will recommend that the project be accepted.
A. If a project is found to be acceptable under the NPV approach, it would C. All three managers will recommend acceptance of the project.
also be acceptable under the internal rate of return (IRR) approach. D. All three managers will recommend rejection of the project.
B. The NPV and the IRR approaches will always rank projects in the same
order. 103
. Which manager will assess the project as having the shortest payback
C. If a project is found to be acceptable under the NPV approach, it would period?
also be acceptable under the payback approach. CIA adapted A. Manager one.
D. The NPV and the payback approaches will always rank projects in the B. Manager two.
same order. C. Manager three.
D. All three managers will agree on the payback period.
Questions 140 and 141 are based on the following information. CIA 0594 IV-45
& 46 Group Project
The financial management team of a company is assessing an investment 6. You are given a job to make a decision on project X, which is composed of
proposal involving a $100,000 outlay today. Manager one expects the project three projects A, B, and C which have NPVs of +$50, -$20 and +$100,
to provide cash inflows of $20,000 at the end of each year for 6 years. She respectively. How would you go about making the decision about whether
considers the project to be of low risk, requiring only a 10% rate of return. to accept or reject the project? (M)
Manager two expects the project to provide cash inflows of $5,000 at the end A. Accept the firm's joint project as it has a positive NPV
of the first year, followed by $23,000 at the end of each year in years two B. Reject the joint project
through six. He considers the project to be of medium risk, requiring a 14% C. Break up the project into its components: accept A and C and reject B
rate of return. Manager three expects the project to be of high risk, providing D. None of the above B&M
one large cash inflow of $135,000 at the end of the sixth year. She proposes a
15% rate of return for the project. Unlimited Capital
Additional Information: 104
. A company has unlimited capital funds to invest. The decision rule for the
Numbe Discount Present Value of $1 Annuity of $1 per company to follow in order to maximize shareholders' wealth is to invest in
r Rate (%) Due at the End of n Period for n Periods all projects having a(n)
of Periods (PVIF) (PVIFA) A. Present value greater than zero.
Years B. Net present value greater than zero.
1 10 .9091 .9091 C. Internal rate of return greater than zero. CMA 1293 4-15
1 14 .8772 .8772 D. Accounting rate of return greater than the hurdle rate used in capital
1 15 .8696 .8696 budgeting analyses.
5 10 .6209 3.7908
CMA EXAMINATION QUESTIONS Page 44 of 121
MANAGEMENT ADVISORY SERVICES CAPITAL BUDGETING
105 106
. Future, Inc. is in the enviable situation of having unlimited capital funds. . Barker Inc. has no capital rationing constraint and is analyzing many
The best decision rule, in an economic sense, for it to follow would be to independent investment alternatives. Barker should accept all investment
invest in all projects in which the proposals
A. Accounting rate of return is greater than the earnings as a percent of a. If debt financing is available for them.
sales. b. That have positive cash flows.
B. Payback reciprocal is greater than the internal rate of return. c. That provide returns greater than the after-tax cost of debt.
C. Internal rate of return is greater than zero. d. That have a positive net present value. CMA 1295 4-2
D. Net present value is greater than zero. CMA 1278 5-12
Independent Projects
107
. An organization is using capital budgeting techniques to compare two
independent projects. It could accept one, both, or neither of the projects.
Which of the following statements is true about the use of net-present-
value (NPV) and internal-rate-of-return (IRR) methods for evaluating these
two projects?
a. NPV and IRR criteria will always lead to the same accept or reject
decision for two independent projects.
b. If the first project’s IRR is higher than the organization’s cost of capital,
the first project will be accepted but the second project will not.
c. If the NPV criterion leads to accepting or rejecting the first project, one
cannot predict whether the IRR criterion will lead to accepting or
rejecting the first project.
d. If the NPV criterion leads to accepting the first project, the IRR criterion
will never lead to accepting the first project. CIA 0597 IV-43
108
. Which of the following is always true with regard to the net present value
(NPV) approach?
A. If a project is found to be acceptable under the NPV approach, it would
also be acceptable under the internal rate of return (IRR) approach.
B. The NPV and the IRR approaches will always rank projects in the same
order.
C. If a project is found to be acceptable under the NPV approach, it would
also be acceptable under the payback approach. CIA 0586 IV-29
D. The NPV and payback approaches will always rank projects in the same
order.

PROJECT RANKING METHOD


Capital Rationing
38. Soft rationing is imposed in order to: (E)
CMA EXAMINATION QUESTIONS Page 45 of 121
MANAGEMENT ADVISORY SERVICES CAPITAL BUDGETING

A. Control managers' actions C. Cope with market imperfections B. Disregards discounted cash flows.
B. Limit runaway growth D. A and B B&M C. May produce different rankings from the net present value method on
mutually exclusive projects.
39. Hard rationing is imposed by the: (E) D. Would tend to be reduced if a company used an accelerated method of
A. Market C. Management depreciation for tax purposes rather than the straight-line method. CMA
B. Majority stockholder D. Both B and C B&M 0691 4-19
111
Mutually Inclusive Projects . The rankings of mutually exclusive investments determined using the
63. If management judges one project in a mutually inclusive set to be internal rate of return method (IRR) and the net present value method
acceptable for investment, (NPV) may be different when
a. all the other projects in the set are rejected. a. The lives of the multiple projects are equal and the size of the required
b. only one other project in the set can be accepted. investments are equal.
c. all other projects in the set are also accepted. b. The required rate of return equals that IRR of each project.
d. only one project in the set will be rejected. Barfield c. The required rate of return is higher than the IRR of each project.
d. Multiple projects have unequal lives and the size of the investment for
Mutually Exclusive Projects each project is different. CMA 1292 4-15, RPCPA 1096
Net Present Value (Preferred Method)
7. In choosing from among mutually exclusive investments the manager 50. Why do the NPV method and the IRR method sometimes produce different
should normally select the one with the highest (M) rankings of mutually exclusive investment projects?
a. NPV. c. Profitability index. A. The NPV method does not assume reinvestment of cash flows while the
b. IRR. d. Book rate of return. L&H IRR method assumes the cash flows will be reinvested at the internal
rate of return.
57. Which of the following capital investment models would be preferred when B. The NPV method assumes a reinvestment rate equal to the discount
choosing among mutually exclusive alternatives? (M) rate while the IRR method assumes a reinvestment rate equal to the
a. payback period c. IRR internal rate of return.
b. accounting rate of return d. NPV H&M C. The IRR method does not assume reinvestment of the cash flows while
the NPV assumes the reinvestment rate is equal to the discount rate.
Profitability Index D. The NPV method assumes a reinvestment rate equal to the bank loan
109
. When ranking two mutually exclusive investments with different initial interest rate while the IRR rate method assumes a reinvestment rate
amounts, management should give first priority to the project equal to the discount rate. Pol Bobadilla
A. That generates cash flows for the longer period of time.
B. Whose net after-tax flows equal the initial investment. Ranking Decision
112
C. That has the greater accounting rate of return. . Which mutually exclusive project would you select, if both are priced at
D. That has the greater profitability index. CMA 1291 4-6 $1,000 and your discount rate is 15%; Project A with three annual cash
flows of $1,000, or Project B, with 3 years of zero cash flow followed by 3
NPV & IRR years of $1,500 annually?
110
. The internal rate of return on an investment A. Project A.
A. Usually coincides with the company's hurdle rate. B. Project B.
CMA EXAMINATION QUESTIONS Page 46 of 121
MANAGEMENT ADVISORY SERVICES CAPITAL BUDGETING

C. The IRRs are equal, hence you are indifferent. b. Net present value method. d. Discounted cash flow method.
D. The NPVs are equal, hence you are indifferent. Gleim
Ranking Decision
Independent Projects 37. A company is evaluating three possible investments. Information relating to
Definition the company and the investments follow:
*. The kind of investment project which has no direct relationship with other Fisher rate for the three projects 7%
projects and can therefore be implemented or rejected independently of Cost of capital 8%
others (E) Based on this information, we know that (D)
a. Independent investment project a. all three projects are acceptable.
b. Complimentary investment project b. none of the projects are acceptable.
c. None of these RPCPA 0588 c. the capital budgeting evaluation techniques profitability index, net
present value, and internal rate of return will provide a consistent
Examples ranking of the projects.
60. Below are pairs of projects. Which pair best represents independent d. the net present value method will provide a ranking of the projects that
projects? is superior to the ranking obtained using the internal rate of return
a. buy computer; buy software package method. Barfield
b. buy computer #1; buy computer #2
c. buy computer; buy computer security system *. Several proposed capital projects which are economically acceptable may
d. buy computer; repave parking lot Barfield have to be ranked due to constraints in financial resources. In ranking
these projects, the least pertinent is this statement. (M)
Profitability Index a. If the internal rate of return method is used in the capital rationing
35. Profitability index is useful under: (E) problem, the higher the rate, the better the project.
A. Capital rationing C. Non-normal projects b. In selecting the required rate of return, one may either calculate the
B. Mutually exclusive projects D. None of the above B&M organization’s cost of capital or use a rate generally acceptable in the
industry.
40. The profitability index can be used for ranking projects under: (E) c. A ranking procedure on the basis of quantitative criteria may be
A. Soft capital rationing C. Capital rationing at t = 0 established by specifying a minimum desired rate of return, which rate
B. Hard capital rationing D. Both A and B B&M is used in calculating the net present value of each project.
d. If the net present value method is used, the profitability index is
113
. The recommended technique for evaluating projects when capital is calculated to rank the projects. The lower the index, the better the
rationed and there are no mutually exclusive projects from which to choose project. RPCPA 1094
is to rank the projects by
A. Accounting rate of return. C. Internal rate of return. Profitability Index
B. Payback. D. Profitability index. CMA 0692 4-15 115
. Capital budgeting methods are often divided into two classifications:
project screening and project ranking. Which one of the following is
114
. The technique used to evaluate all possible capital projects of different considered a ranking method rather than a screening method?
dollar amounts and then rank them according to their desirability is the (M) A. Net present value. C. Profitability index.
a. Profitability index method. c. Payback method. CMA 1294 4-26
CMA EXAMINATION QUESTIONS Page 47 of 121
MANAGEMENT ADVISORY SERVICES CAPITAL BUDGETING

B. Time-adjusted rate of return. D. Accounting rate of return. CMA 3 1,000 .70


0691 4-17 The discount rates available for this analysis are: risk-free rate = 5%, cost
of capital = 10%, and risk-adjusted discount rate = 15%. How should these
IRR cash flows be discounted using the certainty-equivalent method (CE) and
116
. A company has analyzed seven new projects, each of which has its own the risk-adjusted discount rate method (RADR)?
internal rate of return. It should consider each project whose internal rate CE RADR
of return is _____ its marginal cost of capital and accept those projects in A. (Col.2xCol.3) at 10% (Col.2xCol.3) at 5%
_____ order of their internal rate of return. B. (Col.2xCol.3) at 5% (Col.2xCol.3) at 10%
A. Below; decreasing. C. Above; increasing. C. (Col.2xCol.3) at 10% Col.2 at 15%
B. Above; decreasing. D. Below; increasing. CIA 0593 IV- D. (Col.2xCol.3) at 5% Col.2 at 15%
55 CIA 0586 IV-32

Internal Rate of Return & Net Present Value 119


. A firm has negotiated a contract with the government and has locked in the
22. The three frequently used methods for ranking investment proposals are payment it will receive in each of the future years from this project.
payback, net present value, and internal rate of return. One of the three is However, the firm's costs for this project are uncertain. How should the
defined as the interest rate that equates the present value of expected certainty-equivalent (CE) approach be applied in this situation?
cash flows and the cost of the project. A second method finds the present A. Discount cash inflows using cost of capital and CE values of cost using
value of expected cash flows and subtracts the initial cost of the project. cost of capital.
The following terms that match these respective definitions are: B. Discount cash inflows using cost of capital and CE values of cost using
A. internal rate of return and net present value risk-free rate.
B. internal rate of return and payback C. Determine net cash inflows using CE values of cost and discount using
C. net present value and internal rate of return cost of capital.
D. net present value and payback CIA adapted D. Determine net cash inflows using CE values of cost and discount using
risk-free rate.
RISK ANALYSIS CIA 0586 IV-34
Risk
Risk-free Rate Uncertainties
117
. The proper discount rate to use in calculating certainty equivalent net 120
. Carco, Inc. wants to use discounted cash flow techniques when analyzing
present value is the (E) its capital investment projects. The company is aware of the uncertainty
a. Risk-adjusted discount rate. d. Cost of equity capital. involved in estimating future cash flows. A simple method some companies
b. Cost of capital. c. Risk-free rate. CMA 1292 4-19 employ to adjust for the uncertainty inherent in their estimates is to
118
A. Prepare a direct analysis of the probability of outcomes.
. The following data are related to the cash flows of a risky capital-budgeting B. Use accelerated depreciation.
alternative: C. Adjust the minimum desired rate of return.
Col. 1 Col. 2 Col. 3 D. Increase the estimates of the cash flows. CMA 1278 5-8
Period Expected Cash Flows Certainty Equivalent Factors
1 1,000 .85 23. To reflect greater uncertainty (greater risk) about a future cash inflow, an
2 1,000 .75 analyst could
CMA EXAMINATION QUESTIONS Page 48 of 121
MANAGEMENT ADVISORY SERVICES CAPITAL BUDGETING

a. increase the discount rate for the cash flow. d. Picking a risk factor equal to the average discount rate.
b. decrease the discounting period for the cash flow. e. Reducing the NPV by 10 percent for risky projects. Brigham
c. increase the expected value of the future cash flow before it is
124
discounted. . Mega Inc., a large conglomerate with operating divisions in many
d. extend the acceptable length for the payback period. Barfield industries, uses risk-adjusted discount rates in evaluating capital
investment decisions. Consider the following statements concerning Mega's
Risk Factor use of risk-adjusted discount rates.
121
. For capital budgeting purposes, management would select a high hurdle I. Mega may accept some investments with internal rates of return less
rate of return for certain projects because management than Mega's overall average cost of capital.
a. Wants to use equity funding exclusively. II. Discount rates vary depending on the type of investment.
b. Believes too many proposals are being rejected. III. Mega may reject some investments with internal rates of return greater
c. Believes bank loans are riskier than capital investments. than the cost of capital.
d Wants to factor risk into its consideration of projects. CMA 1294 4-22 IV. Discount rates may vary depending on the division.
Which of the above statements are correct? (D)
16. In a discounted cash flow analysis, which of the following would not be A. I and III only. C. II, III, and IV only.
consistent with adjusting a project's cash flows to account for higher-than- B. II and IV only. D. I, II, III, and IV.
normal risk? CMA Samp Q4-5
a. increasing the expected amount for cash outflows
125
b. increasing the discounting period for expected cash inflows . Dick Boe Enterprises, an all-equity firm, has a corporate beta coefficient of
c. increasing the discount rate for cash outflows 1.5. The financial manager is evaluating a project with an expected return
d. decreasing the amount for expected cash inflows Barfield of 21 percent, before any risk adjustment. The risk-free rate is 10 percent,
and the required rate of return on the market is 16 percent. The project
Different Risk Levels being evaluated is riskier than Boe’s average project, in terms of both beta
122
. When the risks of the individual components of a project’s cash flows are risk and total risk. Which of the following statements is most correct? (E)
different, an acceptable procedure to evaluate these cash flows is to a. The project should be accepted since its expected return (before risk
a. Divide each cash flow by the payback period. adjustment) is greater than its required return.
b. Compute the net present value of each cash flow using the firm’s cost b. The project should be rejected since its expected return (before risk
of capital. adjustment) is less than its required return.
c. Compare the internal rate of return from each cash flow to its risk. CMA c. The accept/reject decision depends on the risk-adjustment policy of the
1295 4-6 firm. If the firm’s policy were to reduce a riskier-than-average project’s
d. Discount each cash flow using a discount rate that reflects the degree expected return by 1 percentage point, then the project should be
of risk. accepted.
d. Riskier-than-average projects should have their expected returns
Risk-Adjusted Discount Rates increased to reflect their added riskiness. Clearly, this would make the
123
. Risk in a revenue-producing project can best be adjusted for by(E) project acceptable regardless of the amount of the adjustment.
a. Ignoring it. e. Projects should be evaluated on the basis of their total risk alone. Thus,
b. Adjusting the discount rate upward for increasing risk. there is insufficient information in the problem to make an accept/reject
c. Adjusting the discount rate downward for increasing risk. decision. Brigham
CMA EXAMINATION QUESTIONS Page 49 of 121
MANAGEMENT ADVISORY SERVICES CAPITAL BUDGETING

. A company estimates that an average-risk project has a WACC of 10


126
. Assume you are the director of capital budgeting for an all-equity firm. The percent, a below-average risk project has a WACC of 8 percent, and an
firm’s current cost of equity is 16 percent; the risk-free rate is 10 percent; above-average risk project has a WACC of 12 percent. Which of the
and the market risk premium is 5 percent. You are considering a new following independent projects should the company accept? (E)
project that has 50 percent more beta risk than your firm’s assets currently a. Project A has average risk and a return of 9 percent.
have, that is, its beta is 50 percent larger than the firm’s existing beta. The b. Project B has below-average risk and a return of 8.5 percent.
expected return on the new project is 18 percent. Should the project be c. Project C has above-average risk and a return of 11 percent.
accepted if beta risk is the appropriate risk measure? Choose the correct d. All of the projects above should be accepted.
statement. (M) e. None of the projects above should be accepted. Brigham
a. Yes; its expected return is greater than the firm’s cost of capital.
b. Yes; the project’s risk-adjusted required return is less than its expected Risk and Project Selection
return. . If a company uses the same discount rate for evaluating all projects, which
c. No; a 50 percent increase in beta risk gives a risk-adjusted required of the following results is likely? (M)
return of 24 percent. a. Accepting poor, high-risk projects. d. Accepting no projects.
d. No; the project’s risk-adjusted required return is 2 percentage points b. Rejecting good, low-risk projects. e. Statements a and b are correct.
above its expected return. c. Accepting only good, low risk projects. Brigham
e. No; the project’s risk-adjusted required return is 1 percentage point
above its expected return. Brigham . If a typical U. S. company uses the same discount rate to evaluate all
projects, the firm will most likely become (M)
127
. Assume you are the director of capital budgeting for an all-equity firm. The a. Riskier over time, and its value will decline.
firm’s current cost of equity is 16 percent; the risk-free rate is 10 percent; b. Riskier over time, and its value will rise.
and the market risk premium is 5 percent. You are considering a new c. Less risky over time, and its value will rise.
project that has 50 percent more beta risk than your firm’s assets currently d. Less risky over time, and its value will decline.
have, that is, its beta is 50 percent larger than the firm’s existing beta. The e. There is no reason to expect its risk position or value to change over
expected return on the new project is 18 percent. Should the project be time as a result of its use of a single discount rate. Brigham
accepted if beta risk is the appropriate risk measure? Choose the correct
128
statement. (M) . A company estimates that an average-risk project has a WACC of 10
a. Yes; its expected return is greater than the firm’s cost of capital. percent, a below-average risk project has a WACC of 8 percent, and an
b. Yes; the project’s risk-adjusted required return is less than its expected above-average risk project has a WACC of 12 percent. Which of the
return. following independent projects should the company accept? (E)
c. No; a 50 percent increase in beta risk gives a risk-adjusted required a. Project A has average risk and an IRR = 9 percent.
return of 24 percent. b. Project B has below-average risk and an IRR = 8.5 percent.
d. No; the project’s risk-adjusted required return is 2 percentage points c. Project C has above-average risk and an IRR = 11 percent.
above its expected return. d. All of the projects above should be accepted.
e. No; the project’s risk-adjusted required return is 1 percentage point e. None of the projects above should be accepted. Brigham
above its expected return. Brigham

CMA EXAMINATION QUESTIONS Page 50 of 121


MANAGEMENT ADVISORY SERVICES CAPITAL BUDGETING
129
. A firm is considering the purchase of an asset whose risk is greater than IV. Discount rates may vary depending on the division.
the current risk of the firm, based on any method for assessing risk. In Which of the above statements are correct?
evaluating this asset, the decision maker should(E) A. I and III only. C. II, III, and IV only.
a. Increase the IRR of the asset to reflect the greater risk. B. II and IV only. D. I, II, III, and IV. CMA Samp Q4-5
b. Increase the NPV of the asset to reflect the greater risk.
132
c. Reject the asset, since its acceptance would increase the risk of the . Kemp Consolidated has two divisions of equal size: a computer division and
firm. a restaurant division. Stand-alone restaurant companies typically have a
d. Ignore the risk differential if the asset to be accepted would comprise cost of capital of 8 percent, while stand-alone computer companies
only a small fraction of the total assets of the firm. typically have a 12 percent cost of capital. Kemp’s restaurant division has
e. Increase the cost of capital used to evaluate the project to reflect the the same risk as a typical restaurant company, and its computer division
higher risk of the project. Brigham has the same risk as a typical computer company. Consequently, Kemp
estimates that its composite corporate cost of capital is 10 percent. The
130
. Downingtown Industries has an overall (composite) WACC of 10 percent. company’s consultant has suggested that they use an 8 percent hurdle rate
This cost of capital reflects the cost of capital for a Downingtown project for the restaurant division and a 12 percent hurdle rate for the computer
with average risk; however, there are large differences among the projects. division. However, Kemp has chosen to ignore its consultant, and instead,
The company estimates that low-risk projects have a cost of capital of 8 chooses to assign a 10 percent cost of capital to all projects in both
percent and high-risk projects have a cost of capital of 12 percent. The divisions. Which of the following statements is most correct? (M)
company is considering the following projects: a. While Kemp’s decision to not risk adjust its cost of capital will lead it to
Project Expected Return Risk accept more projects in its computer division and fewer projects in its
A 15% High restaurant division, this should not affect the overall value of the
B 12 Average company.
C 11 High b. Kemp’s decision to not risk adjust means that it is effectively
D 9 Low subsidizing its restaurant division, which means that its restaurant
E 6 Low division is likely to become a larger part of the overall company over
Which of the projects will the company select? (E) time.
a. A and B. d. A, B, C, and D. c. Kemp’s decision to not risk adjust means that the company will accept
b. A, B, and C. e. A, B, C, D, and E. too many projects in the computer business and too few projects in the
c. A, B, and D. Brigham restaurant business. This will lead to a reduction in the overall value of
the company.
131
. Mega Inc., a large conglomerate with operating divisions in many d. Statements a and b are correct. Brigham
industries, uses risk-adjusted discount rates in evaluating capital
133
investment decisions. Consider the following statements concerning Mega's . The Barabas Company has an equal amount of low-risk projects, average-
use of risk-adjusted discount rates. risk projects, and high-risk projects. Barabas estimates that the overall
I. Mega may accept some investments with internal rates of return less company’s WACC is 12 percent. This is also the correct cost of capital for
than Mega's overall average cost of capital. the company’s average-risk projects. The company’s CFO argues that, even
II. Discount rates vary depending on the type of investment. though the company’s projects have different risks, the cost of capital for
III. Mega may reject some investments with internal rates of return greater each project should be the same because the company obtains its capital
than the cost of capital.
CMA EXAMINATION QUESTIONS Page 51 of 121
MANAGEMENT ADVISORY SERVICES CAPITAL BUDGETING

from the same sources. If the company follows the CFO’s advice, what is e. All of the statements above are methods of analyzing risk in capital
likely to happen over time? (M) budgeting. Brigham
a. The company will take on too many low-risk projects and reject too
136
many high-risk projects. . Which of the following statements is correct? (M)
b. The company will take on too many high-risk projects and reject too a. Sensitivity analysis is incomplete because it fails to consider the range
many low-risk projects. of likely values of key variables as reflected in their probability
c. Things will generally even out over time, and therefore, the risk of the distributions.
firm should remain constant over time. b. In comparing two projects using sensitivity analysis, the one with the
d. Statements a and c are correct. Brigham steeper lines would be considered less risky, because a small error in
estimating a variable, such as unit sales, would produce only a small
134
. The Oneonta Chemical Company is evaluating two mutually exclusive error in the project’s NPV.
pollution control systems. Since the company’s revenue stream will not be c. The primary advantage of simulation analysis over scenario analysis is
affected by the choice of control systems, the projects are being evaluated that scenario analysis requires a relatively powerful computer, coupled
by finding the PV of each set of costs. The firm’s required rate of return is with an efficient financial planning software package, whereas
13 percent, and it adds or subtracts 3 percentage points to adjust for simulation analysis can be done using a PC with a spreadsheet program
project risk differences. System A is judged to be a high-risk project (it or even a calculator.
might end up costing much more to operate than is expected). System A’s d. Sensitivity analysis is a risk analysis technique that considers both the
risk-adjusted cost of capital is(M) sensitivity of NPV to changes in key variables and the likely range of
a. 10 percent; this might seem illogical at first, but it correctly adjusts for variable values. Brigham
risk where outflows, rather than inflows, are being discounted.
b. 13 percent; the firm’s cost of capital should not be adjusted when Simulation and Sensitivity Analysis
137
evaluating outflow only projects. . A company is deciding whether to purchase an automated machine to
c. 16 percent; since A is more risky, its cash flows should be discounted at manufacture one of its products. Expected net cash flows from this
a higher rate, because this correctly penalizes the project for its high decision depend on several factors, interactions among those factors and
risk. the probabilities associated with different levels of those factors. The
d. Somewhere between 10 percent and 16 percent, with the answer method that the company should use to evaluate the distribution of net
depending on the riskiness of the relevant inflows. cash flows from this decision and changes in net cash flows resulting from
e. Indeterminate, or, more accurately, irrelevant, because for such changes in levels of various factors is
projects we would simply select the process that meets the a. Simulation and sensitivity analysis. c. Correlation analysis.
requirements with the lowest required investment. Brigham b. Linear programming. d. Differential analysis. CIA 1194 III-
61
Methods of Analyzing Risk
135
. Which of the following is not a method for analyzing risk in capital Simulation Analysis
138
budgeting? (E) . A firm is evaluating a large project. It desires to develop not only the best
a. Sensitivity analysis. guess of the outcome of the project, but also a list (or distribution) of
b. Beta, or CAPM, analysis. outcomes that might occur. This firm would best achieve its objective by
c. Monte Carlo simulation. using
d. Scenario analysis. A. The net-present-value (NPV) approach for capital budgeting.
CMA EXAMINATION QUESTIONS Page 52 of 121
MANAGEMENT ADVISORY SERVICES CAPITAL BUDGETING

B. The profitability-index approach for capital budgeting. a. an appropriate response to uncertainty in cash flow projections.
C. Simulation as applied to capital budgeting. b. useful in measuring the variance of the Fisher rate.
D. The internal-rate-of-return (IRR) approach for capital budgeting. CIA c. typically conducted in the post investment audit.
0589 IV-51 d. useful to compare projects requiring vastly different levels of initial
investment. Barfield
139
. A statistical technique used to evaluate possible rates of return for a capital
143
budgeting project is . Sensitivity analysis is used in capital budgeting to
A. Regression analysis. C. Markov chain analysis. A. Estimate a project's internal rate of return.
B. Simulation analysis. D. Gantt charting. CMA 0689 5-15 B. Determine the amount that a variable can change without generating
unacceptable results.
140
. Which of the following statements is most correct? (E) C. Simulate probabilistic customer reactions to a new product.
a. Sensitivity analysis is a good way to measure market risk because it D. Identify the required market share to make a new product viable and
explicitly takes into account the effects of diversification. produce acceptable results. CMA 1293 4-16
b. One advantage of sensitivity analysis relative to scenario analysis is it
explicitly takes into account the probability of certain effects occurring, 24. In capital budgeting, sensitivity analysis is used
whereas scenario analysis does not take into account probabilities. a. To determine whether an investment is profitable.
c. Simulation analysis is a computerized version of scenario analysis that b. To see how a decision would be affected by changes in variables.
uses continuous probability distributions of the input variables. c. To test the relationship of the IRR and NPV.
d. Statements a and b are correct. d. To evaluate mutually exclusive investments. L&H
e. All of the statements above are correct. Brigham
17. Which of the following makes investments more desirable than they had
Sensitivity Analysis been?
141
. Sensitivity analysis, if used with capital projects (M) a. An increase in income tax rate.
a. Is used extensively when cash flows are known with certainty. b. An increase in interest rates.
b. Measures the change in the discounted cash flows when using the c. An increase in the number of years over which assets must be
discounted payback method rather than the net present value method. depreciated.
c. Is a “what-if” technique that asks how a given outcome will change if d. None of the above. L&H
the original estimates of the capital budgeting model are changed.
d. Is a technique used to rank capital expenditure requests. CMA 0695 4-2, 28. Which statement could express the results of a sensitivity analysis of an
RPCPA 0596 investment decision?
a. The NPV of the project is $50,000.
142
. A manager wants to know the effect of a possible change in cash flows on b. A 5% decline in volume will make the project unprofitable.
the net present value of a project. The technique used for this purpose is c. This project ranks third out of the five available.
A. Sensitivity analysis. C. Cost behavior analysis. CMA 1286 d. This project does not meet the cutoff rate of return. L&H
5-4
B. Risk analysis. D. Return on investment analysis. 36. If X Co. expects to get a one-year bank loan to help cover the initial
financing of capital project Q, the analysis of Q should (D)
62. Sensitivity analysis is
CMA EXAMINATION QUESTIONS Page 53 of 121
MANAGEMENT ADVISORY SERVICES CAPITAL BUDGETING

a. Offset the loan against any investment in inventory or receivables d. More people would invest in apartment buildings. L&H
required by the project.
b. Show the loan as an increase in the investment. 47. For a profitable company, an increase in the rate of depreciation on a
c. Show the loan as a cash outflow in the second year of the project’s life. specific project could
d. Ignore the loan. L&H a. increase the project's profitability index.
b. increase the project's payback period.
54. A "what-if" technique that examines how a result will change if the original c. decrease the project's net present value.
predicted data are not achieved or if an underlying assumption changes is d. increase the project's internal rate of return. Barfield
called (E)
a. sensitivity analysis. c. internal rate-of-return analysis. Depreciation and Savings on Cash Operating Costs
Horngren 41. If depreciation of a new asset exceeds its savings in cash operating costs,
b. net present value analysis. d. adjusted rate-of-return analysis. which of the following is true?
a. The project is usually unacceptable.
Monte Carlo simulation b. The annual after-tax cash flow on the new asset will be greater than the
144
. Monte Carlo simulation(M) savings in cash operating costs.
a. Can be useful for estimating a project’s stand-alone risk. c. The project has a negative NPV.
b. Is capable of using probability distributions for variables as input data d. All of the above. L&H
instead of a single numerical estimate for each variable.
c. Produces both an expected NPV (or IRR) and a measure of the riskiness Sale of Old Plant Assets
of the NPV or IRR. 26. Because of idle capacity, a company is considering two assets for sale.
d. Statements a and b are correct. They are identical in all respects except that asset A has a higher tax basis
e. All of the statements above are correct. Brigham than asset B. Only one need be sold now and the market price is the same
for both assets. Which of the following is true?
Net Investment a. The cash flow is greater from selling asset A.
64. All other factors equal, which of the following would affect a project's b. The cash flow is greater from selling asset B.
internal rate of return, net present value, and payback period? (M) c. The cash flow is the same no matter which one is sold.
a. an increase in the discount rate c. an increase in the initial cost of d. It is not possible to determine how the cash flows from sale of the
the project assets will differ. L&H
b. a decrease in the life of the project d. all of the above
Barfield Sale of Old Plant Asset at a Loss
51. When a profitable corporation sells an asset at a loss, the after-tax cash
Tax Effect on Transactions flow on the sale will (D)
Change in Depreciation Rate a. exceed the pre-tax cash flow on the sale.
27. If the tax law were changed so that owners of apartment buildings had to b. be less than the pre-tax cash flow on the sale.
depreciate them over 50 years instead of the current 31.5 years, c. be the same as the pre-tax cash flow on the sale.
a. Rents would rise. d. increase the corporation's overall tax liability. Barfield
b. Rents would fall because annual depreciation charges would fall.
c. Rents would stay about the same.
CMA EXAMINATION QUESTIONS Page 54 of 121
MANAGEMENT ADVISORY SERVICES CAPITAL BUDGETING

50. As the marginal tax rate goes up, the benefit from the depreciation tax
shield Relative Profitability
a. decreases. 24. Investment A has a payback period of 5.4 years, investment B one of 6.7
b. increases. years. From this information we can conclude
c. stays the same. Barfield a. That investment A has a higher NPV than B.
d. can move up or down depending on whether the firm's cost of capital is b. That investment A has a higher IRR than B.
high or low. c. That investment A’s book rate of return is higher than B’s.
d. None of the above. L&H
Payback Period
In general 9. If investment A has a payback period of three years and investment B has a
6. All other factors equal, a large number is preferred to a smaller number for payback period of four years, then
all capital project evaluation measures except (E) a. A is more profitable than B.
a. net present value. c. internal rate of return. b. A is less profitable than B.
b. payback period. d. profitability index. Barfield c. A and B are equally profitable. Barfield
d. the relative profitability of A and B cannot be determined from the
26. Risk can be controlled in capital budgeting situations by assuming a: information given.
A. high accounting rate of return C. high net income
B. large net present value D. short payback period CIA adapted Accounting Rate of Return
25. Investment A has a book rate of return of 26%, investment B one of 18%.
3. In comparing two projects, the _______ is often used to evaluate the relative From this information we can conclude
riskiness of the projects. (D) a. That investment A has a higher NPV than B.
a. payback period c. internal rate of return b. That investment A has a higher IRR than B.
b. net present value d. discount rate Barfield c. That investment A has a shorter payback period than B.
d. None of the above. L&H
Effect of Cash Flow
28. An investment project that requires a present investment of $210,000 will Discount Rate
have cash inflows of "R" dollars each year for the next five years. The Effect of Change in Acceptability of Projects
project will terminate in five years. Consider the following statements 10. All other things being equal, as cost of capital increases
(ignore income tax considerations): a. More capital projects will probably be acceptable.
I. If "R" is less than $42,000, the payback period exceeds the life of the b. Fewer capital projects will probably be acceptable.
project. c. The number of capital projects that are acceptable will change, but the
II. If "R" is greater than $42,000, the payback period exceeds the life of direction of the change is not determinable just by knowing the
the project. direction of the change in cost of capital.
III. If "R" equals $42,000, the payback period equals the life of the project. d. The company will probably want to borrow money rather than issue
(M) stock. L&H
Which statement(s) is (are) true?
a. Only I and II. c. Only II and III.
b. Only I and III. d. I, II, and III. G & N 9e
CMA EXAMINATION QUESTIONS Page 55 of 121
MANAGEMENT ADVISORY SERVICES CAPITAL BUDGETING

Effect on Payback Period, Profitability Index & Accounting Rate of Return


*. Payback period (PP), profitability index (PI), and simple accounting rate of 17. Which of the following events will increase the NPV of an investment
return (SARR) are some of the capital budgeting techniques. What is the involving a new product?
effect of an increase in the cost of capital on these techniques? (M) a. An increase in the income tax rate.
RPCPA a. b. c. d. b. An increase in the expected per-unit variable cost of the product.
0594 c. An increase in the expected annual unit volume of the product.
PP Increase No change No change Decrease d. A decrease in the expected salvage value of equipment. L&H
PI Decrease Decrease Increase No change
SARR Increase No change Decrease No change 55. A project's after-tax net present value is increased by all of the following
except
Present Value Factors a. revenue accruals. c. depreciation deductions.
Effect of Discount Rate b. cash inflows. d. expense accruals. Barfield
30. As the discount rate increases
a. Present value factors increase. Change in Discount Rate
b. Present value factors decrease. 24. A change in the discount rate used to evaluate a specific project will affect
c. Present value factors remain constant. the project's
d. It is impossible to tell what happens to present value factors. L&H a. life. c. net present value.
b. payback period. d. total cash flows. Barfield
Effect of Length of an Annuity
145
31. As the length of an annuity increases . The net present value (NPV) of a project has been calculated to be
a. Present value factors increase. $215,000. Which one of the following changes in assumptions would
b. Present value factors decrease. decrease the NPV? (M)
c. Present value factors remain constant. a. Decrease the estimated effective income tax rate.
d. It is impossible to tell what happen to present value factors. L&H b. Decrease the initial investment amount.
c. Extend the project life and associated cash inflows.
96. All other things being equal, as the time period for receiving an annuity d. Increase the discount rate. CMA 1295 4-7
lengthens,
a. the related present value factors increase. 16. An increase in the discount rate: (M)
b. the related present value factors decrease. a. will increase the present value of future cash flows.
c. the related present value factors remain constant. Barfield b. will have no effect on net present value.
d. it is impossible to tell what happens to present value factors from the c. will reduce the present value of future cash flows.
information given. d. is one method of compensating for reduced risk. G & N 9e

Net Present Value 22. Which of the following changes would not decrease the present value of the
Factors Affecting Net Present Value future depreciation deductions on a specific depreciable asset? (D)
54. The after-tax net present value of a project is affected by a. a decrease in the marginal tax rate
a. tax-deductible cash flows. c. accounting accruals. b. a decrease in the discount rate
b. non-tax-deductible cash flows. d. all of the above. Barfield c. a decrease in the rate of depreciation
CMA EXAMINATION QUESTIONS Page 56 of 121
MANAGEMENT ADVISORY SERVICES CAPITAL BUDGETING

d. an increase in the life expectancy of the depreciable asset Barfield c. A decrease in the discount rate associated with the project.
d. An increase in required net operating working capital.
17. Suppose an investment has cash inflows of R dollars at the end of each e. All of the statements above will increase the project’s NPV. Brigham
year for two years. The present value of these cash inflows using a 12%
discount rate will be: (M) *. You have determined the profitability of a planned project by finding the
a. greater than under a 10% discount rate. present value of all the cash flows from that project. Which of the following
b. less than under a 10% discount rate. would cause the project to look less appealing, that is, have a lower present
c. equal to that under a 10% discount rate. G & N 9e value? (M)
d. sometimes greater than under a 10% discount rate and sometimes less; a. The discount rate increases.
it depends on R. b. The cash flows are extended over a longer period of time.
c. The investment cost decreases without affecting the expected income
24. A firm is evaluating a project that has a net present value of $0 when a and life of the project. RPCPA 0595
discount rate of 8% is used. A discount rate of 10% will result in d. The cash flows are accelerated and the project life is correspondingly
a. a negative net present value shortened.
b. a positive net present value
c. a net present value of $0 *. Velasquez & Co. is considering an investment proposal for P10 million
d. The question cannot be answered based upon the information provided. yielding a net present value of P450,000. The project has a life of 7 years
H&M with salvage value of P200,000. The company uses a discount rate of 12%.
Which of the following would decrease the net present value? (M)
27. A firm is evaluating a project that has a net present value of $0 when a a. Extend the project life and associated cash inflows.
discount rate of 8% is used. A discount rate of 6% will result in b. Increase discount rate to 15%.
a. a negative net present value c. Decrease the initial investment amount to P9.0 million.
b. a positive net present value d. Increase the salvage value. RPCPA 0597
c. a net present value of $0
d. The question cannot be answered based upon the information provided. Effect of Salvage Value
H&M 34. The salvage value of an old lathe is zero. If instead, the salvage value of
the old lathe was $20,000, what would be the impact on the net present
12. Which of the following would decrease the net present value of a project? value of the proposal to purchase a new lathe? (M)
A. A decrease in the income tax rate. a. It would increase the net present value of the proposal.
B. A decrease in the initial investment. b. It would decrease the net present value of the proposal.
C. An increase in the useful life of the project. c. It would not affect the net present value of the proposal.
D. An increase in the discount rate. Pol Bobadilla d. Potentially it could increase or decrease the net present value of the
new lathe. Barfield
146
. Other things held constant, which of the following would increase the NPV
of a project being considered? (E) Timing of Cash Flow Stream
a. A shift from MACRS to straight-line depreciation. 22. Two new products, X and Y, are alike in every way except that the sales of
b. Making the initial investment in the first year rather than spreading it X will start low and rise throughout its life, while those of Y will be the same
over the first three years. each year. Total volumes over their five-year lives will be the same, as will
CMA EXAMINATION QUESTIONS Page 57 of 121
MANAGEMENT ADVISORY SERVICES CAPITAL BUDGETING

selling prices, unit variable costs, cash fixed costs, and investment. The Capital %
NPV of product X
a. Will be less than that of product Y. c. Will be greater than that of NPV Profile for Project 1
product Y. a. Project 2 has a higher internal rate of return that Project 1.
b. Will beNPV
the same as that of product Y. d. None of the above. b. Project 1 has a higher internal rate of return than Project 2.
L&H c. Project 1 has a higher net present value than Project 2.
d. Project 2 has a higher net present value than Project 1.
Income Tax Rate
A
6. Which of the following events is most likely to reduce the expected NPV of 43 Projects A and B have the same expected lives and initial cash outflows.
an investment? However, one project’s cash flows are larger in the early years, while the
a. The major competitor for the product to be manufactured with the other project has larger cash flows in the later years. The two NPV profiles
B
machinery being considered for purchase has been rated are given below:
“unsatisfactory” by a consumer group.
k
b. The interest rate on long-term debt declines.
c. The income tax rate is raised by the Congress.
d. Congress approves the use of faster depreciation than was previously
available. L&H

Expected Returns
147
. Stock C has a beta of 1.2, while Stock D has a beta of 1.6. Assume that the
stock market is efficient. Which of the following statements is most
correct? (E)
a. The required rates of return of the two stocks should be the same.
b. The expected rates of return of the two stocks should be the same.
c. Each stock should have a required rate of return equal to zero. Which of the following statements is most correct? (E)
d. The NPV of each stock should equal its expected return. a. Project A has the smaller cash flows in the later years.
e. The NPV of each stock should equal zero. Brigham b. Project A has the larger cash flows in the later years.
c. We require information on the cost of capital in order to determine
NPV profiles which project has larger early cash flows.
148
. If the net present value profiles for two mutually exclusive capital projects d. The NPV profile graph is inconsistent with the statement made in the
are shaped as in the graph below, which of the following statements is problem.
true? e. None of the statements above is correct. Brigham
$
149
. Projects A and B both have normal cash flows. In other words, there is an
up-front cost followed over time by a series of positive cash flows. Both
projects have the same risk and a WACC equal to 10 percent. However,
NPV Profile Project A has a higher internal rate of return than Project B. Assume that
for Project 2 Cost of
CMA EXAMINATION QUESTIONS Page 58 of 121
MANAGEMENT ADVISORY SERVICES CAPITAL BUDGETING

changes in the WACC have no effect on the projects’ cash flow levels. 6. Two mutually exclusive projects each have a cost of $10,000. The total,
Which of the following statements is most correct? (E) undiscounted cash flows from Project L are $15,000, while the
a. Project A must have a higher net present value than Project B. undiscounted cash flows from Project S total $13,000. Their NPV profiles
b. If Project A has a positive NPV, Project B must also have a positive NPV. cross at a discount rate of 10 percent. Which of the following statements
c. If Project A’s WACC falls, its internal rate of return will increase. best describes this situation? (M)
d. If Projects A and B have the same NPV at the current WACC, Project B a. The NPV and IRR methods will select the same project if the cost of
would have a higher NPV if the WACC of both projects was lower. capital is greater than 10 percent; for example, 18 percent.
Brigham b. The NPV and IRR methods will select the same project if the cost of
capital is less than 10 percent; for example, 8 percent.
150
. Cherry Books is considering two mutually exclusive projects. Project A has c. To determine if a ranking conflict will occur between the two projects
an internal rate of return of 18 percent, while Project B has an internal rate the cost of capital is needed as well as an additional piece of
of return of 30 percent. The two projects have the same risk, the same cost information.
of capital, and the timing of the cash flows is similar. Each has an up-front d. Project L should be selected at any cost of capital, because it has a
cost followed by a series of positive cash flows. One of the projects, higher IRR.
however, is much larger than the other. If the cost of capital is 16 percent, e. Project S should be selected at any cost of capital, because it has a
the two projects have the same net present value (NPV); otherwise, their higher IRR. Brigham
NPVs are different. Which of the following statements is most correct? (E)
151
a. If the cost of capital is 12 percent, Project B will have a higher NPV. . A company is comparing two mutually exclusive projects with normal cash
b. If the cost of capital is 17 percent, Project B will have a higher NPV. flows. Project P has an IRR of 15 percent, while Project Q has an IRR of 20
c. Project B is larger than Project A. percent. If the WACC is 10 percent, the two projects have the same NPV.
d. Statements a and c are correct. Brigham Which of the following statements is most correct? (M)
a. If the WACC is 12 percent, both projects would have a positive NPV.
5. Projects L and S each have an initial cost of $10,000, followed by a series of b. If the WACC is 12 percent, Project Q would have a higher NPV than
positive cash inflows. Project L has total, undiscounted cash inflows of Project P.
$16,000, while S has total undiscounted inflows of $15,000. Further, at a c. If the WACC is 8 percent, Project Q would have a lower NPV than Project
discount rate of 10 percent, the two projects have identical NPVs. Which P.
project’s NPV will be more sensitive to changes in the discount rate? (Hint: d. All of the statements above are correct. Brigham
Projects with steeper NPV profiles are more sensitive to discount rate
152
changes.) (M) . Project C and Project D are two mutually exclusive projects with normal
a. Project S. cash flows and the same risk. If the WACC were equal to 10 percent, the
b. Project L. two projects would have the same positive NPV. However, if the WACC <
c. Both projects are equally sensitive to changes in the discount rate since 10%, Project C has a higher NPV, whereas if the WACC > 10%, Project D
their NPVs are equal at all costs of capital. has a higher NPV. On the basis of this information, which of the following
d. Neither project is sensitive to changes in the discount rate, since both statements is most correct? (M)
have NPV profiles which are horizontal. a. Project D has a higher IRR, regardless of the cost of capital.
e. The solution cannot be determined unless the timing of the cash flows b. If the WACC < 10%, Project C has a higher IRR.
is known. Brigham c. If the WACC < 10%, Project D’s MIRR is less than its IRR.
d. Statements a and c are correct. Brigham
CMA EXAMINATION QUESTIONS Page 59 of 121
MANAGEMENT ADVISORY SERVICES CAPITAL BUDGETING

a. If the cost of capital is 10 percent, each project will have a positive net
7. Your assistant has just completed an analysis of two mutually exclusive present value.
projects. You must now take her report to a board of directors meeting and b. If the cost of capital is 6 percent, Project B has a higher net present
present the alternatives for the board’s consideration. To help you with value than Project A.
your presentation, your assistant also constructed a graph with NPV profiles c. If the cost of capital is 13 percent, Project B has a higher net present
for the two projects. However, she forgot to label the profiles, so you do not value than Project A.
know which line applies to which project. Of the following statements d. Statements a and b are correct.
regarding the profiles, which one is most reasonable? (D) e. Statements a and c are correct. Brigham
a. If the two projects have the same investment cost, and if their NPV
154
profiles cross once in the upper right quadrant, at a discount rate of 40 . Sacramento Paper is considering two mutually exclusive projects. Project A
percent, this suggests that a NPV versus IRR conflict is not likely to has an internal rate of return (IRR) of 12 percent, while Project B has an IRR
exist. of 14 percent. The two projects have the same risk, and when the cost of
b. If the two projects’ NPV profiles cross once, in the upper left quadrant, capital is 7 percent the projects have the same net present value (NPV).
at a discount rate of minus 10 percent, then there will probably not be a Assume each project has an initial cash outflow followed by a series of
NPV versus IRR conflict, irrespective of the relative sizes of the two inflows. Given this information, which of the following statements is most
projects, in any meaningful, practical sense (that is, a conflict which will correct? (E)
affect the actual investment decision). a. If the cost of capital is 13 percent, Project B’s NPV will be higher than
c. If one of the projects has a NPV profile which crosses the X-axis twice, Project A’s NPV.
hence the project appears to have two IRRs, your assistant must have b. If the cost of capital is 9 percent, Project B’s NPV will be higher than
made a mistake. Project A’s NPV.
d. Whenever a conflict between NPV and IRR exist, then, if the two c. If the cost of capital is 9 percent, Project B’s modified internal rate of
projects have the same initial cost, the one with the steeper NPV profile return (MIRR) will be less than its IRR.
probably has less rapid cash flows. However, if they have identical cash d. Statements a and c are correct.
flow patterns, then the one with the steeper profile probably has the e. All of the statements above are correct. Brigham
lower initial cost. Brigham
e. If the two projects both have a single outlay at t = 0, followed by a Profitability Index
series of positive cash inflows, and if their NPV profiles cross in the *. What is the effect of changes in cash inflows, investment cost and cash
lower left quadrant, then one of the projects should be accepted, and outflows on profitability (present value) index (PI) (M)
both would be accepted if they were not mutually exclusive. a. PI will increase with an increase in cash inflows, a decrease in
investment cost, or a decrease in cash outflows.
Cross-Over Rate or Fisher Rate b. PI will increase with an increase in cash inflows, an increase in
153
. Project A and Project B are mutually exclusive projects with equal risk. investment cost, or an increase in cash outflows.
Project A has an internal rate of return of 12 percent, while Project B has an c. PI will decrease with an increase in cash inflows, a decrease in
internal rate of return of 15 percent. The two projects have the same net investment cost, or a decrease in cash outflows.
present value when the cost of capital is 7 percent. (In other words, the d. PI will decrease with an increase in cash outflows, an increase in
“crossover rate” is 7 percent.) Which of the following statements is most investment cost, or an increase in cash inflows. RPCPA 0594
correct? (E)

CMA EXAMINATION QUESTIONS Page 60 of 121


MANAGEMENT ADVISORY SERVICES CAPITAL BUDGETING

36. Which of the following factors increase NPV and IRR?


Internal Rate of Return a. An upward revision in expected annual cash flows.
IRR of Zero b. An upward revision of expected life.
40. If the IRR on an investment is zero, c. An upward revision of the residual value of the long-lived assets being
a. Its NPV is positive. acquired for the project.
b. Its annual cash flows equal its required investment. d. All of the above. L&H
c. It is generally a wise investment.
d. Its cash flows decrease over its life. L&H NPV, IRR, and MIRR
17 Assume a project has normal cash flows (that is, the initial cash flow is
Timing of Cash Flow Stream negative, and all other cash flows are positive). Which of the following
155
. Everything else being equal, the internal rate of return (IRR) of an statements is most correct? (M)
investment project will be lower if (M) a. All else equal, a project’s IRR increases as the cost of capital declines.
a. The investment cost is lower. b. All else equal, a project’s NPV increases as the cost of capital declines.
b. Cash inflows are received later in the life of the project. c. All else equal, a project’s MIRR is unaffected by changes in the cost of
c. Cash inflows are larger. capital.
d. The project has a shorter payback period. CIA 0595 IV-37 d. Statements a and b are correct. Brigham

Cash Outflow and Cash Inflow NPV & Market Value of Stocks
156
43. If the total cash inflows associated with a project exceed the total cash . If a firm identifies (or creates) an investment opportunity with a present
outflows associated with the project, the project's (D) value <List A> its cost, the value of the firm and the price of its common
a. net present value is greater than zero. shares will <List B>.
b. internal rate of return is greater than zero. CIA 1195 IV- A. B. C. D.
c. profitability index is greater than 1. 44
d. payback period is acceptable. Barfield List A Greater than Greater than Equal to Equal to
List B Increase Decrease Increase Decrease
NPV and IRR
Change in Sales and Cost of Capital 157
. The economic value of the firm will rise following an increase in
*. You are engaged by the Baquis Co. to evaluate the introduction of a new A. Net cash flow. C. Unsystematic risk.
product line with an innovative packaging. You computed the net present B. Systematic risk. D. The discount rate. CIA 0591 IV-
value (NPV) and internal rate of return (IRR). If your client would reduce 47
the estimate for its sales of the new product and increase the projected
cost of capital, what would be the impact of these revisions on NPV and Comprehensive
IRR? (M) RPCPA 1094 33. Which of the following is true of an investment?
a. NPV will increase, IRR will increase. c. NPV will increase, IRR will a. The lower the cost of capital, the higher the NPV.
decrease. b. The lower the cost of capital, the higher the IRR.
b. NPV will decrease, IRR will increase. d. NPV will decrease, IRR will c. The longer the project’s life, the shorted its payback period.
decrease. d. The higher the project’s NPV, the shorter its life. L&H

CMA EXAMINATION QUESTIONS Page 61 of 121


MANAGEMENT ADVISORY SERVICES CAPITAL BUDGETING
158
. Lieber Technologies is considering two potential projects, Project X and 8. Which of the following type of projects has average risk?
Project Y. In assessing the projects’ risk, the company has estimated the A. Speculation ventures C. Expansion of existing business
beta of each project and has also conducted a simulation analysis. Their B. New products D. Cost improvement B&M
efforts have produced the following numbers:
Project X Project Y Cost of Capital Consideration
Expected NPV $350,000 $350,000 2. If a firm uses the same company cost of capital for evaluating all projects,
Standard deviation $100,000 $150,000 which of the following is likely?
(NPV) A. Rejecting good low risk projects C. Both A and B
Estimated project 1.4 0.8 B. Accepting poor high risk projects D. Neither A nor B B&M
beta
Estimated correlation of Cash flows are not Cash flows are 3. If a firm uses the same company cost of capital for evaluating all projects,
project’s cash flows highly correlated highly correlated which of the following is likely?
with the cash flows of with the cash flows with the cash flows A. Accepting poor low risk projects. C. Both A and B
the Company’s existing of the existing of the existing B. Rejecting good high risk projects D. Neither A nor B B&M
projects. projects. projects.
Which of the following statements is most correct? (E) 4. Using the company cost of capital to evaluate a project is:
a. Project X has a higher level of stand-alone risk relative to Project Y. A. Always correct
b. Project X has a higher level of corporate risk relative to Project Y. B. Always incorrect
c. Project X has a higher level of market risk relative to Project Y. C. Correct for projects that are about as risky as the average of the firm's
d. Statements b and c are correct. Brigham other assets
D. None of the above B&M
PROJECT EVALUATION & SELECTION
Categories of Projects Project Analysis
5. A firm might categorize its projects into: 1. Project analysis includes the following procedures:
A. Cost improvement projects D. Speculative ventures A. Sensitivity analysis C. Monte Carlo simulation
B. Expansion projects (existing business) E. All of the above B. Break-even analysis D. All of the above B&M
C. New products projects B&M
Break-even Analysis
6. Which of the following types of projects have the highest risk? 16. Firms often calculate a project's break-even sales using book earnings.
A. Speculation ventures C. Expansion of existing business B Generally, break-even sales based on NPV is:
&M A. Higher than the one calculated using book earnings
B. New products D. Cost improvement, (known B. Lower than the one calculated using book earnings
technology) C. Equal to the one calculated using book earnings
D. None of the above B&M
7. Which of the following type of projects has the lowest risk?
A. Speculation ventures C. Expansion of existing business Decision Tree
B. New products D. Cost improvement B&M 32. Which of the following statements applied to decision trees?
A. They are simple to construct and analyze
CMA EXAMINATION QUESTIONS Page 62 of 121
MANAGEMENT ADVISORY SERVICES CAPITAL BUDGETING

B. They should include all possible future events and decisions different time periods
C. They help the financial manager to assess the value of options to C. Simulation models are easy to understand and communicate
abandon or expand the project D. Simulation models enable the financial manager to visualize how
D. All of the above B&M outcomes may be affected if the project is modified B&M

Simulation Models Monte Carlo Simulation


2. Simulation models are useful: 19. Monte Carlo simulation is a tool for considering the:
A. To understand the project better C. To assess the project risk A. Effect of changing one variable on the NPV of the project
B. To forecast expected cash flows D. All of the above B&M B. Effect of changing a limited number of plausible combination of
variables on the NPV of the project
23. Generally, the simulation models for projects are developed using a: C. Effect of changing all possible combinations of variables on the NPV of
A. Computer C. Pair of dice the project
B. Roulette wheel D. Pack of cards B&M D. None of the above B&M

28. The hardest and most important part of a simulation is: 24. Monte Carlo simulation is likely to be most useful:
A. Simulating the cash flows A. If small amounts of funds are at stake
B. Specifying the inter-dependencies B. If large amounts of funds are at stake
C. Specifying probabilities C. If moderate amounts of funds are at stake
D. Specifying the numbers on the roulette wheel B&M D. Regardless of amount at stake B&M

20. Which of the following simulation outputs is likely to be most useful and 25. Monte Carlo simulation is likely to be most useful:
easy to interpret? The output shows the distribution(s) of the project: A. For simple problems
A. Earnings C. Cash flows B. For problems of moderate complexity
B. Internal rate of return D. Profits B&M C. For very complex problems
D. Regardless of the problem's complexity B&M
21. The following statements about simulation models are true except:
A. Simulation models enable the financial manager to analyze risky 26. There are three steps involved in Monte Carlo simulations. One of the
projects without estimating the approximate cost of capital following is not one of them:
B. Simulation models are complex and expensive to develop A. Modeling the project C. Modeling the strategy
C. Simulation models are specific to the project and every project requires B. Specifying probabilities D. Simulating the cash flows B & M
a new simulation model
D. Simulation models usually ignore opportunities to expand or abandon 27. The following is not among the steps involved in the Monte Carlo method:
the project B&M A. Modeling the project
B. Specifying the numbers on the roulette wheel
22. The following statements about simulation models are true except: C. Specifying probabilities
A. Simulation models enable the financial manager to analyze what would D. Simulating the cash flows B&M
happen if the uncertainty about any of the variables were reduced
B. Simulation models take into account the interdependencies between
CMA EXAMINATION QUESTIONS Page 63 of 121
MANAGEMENT ADVISORY SERVICES CAPITAL BUDGETING

29. The pharmaceutical companies have used the following method to analyze
investments in R&D (research and development) of new drugs:
A. Monte Carlo Simulation C. Sensitivity analysis
B. Decision trees D. None of the above B&M

30. The pharmaceutical companies face three types of uncertainty. They are
the following except:
A. Scientific and clinical C. Bureaucratic
B. Production and distribution D. Market success B&M

31. According to the simulation model used by Merck and Company, the
following types of variables are used in their model except:
A. Research and development
B. Manufacturing variables
C. Marketing variables
D. FDA (Food and Drug Administration) variables B&M

Basis for Decision


3. After the completion of project analysis, the final decision on the project
would be from:
A. Sensitivity analysis C. Decision trees
B. Break-even analysis D. NPV B&M

CMA EXAMINATION QUESTIONS Page 64 of 121


MANAGEMENT ADVISORY SERVICES CAPITAL BUDGETING

CMA EXAMINATION QUESTIONS Page 65 of 121


MANAGEMENT ADVISORY SERVICES CAPITAL BUDGETING

Mutually Exclusive Projects


Independent Projects 11. Assume that you are comparing two mutually exclusive projects. Which of
159
. Moynihan Motors has a cost of capital of 10.25 percent. The firm has two the following statements is most correct? (M)
normal projects of equal risk. Project A has an internal rate of return of 14 a. The NPV and IRR rules will always lead to the same decision unless one
percent, while Project B has an internal rate of return of 12.25 percent. or both of the projects are “non-normal” in the sense of having only one
Which of the following statements is most correct? (E) change of sign in the cash flow stream, that is, one or more initial cash
a. Both projects have a positive net present value. outflows (the investment) followed by a series of cash inflows.
b. If the projects are mutually exclusive, the firm should always select b. If a conflict exists between the NPV and the IRR, the conflict can always
Project A. be eliminated by dropping the IRR and replacing it with the MIRR.
c. If the crossover rate (that is, the rate at which the Project’s NPV profiles c. There will be a meaningful (as opposed to irrelevant) conflict only if the
intersect) is 8 percent, Project A will have a higher net present value projects’ NPV profiles cross, and even then, only if the cost of capital is
than Project B. to the left of (or lower than) the discount rate at which the crossover
d. Statements a and b are correct. occurs.
e. Statements a and c are correct. Brigham d. All of the statements above are correct. Brigham
160 162
. A company estimates that its weighted average cost of capital (WACC) is . Jurgensen Medical is considering two mutually exclusive projects with the
10 percent. Which of the following independent projects should the following characteristics:
company accept? (M)  The two projects have the same risk and the same cost of capital.
a. Project A requires an up-front expenditure of $1,000,000 and generates  Both projects have normal cash flows. Specifically, each has an up-
a net present value of $3,200. front cost followed by a series of positive cash flows.
b. Project B has a modified internal rate of return of 9.5 percent.  If the cost of capital is 12 percent, Project X’s IRR is greater than its
c. Project C requires an up-front expenditure of $1,000,000 and generates MIRR.
a positive internal rate of return of 9.7 percent.  If the cost of capital is 12 percent, Project Y’s IRR is less than its MIRR.
d. Project D has an internal rate of return of 9.5 percent. Brigham
 If the cost of capital is 10 percent, the two Project’s have the same NPV.
161 Which of the following statements is most correct? (M)
. Project A has an internal rate of return of 18 percent, while Project B has an
a. Project X’s IRR is greater than 12 percent.
internal rate of return of 16 percent. However, if the company’s cost of
b. Project Y’s IRR is less than 12 percent.
capital (WACC) is 12 percent, Project B has a higher net present value.
c. If the cost of capital is 8 percent, Project X has a lower NPV than Project
Which of the following statements is most correct? (D)
Y.
a. The crossover rate for the two projects is less than 12 percent.
d. All of the statements above are correct.
b. Assuming the timing of the two projects is the same, Project A is
e. None of the statements above is correct. Brigham
probably of larger scale than Project B.
c. Assuming that the two projects have the same scale, Project A probably
Ranking methods
has a faster payback than Project B. 163
. Which of the following statements is correct? (M)
d. Statements a and b are correct.
a. Because discounted payback takes account of the cost of capital, a
e. Statements b and c are correct. Brigham
project’s discounted payback is normally shorter than its regular
payback.

CMA EXAMINATION QUESTIONS Page 66 of 121


MANAGEMENT ADVISORY SERVICES CAPITAL BUDGETING

b. The NPV and IRR methods use the same basic equation, but in the NPV Comprehensive
method the discount rate is specified and the equation is solved for 18 Which of the following statements is most correct? (D)
NPV, while in the IRR method the NPV is set equal to zero and the a. When dealing with independent projects, discounted payback (using a
discount rate is found. payback requirement of 3 or less years), NPV, IRR, and modified IRR
c. If the cost of capital is less than the crossover rate for two mutually always lead to the same accept/reject decisions for a given project.
exclusive projects’ NPV profiles, a NPV/IRR conflict will not occur. b. When dealing with mutually exclusive projects, the NPV and modified
d. If you are choosing between two projects which have the same life, and IRR methods always rank projects the same, but those rankings can
if their NPV profiles cross, then the smaller project will probably be the conflict with rankings produced by the discounted payback and the
one with the steeper NPV profile. regular IRR methods.
e. If the cost of capital is relatively high, this will favor larger, longer-term c. Multiple rates of return are possible with the regular IRR method but not
projects over smaller, shorter-term alternatives because it is good to with the modified IRR method, and this fact is one reason given by the
earn high rates on larger amounts over longer periods. Brigham textbook for favoring MIRR (or modified IRR) over IRR.
d. Statements a and c are correct.
164
. In comparing two mutually exclusive projects of equal size and equal life, e. None of the statements above is correct. Brigham
which of the following statements is most correct? (M)
a. The project with the higher NPV may not always be the project with the Decision-Making
165
higher IRR. . Which of the following statements is correct? (D)
b. The project with the higher NPV may not always be the project with the a. There can never be a conflict between NPV and IRR decisions if the
higher MIRR. decision is related to a normal, independent project, that is, NPV will
c. The project with the higher IRR may not always be the project with the never indicate acceptance if IRR indicates rejection.
higher MIRR. b. To find the MIRR, we first compound CFs at the regular IRR to find the
d. Statements a and c are correct. TV, and then we discount the TV at the cost of capital to find the PV.
e. All of the statements above are correct. Brigham c. The NPV and IRR methods both assume that cash flows are reinvested
at the cost of capital. However, the MIRR method assumes
Ranking conflicts reinvestment at the MIRR itself.
26 Which of the following statements is most correct? (E) d. If you are choosing between two projects which have the same cost,
a. The NPV method assumes that cash flows will be reinvested at the cost and if their NPV profiles cross, then the project with the higher IRR
of capital while the IRR method assumes reinvestment at the IRR. probably has more of its cash flows coming in the later years. Brigham
b. The NPV method assumes that cash flows will be reinvested at the e. A change in the cost of capital would normally change both a project’s
risk-free rate while the IRR method assumes reinvestment at the IRR. NPV and its IRR.
c. The NPV method assumes that cash flows will be reinvested at the cost
166
of capital while the IRR method assumes reinvestment at the risk-free . In an operational audit of the finance department, the auditor observed
rate. that the department always used proper quantitative techniques based on
d. The NPV method does not consider the inflation premium. sound economic assumptions to evaluate proposed alternative capital
e. The IRR method does not consider all relevant cash flows, and investments. However, management did not always choose the investment
particularly cash flows beyond the payback period. Brigham option with the most favorable quantitative assessment. In fact, sometimes
management opted for what appeared to be the third or fourth most
favorable investment. The chief financial officer indicated that
CMA EXAMINATION QUESTIONS Page 67 of 121
MANAGEMENT ADVISORY SERVICES CAPITAL BUDGETING

management ultimately makes a subjective decision as to which *. Which of the following statements is False? (M)
investment is best regardless of which investment option looks best a. The net present value (NPV) of a project with cash flows that come in
according to the quantitative analysis. Which of the following statements is relatively slowly is more sensitive to changes in the discount rate than
most accurate? is the NPV of a project with cash flows that come in rapidly.
A. The approach is justifiable if the economic results of capital investments b. Other things held constant, a decrease in the cost of capital (discount
are highly uncertain. rate) will cause an increase in a project’s internal rate of return.
B. The approach is an irrational, intuitive decision process. c. The IRR method can be used in place of the NPV method for all
C. The approach results in the organization not maximizing its profits. independent projects because the two methods then result in identical
D. The approach is an example of the bounded rationality model of decisions.
decision making whereby managers simplify problems. d. The NPV method is preferred over the IRR method because the NPV
CIA 1195 II-4 method’s reinvestment rate assumption is the correct assumption.
RPCPA 0595
COMPREHENSIVE
167
*. In capital budgeting decision, the following are relevant statements except: . Which of the following is most correct? (M)
(E) a. The NPV and IRR rules will always lead to the same decision in choosing
a. Since resources are scarce, all capital expenditures must be ranked between mutually exclusive projects, unless one or both of the projects
according to priority. are “non-normal” in the sense of having only one change of sign in the
b. The company must be able to define what falls under this category, cash flow stream.
whether they are for expansion, for replacements, or for improvements b. The Modified Internal Rate of Return (MIRR) compounds cash outflows
in operations. at the cost of capital.
c. Capital investments are short-term commitments of resources, and they c. Conflicts between NPV and IRR rules arise in choosing between two
are decided in the same process as operating expenses/ mutually exclusive projects (that each have normal cash flows) when
d. A careful analysis of the economic and non-economic reasons or the cost of capital exceeds the crossover point (that is, the point at
justifications for these investments must be made to arrive at the which the NPV profiles cross).
appropriate decision. RPCPA 0593 d. The discounted payback method overcomes the problems that the
payback method has with cash flows occurring after the payback
*. Which of the following statements is correct? (E) period.
a. One key shortcoming of discounted cash flow method is that they e. None of the statements above is correct. Brigham
ignore the recovery of original investment.
168
b. Although a cash outlay for noncurrent asset such as a machine would . Which of the following statements is most correct? (M)
be considered in a capital budgeting analysis, a cash outlay for working a. The IRR method is appealing to some managers because it produces a
capital item such as inventory would not be considered. rate of return upon which to base decisions rather than a dollar amount
c. To be acceptable, a project’s time adjusted rate of return cannot be less like the NPV method.
than the company’s cost of capital. b. The discounted payback method solves all the problems associated
d. If the net present value of an investment is zero, then the project with the payback method.
should be rejected since it is not providing any return on investment. c. For independent projects, the decision to accept or reject will always be
RPCPA 1095 the same using either the IRR method or the NPV method.
d. Statements a and c are correct.
CMA EXAMINATION QUESTIONS Page 68 of 121
MANAGEMENT ADVISORY SERVICES CAPITAL BUDGETING

e. All of the statements above are correct. Brigham MACRS


16. Under the Tax Reform Act of 1986, the system that increased the number
169
. Which of the following statements is most correct? (M) of property classes and lengthened the recovery periods of most kinds of
a. One of the disadvantages of choosing between mutually exclusive depreciable property is known as:
projects on the basis of the discounted payback method is that you A. MACRS C. ACRS
might choose the project with the faster payback period but with the B. 200% declining balance D. 150% declining balance Carter &
lower total return. Usry
b. Multiple IRRs can occur in cases when project cash flows are normal,
but they are more common in cases where project cash flows are 21. When computing depreciation deductions under the MACRS system,
nonnormal. taxpayers must: (M)
c. When choosing between mutually exclusive projects, managers should a. use the half-year convention under which taxpayers are allowed to take
accept all projects with IRRs greater than the weighted average cost of only a half year's depreciation in the first year of an asset's life.
capital. b. use the half-year convention under which taxpayers are allowed to take
d. Statements a and b are correct. only a half year's depreciation in the last year of an asset's life.
e. All of the statements above are correct. Brigham c. use the half-year convention under which taxpayers are allowed to take
only a half year's depreciation in the first and last years of an asset's
170
. Normal projects C and D are mutually exclusive. Project C has a higher net life.
present value if the WACC is less than 12 percent, whereas Project D has a d. calculate depreciation for partial periods using the exact number of
higher net present value if the WACC exceeds 12 percent. Which of the days if the asset is acquired at some time other than the beginning or
following statements is most correct? (M) end of the fiscal year. G & N 9e
a. Project D has a higher internal rate of return.
b. Project D is probably larger in scale than Project C. 20. Under MACRS, the depreciation on tangible personal property is computed
c. Project C probably has a faster payback. as if the property were placed into service at the:
d. Statements a and c are correct. A. beginning of the year C. midpoint of the year
e. All of the statements above are correct. Brigham B. end of the year D. midpoint of the month Carter &
Usry
MODIFIED ACCELERATED COST RECOVERY SYSTEM (MACRS)
MACRS accelerated depreciation rates should be given for many of these 21. Under MACRS, the depreciation on real property is computed as if the
problems. These rates are provided in the text in Appendix 12A. property were placed into service at the:
A. beginning of the year C. midpoint of the year
ACRS B. end of the year D. midpoint of the month Carter &
15. The system for recovering the cost of capital expenditures through federal Usry
income tax deductions that was required for tangible, depreciable property
placed in service after 1980 is known as: 16. Classifying an asset in a MACRS life category is based on
A. MACRS C. ACRS a. Useful life estimated by the company.
B. 200% declining balance D. 150% declining balance Carter & b. Asset depreciation range (ADR) guidelines.
Usry c. The cost of the asset.
d. Any of the above factors. L&H
CMA EXAMINATION QUESTIONS Page 69 of 121
MANAGEMENT ADVISORY SERVICES CAPITAL BUDGETING

b. The same NPV and IRR for the project.


17. An example of 5-year property under MACRS is: c. A higher NPV for the project.
A. most manufacturing machinery C. commercial aircraft d. Lower total cash flows over the 10 years. L&H
B. railroad cars D. light trucks Carter & Usry
13. If a company uses a five-year MACRS period to depreciate assets instead of
18. An example of 7-year property under MACRS is: a 10-year life with straight-line depreciation,
A. automobiles C. light trucks a. The NPV of the investment is higher.
B. most manufacturing machinery D. small aircraft Carter & Usry b. The IRR of the investment is lower.
c. There is no difference in either NPV or IRR.
19. An example of 27.5-year property under MACRS is: d. Total cash flows over the useful life would be lower. L&H
A. residential rental property C. nonresidential buildings
171
B. commercial aircraft D. railroad cars Carter & Usry . Flex Corporation is studying a capital acquisition proposal in which newly
acquired assets will be depreciated using the straight-line method. Which
20. With respect to income taxes, the principal advantage of MACRS over one of the following statements about the proposal would be incorrect if a
straight-line depreciation is that switch is made to the Modified Accelerated Cost Recovery System
a. Total taxes will be lower under MACRS. (MACRS)? CMA 0693 4-29
b. Taxes will be constant from year to year under MACRS. A. The net present value will increase. C. The payback period will be
c. Taxes will be lower in the earlier years under MACRS. shortened.
d. Taxes will decline in future years under MACRS. L&H B. The internal rate of return will increase. D. The profitability
index will decrease.
4. Companies using MACRS for tax purposes and straight-line depreciation for
172
financial reporting purposes usually find that the relationship between the . Which of the following statement completions is incorrect? For a profitable
tax basis and book value of their assets is firm, when MACRS accelerated depreciation is compared to straight-line
a. The tax basis is lower than book value. depreciation, MACRS accelerated allowances produce (M)
b. The tax basis is higher than book value. a. Higher depreciation charges in the early years of an asset’s life.
c. The tax basis is the same as book value. b. Larger cash flows in the earlier years of an asset’s life.
d. None of the above. L&H c. Larger total undiscounted profits from the project over the project’s life.
d. Smaller accounting profits in the early years, assuming the company
5. A company that wants to use MACRS for tax purposes must uses the same depreciation method for tax and book purposes.
a. Request permission from the IRS. e. None of the statements above. (All of the statements above are
b. Acquire new assets to or near the middle of the year. correct.) Brigham
c. Ignore salvage value in calculating depreciation.
d. Do none of the above. L&H Questions 74 and 75 are based on the following information. CMA 0694 4-13 &
14
38. A company evaluates a project using straight-line depreciation over its 10- The tax impact of equipment depreciation affects capital budgeting decisions.
year estimated useful life and then reevaluates it using a 7-year MACRS Currently, the Modified Accelerated Cost Recovery System (MACRS) is used as
class life. The second analysis will show the depreciation method for most assets for tax purposes.
a. A lower IRR for the project.
CMA EXAMINATION QUESTIONS Page 70 of 121
MANAGEMENT ADVISORY SERVICES CAPITAL BUDGETING
173
. The MACRS method of depreciation for assets with 3, 5, 7, and 10-year c. The use of both MACRS tables and the optional straight-line method
recovery periods is most similar to which one of the following depreciation requires that salvage value be deducted in computing depreciation
methods used for financial reporting purposes? deductions.
A. Straight-line. C. Sum-of-the-years'-digits. d. None of the above are true. G & N 9e
B. Units-of-production. D. 200% declining-balance.
Taxes on gain on sale
174 175
. When employing the MACRS method of depreciation in a capital budgeting . St. John’s Paper is considering purchasing equipment today that has a
decision, the use of MACRS as compared with the straight-line method of depreciable cost of $1 million. The equipment will be depreciated on a
depreciation will result in MACRS 5-year basis, which implies the following depreciation schedule:
A. Equal total depreciation for both methods. Year MACRS Depreciation Rate
B. MACRS producing less total depreciation than straight line. 1 0.20
C. Equal total tax payments, after discounting for the time value of money. 2 0.32
D. MACRS producing more total depreciation than straight line. 3 0.19
4 0.12
MACRS vs. Optional Straight-line Method 5 0.11
18. In a capital budgeting decision, the use of MACRS tables as compared to 6 0.06
the optional straight-line method will result in: (M) Assume that the company sells the equipment after three years for
a. equal total depreciation for both methods. $400,000 and the company’s tax rate is 40 percent. What would be the tax
b. more total depreciation for the MACRS tables method. consequences resulting from the sale of the equipment? (E)
c. more total depreciation for the optional straight-line method. CMA a. There are no tax consequences.
adapted b. The company would have to pay $44,000 in taxes.
d. less depreciation for the MACRS tables method in the early years of c. The company would have to pay $160,000 in taxes.
asset life. d. The company would receive a tax credit of $124,000.
e. The company would receive a tax credit of $48,000. Brigham
19. The use of the MACRS tables instead of the optional straight-line method of
depreciation has the effect of: (M) Risk-adjusted discount rate
176
a. raising the hurdle rate necessary to justify the project. . Dick Boe Enterprises, an all-equity firm, has a corporate beta coefficient of
b. decreasing the net present value of the project. 1.5. The financial manager is evaluating a project with an IRR of 21
c. increasing the present value of the depreciation tax shield. percent, before any risk adjustment. The risk-free rate is 10 percent, and
d. increasing the cash outflows at the beginning of the project. CMA the required rate of return on the market is 16 percent. The project being
adapted evaluated is riskier than Boe’s average project, in terms of both beta risk
and total risk. Which of the following statements is most correct? (E)
20. Which of the following is correct? (M) a. The project should be accepted since its IRR (before risk adjustment) is
a. Use of the MACRS tables requires that salvage value be deducted in greater than its required return.
computing depreciation deductions. b. The project should be rejected since its IRR (before risk adjustment) is
b. Use of the optional straight-line method requires that salvage value not less than its required return.
be considered in computing depreciation deductions.

CMA EXAMINATION QUESTIONS Page 71 of 121


MANAGEMENT ADVISORY SERVICES CAPITAL BUDGETING

c. The accept/reject decision depends on the risk-adjustment policy of the b. 10% e. 16%
firm. If the firm’s policy were to reduce a riskier-than-average project’s c. 12% Brigham
IRR by 1 percentage point, then the project should be accepted.
d. Riskier-than-average projects should have their IRRs increased to New project NPV
179
reflect their added riskiness. Clearly, this would make the project . Given the following information, calculate the NPV of a proposed project:
acceptable regardless of the amount of the adjustment. Cost = $4,000; estimated life = 3 years; initial decrease in accounts
e. Projects should be evaluated on the basis of their total risk alone. Thus, receivable = $1,000, which must be restored at the end of the project’s life;
there is insufficient information in the problem to make an accept/reject estimated salvage value = $1,000; earnings before taxes and depreciation
decision. Brigham = $2,000 per year; method of depreciation = MACRS; tax rate = 40
percent; and cost of capital = 18 percent. (MACRS table required) (M)
Risk-adjusted discount rate a. $1,137 d. $ 804
177
. The Unlimited, a national retailing chain, is considering an investment in b. -$ 151 e. $ 544
one of two mutually exclusive projects. The discount rate used for Project A c. $ 137 Brigham
is 12 percent. Further, Project A costs $15,000, and it would be depreciated
180
using MACRS. It is expected to have an after-tax salvage value of $5,000 at . Mars Inc. is considering the purchase of a new machine that will reduce
the end of 6 years and to produce after-tax cash flows (including manufacturing costs by $5,000 annually. Mars will use the MACRS
depreciation) of $4,000 for each of the 6 years. Project B costs $14,815 and accelerated method to depreciate the machine, and it expects to sell the
would also be depreciated using MACRS. B is expected to have a zero machine at the end of its 5-year operating life for $10,000. The firm
salvage value at the end of its 6-year life and to produce after-tax cash expects to be able to reduce net operating working capital by $15,000
flows (including depreciation) of $5,100 each year for 6 years. The when the machine is installed, but required net operating working capital
Unlimited’s marginal tax rate is 40 percent. What risk-adjusted discount will return to the original level when the machine is sold after 5 years.
rate will equate the NPV of Project B to that of Project A? (M) Mars’ marginal tax rate is 40 percent, and it uses a 12 percent cost of
a. 15% d. 20% capital to evaluate projects of this nature. If the machine costs $60,000,
b. 16% e. 12% what is the project’s NPV? (MACRS table required) (M)
c. 18% Brigham a. -$15,394 d. -$21,493
b. -$14,093 e. -$46,901
178
. California Mining is evaluating the introduction of a new ore production c. -$58,512 Brigham
process. Two alternatives are available. Production Process A has an initial
181
cost of $25,000, a 4-year life, and a $5,000 net salvage value, and the use . Stanton Inc. is considering the purchase of a new machine that will reduce
of Process A will increase net cash flow by $13,000 per year for each of the manufacturing costs by $5,000 annually and increase earnings before
4 years that the equipment is in use. Production Process B also requires an depreciation and taxes by $6,000 annually. Stanton will use the MACRS
initial investment of $25,000, will also last 4 years, and its expected net method to depreciate the machine, and it expects to sell the machine at
salvage value is zero, but Process B will increase net cash flow by $15,247 the end of its 5-year operating life for $10,000 before taxes. Stanton’s
per year. Management believes that a risk-adjusted discount rate of 12 marginal tax rate is 40 percent, and it uses a 9 percent cost of capital to
percent should be used for Process A. If California Mining is to be indifferent evaluate projects of this type. If the machine’s cost is $40,000, what is the
between the two processes, what risk-adjusted discount rate must be used project’s NPV? (MACRS table required) (M)
to evaluate B? (D) a. $1,014 d. $ 817
a. 8% d. 14% b. $2,292 e. $5,040
CMA EXAMINATION QUESTIONS Page 72 of 121
MANAGEMENT ADVISORY SERVICES CAPITAL BUDGETING

c. $7,550 Brigham  If MacDonald goes ahead with the new business inventories will rise by
$500,000 at t = 0, and its accounts payable will rise by $200,000 at t =
182
. Maple Media is considering a proposal to enter a new line of business. In 0. This increase in net operating working capital will be recovered at t =
reviewing the proposal, the company’s CFO is considering the following 4.
facts:  The new business is expected to have an economic life of four years.
 The new business will require the company to purchase additional The business is expected to generate sales of $3 million at t = 1, $4
fixed assets that will cost $600,000 at t = 0. For tax and accounting million at t = 2, $5 million at t = 3, and $2 million at t = 4. Each year,
purposes, these costs will be depreciated on a straight-line basis over operating costs excluding depreciation are expected to be 75 percent of
three years. (Annual depreciation will be $200,000 per year at sales.
t = 1, 2, and 3.)  The company’s tax rate is 40 percent.
 At the end of three years, the company will get out of the business  The company’s weighted average cost of capital is 10 percent.
and will sell the fixed assets at a salvage value of $100,000.  The company is very profitable, so any accounting losses on this project
 The project will require a $50,000 increase in net operating working can be used to reduce the company’s overall tax burden.
capital at t = 0, which will be recovered at t = 3. What is the expected net present value (NPV) of the new business? (M)
 The company’s marginal tax rate is 35 percent. a. $ 740,298 d. -$1,961,833
 The new business is expected to generate $2 million in sales each b. -$1,756,929 e. –$5,919,974
year (at t = 1, 2, and 3). The operating costs excluding deprecia-tion c. -$1,833,724 Brigham
are expected to be $1.4 million per year.
184
 The project’s cost of capital is 12 percent. . Rio Grande Bookstores is considering a major expansion of its business.
What is the project’s net present value (NPV)? (M) The details of the proposed expansion project are summarized below:
a. $536,697 d. $ 56,331  The company will have to purchase $500,000 in equipment at t = 0.
b. $ 86,885 e. $561,609 This is the depreciable cost.
c. $ 81,243 Brigham  The project has an economic life of four years.
 The cost can be depreciated on a MACRS 3-year basis, which implies
183
. MacDonald Publishing is considering entering a new line of business. In the following depreciation schedule:
analyzing the potential business, their financial staff has accumulated the Year MACRS Depreciation Rate
following information: 1 0.33
 The new business will require a capital expenditure of $5 million at t = 2 0.45
0. This expenditure will be used to purchase new equipment. 3 0.15
 This equipment will be depreciated according to the following 4 0.07
depreciation schedule:  At t = 0, the project requires that inventories increase by $50,000 and
Year MACRS Depreciation Rate accounts payable increase by $10,000. The change in net operating
1 0.33 working capital is expected to be fully recovered at t = 4.
2 0.45  The project’s salvage value at the end of four years is expected to be
3 0.15 $0.
4 0.07
 The equipment will have no salvage value after four years.
CMA EXAMINATION QUESTIONS Page 73 of 121
MANAGEMENT ADVISORY SERVICES CAPITAL BUDGETING

 The company forecasts that the project will generate $800,000 in sales 1 0.33
the first two years (t = 1 and 2) and $500,000 in sales during the last 2 0.45
two years (t = 3 and 4). 3 0.15
 Each year the project’s operating costs excluding depreciation are 4 0.07
expected to be 60 percent of sales revenue. At the end of four years the company expects to be able to sell the
 The company’s tax rate is 40 percent. equipment for an after-tax salvage value of $10,000. The company is in the
 The project’s cost of capital is 10 percent. 40 percent tax bracket. The company has an after-tax cost of capital of 11
What is the net present value (NPV) of the proposed project? (M) percent. Because there is more uncertainty about the salvage value, the
a. $159,145 d. $150,776 company has chosen to discount the salvage value at 12 percent. What is
b. $134,288 e. -$257,060 the net present value (NPV) of purchasing the equipment? (D)
c. $162,817 Brigham a. $ 9,140.78 d. $22,853.90
b. $16,498.72 e. $28.982.64
185
. Foxglove Corp. is faced with an investment project. The following c. $20,564.23 Brigham
information is associated with this project: 187
. Lugar Industries is considering an investment in a proposed project that
Year Net Income* MACRS Depreciation
requires an initial expenditure of $100,000 at t = 0. This expenditure can
Rate
be depreciated at the following annual rates:
1 $50,000 0.33
Year MACRS Depreciation Rate
2 60,000 0.45
1 0.20
3 70,000 0.15
2 0.32
4 60,000 0.07
3 0.19
*Assume no interest expenses and a zero tax rate.
4 0.12
The project involves an initial investment of $100,000 in equipment that
5 0.11
falls in the 3-year MACRS class and has an estimated salvage value of
6 0.06
$15,000. In addition, the company expects an initial increase in net
The project has an economic life of six years. The project’s revenues are
operating working capital of $5,000 that will be recovered in Year 4. The
forecasted to be $90,000 a year. The project’s operating costs (not
cost of capital for the project is 12 percent. What is the project’s net
including depreciation) are forecasted to be $50,000 a year. After six years,
present value? (Round your final answer to the nearest whole dollar.) (D)
the project’s estimated salvage value is $10,000. The company’s WACC is
a. $153,840 d. $168,604
10 percent, and its corporate tax rate is 40 percent. What is the project’s
b. $159,071 e. $182,344
net present value (NPV)? (D)
c. $162,409 Brigham
a. $31,684 d. $38,840
186 b. $33,843 e. $45,453
. Pierce Products is deciding whether it makes sense to purchase a new
c. $34,667 Brigham
piece of equipment. The equipment costs $100,000 (payable at t = 0). The
equipment will provide before-tax cash inflows of $45,000 a year at the end 188
. Mills Mining is considering an expansion project. The proposed project has
of each of the next four years (t = 1, 2, 3, and 4). The equipment can be
the following features:
depreciated according to the following schedule:
 The project has an initial cost of $500,000. This is also the amount
Year MACRS Depreciation Rate
that can be depreciated using the following depreciation schedule:
CMA EXAMINATION QUESTIONS Page 74 of 121
MANAGEMENT ADVISORY SERVICES CAPITAL BUDGETING
190
Year MACRS Depreciation Rate . An all-equity firm is analyzing a potential project that will require an initial,
1 0.33 after-tax cash outlay of $50,000 and after-tax cash inflows of $6,000 per
2 0.45 year for 10 years. In addition, this project will have an after-tax salvage
3 0.15 value of $10,000 at the end of Year 10. If the risk-free rate is 6 percent, the
4 0.07 return on an average stock is 10 percent, and the beta of this project is
 If the project is undertaken, at t = 0 the company will need to 1.50, what is the project’s NPV? (M)
increase its inventories by $50,000, and its accounts payable will rise a. $13,210 d. -$ 6,158
by $10,000. This net operating working capital will be recovered at the b. $ 4,905 e. -$12,879
end of the project’s life (t = 4). c. $ 7,121 Brigham
 If the project is undertaken, the company will realize an additional
191
$600,000 in sales over each of the next four years (t = 1, 2, 3, and 4). . Real Time Systems Inc. is considering the development of one of two
The company’s operating cost (not including depreciation) will equal mutually exclusive new computer models. Each will require a net
$400,000 a year. investment of $5,000. The cash flows for each project are shown below:
 The company’s tax rate is 40 percent. Year Project A Project B
 At t = 4, the project’s economic life is complete, but it will have a 1 $2,000 $3,000
salvage value of $50,000. 2 2,500 2,600
 The project’s WACC = 10 percent. 3 2,250 2,900
What is the project’s net present value (NPV)? (D) Model B, which will use a new type of laser disk drive, is considered a high-
a. $11,122.87 d. $68,336.86 risk project, while Model A is an average-risk project. Real Time adds 2
b. $50,330.14 e. $80,035.52 percentage points to arrive at a risk-adjusted cost of capital when eval-
c. $54,676.59 Brigham uating high-risk projects. The cost of capital used for average-risk projects
is 12 percent. Which of the following statements regarding the NPVs for
Risk-adjusted NPV Models A and B is most correct? (M)
189
. Virus Stopper Inc., a supplier of computer safeguard systems, uses a cost a. NPVA = $380; NPVB = $1,815 c. NPVA = $380; NPVB = $1,590
of capital of 12 percent to evaluate average-risk projects, and it adds or b. NPVA = $197; NPVB = $1,590 d. NPVA = $5,380; NPVB = $6,590
subtracts 2 percentage points to evaluate projects of more or less risk. Brigham
Currently, two mutually exclusive projects are under consideration. Both
have a cost of $200,000 and will last 4 years. Project A, a riskier-than- NPV and risk-adjusted discount rate
192
average project, will produce annual end-of-year cash flows of $71,104. . Garcia Paper is deciding whether to build a new plant. The proposed project
Project B, a less-than-average-risk project, will produce cash flows of would have an up-front cost (at t = 0) of $30 million. The project’s cost can
$146,411 at the end of Years 3 and 4 only. Virus Stopper should accept(M) be depreciated on a straight-line basis over three years. Consequently, the
a. B with a NPV of $10,001. depreciation expense will be $10 million in each of the first three years, t =
b. Both A and B because both have NPVs greater than zero. 1, 2, and 3. Even though the project is depreciated over three years, the
c. B with a NPV of $8,042. project has an economic life of five years.
d. A with a NPV of $7,177. The project is expected to increase the company’s sales by $20 million.
e. A with a NPV of $15,968. Brigham Sales will remain at this higher level for each year of the project (t = 1, 2,
3, 4, and 5). The operating costs, not including depreciation, equal 60
percent of the increase in annual sales. The project’s interest expense is $5
CMA EXAMINATION QUESTIONS Page 75 of 121
MANAGEMENT ADVISORY SERVICES CAPITAL BUDGETING

million per year and the company’s tax rate is 40 percent. The company is
very profitable, so any accounting losses on this project can be used to Risky projects
194
reduce the company’s overall tax burden. The project does not require any . Cochran Corporation has a weighted average cost of capital of 11 percent
additions to net operating working capital. The company estimates that the for projects of average risk. Projects of below-average risk have a cost of
project’s after-tax salvage value at t = 5 will be $1.2 million. The project is capital of 9 percent, while projects of above-average risk have a cost of
of average risk, and, therefore, the CFO has decided to discount the capital equal to 13 percent. Projects A and B are mutually exclusive,
operating cash flows at the company’s overall WACC of 10 percent. whereas all other projects are independent. None of the projects will be
However, the salvage value is more uncertain, so the CFO has decided to repeated. The following table summarizes the cash flows, internal rate of
discount it at 12 percent. What is the net present value (NPV) of the return (IRR), and risk of each of the projects.
proposed project? (D) Year Project Project B Project C Project D Project E
a. $11.86 million d. -$12.55 million A
b. $14.39 million e. -$ 1.18 million 0 - -$100,000 - -$100,000 -$100,000
c. -$26.04 million Brigham $200,00 $100,000
0
Discounting risky outflows 1 66,000 30,000 30,000 30,000 40,000
193
. Alabama Pulp Company (APC) can control its environmental pollution using 2 66,000 30,000 30,000 30,000 25,000
either “Project Old Tech” or “Project New Tech.” Both will do the job, but 3 66,000 40,000 30,000 40,000 30,000
the actual costs involved with Project New Tech, which uses unproved, new 4 66,000 40,000 40,000 50,000 35,000
state-of-the-art technology, could be much higher than the expected cost
levels. The cash outflows associated with Project Old Tech, which uses IRR 12.110 14.038% 10.848% 16.636% 11.630%
standard proven technology, are less risky. (They are about as uncertain as %
the cash flows associated with an average project.) APC’s cost of capital for Projec Below Below Average Above Above
average-risk projects is normally set at 12 percent, and the company adds t Average Average Average Average
3 percent for high-risk projects but subtracts 3 percent for low-risk projects. Risk
The two projects in question meet the criteria for high and average risk, but Which projects will the firm select for investment? (M)
the financial manager is concerned about applying the normal rule to such a. Projects: A, B, C, D, E d. Projects: A, D
cost-only projects. You must decide which project to recommend, and you b. Projects: B, C, D, E e. Projects: B, C, D
should recommend the one with the lower PV of costs. What is the PV of c. Projects: B, D Brigham
costs of the better project? (M)
Cash Outflows Scenario analysis
Years: 0 1 2 3 4 195
. Klott Company encounters significant uncertainty with its sales volume and
Project New 1,500 315 315 315 315 price in its primary product. The firm uses scenario analysis in order to
Tech determine an expected NPV, which it then uses in its budget. The base-
Project Old 600 600 600 600 600 case, best-case, and worst-case scenarios and probabilities are provided in
Tech the table below. What is Klott’s expected NPV, standard deviation of NPV,
a. 2,521 d. 2,543 and coefficient of variation of NPV? (M)
b. 2,399 e. 2,422 Probability of Unit Sales Sales Price NPV
c. 2,457 Brigham
CMA EXAMINATION QUESTIONS Page 76 of 121
MANAGEMENT ADVISORY SERVICES CAPITAL BUDGETING

Outcome Volume (In c. $21,000


Thousands)
199
Worst 0.30 6,000 $3,600 -$6,000 . What is the project’s NPV? (M)
case a. $2,622 d. $5,712
Base 0.50 10,000 4,200 +13,000 b. $2,803 e. $6,438
case c. $2,917
Best case 0.20 13,000 4,400 +28,000
a. Expected NPV = $35,000; σNPV = 17,500; CVNPV = 2.00 Questions 54 thru 57 are based on the following information. Brigham
b. Expected NPV = $35,000; σNPV = 11,667; CVNPV = 0.33 You have been asked by the president of your company to evaluate the
c. Expected NPV = $10,300; σNPV = 12,083; CVNPV = 1.17 proposed acquisition of a new special-purpose truck. The truck’s basic price is
d. Expected NPV = $13,900; σNPV = 8,476; CVNPV = 0.61 $50,000, and it will cost another $10,000 to modify it for special use by your
e. Expected NPV = $10,300; σNPV = 13,900; CVNPV = 1.35 Brigham firm. The truck falls into the MACRS 3-year class, and it will be sold after three
years for $20,000. Use of the truck will require an increase in net operating
Questions 50 thru 53 are based on the following information Brigham working capital (spare parts inventory) of $2,000. The truck will have no effect
The president of Real Time Inc. has asked you to evaluate the proposed on revenues, but it is expected to save the firm $20,000 per year in before-tax
acquisition of a new computer. The computer’s price is $40,000, and it falls operating costs, mainly labor. The firm’s marginal tax rate is 40 percent.
into the MACRS 3-year class. Purchase of the computer would require an (MACRS table required)
increase in net operating working capital of $2,000. The computer would
200
increase the firm’s before-tax revenues by $20,000 per year but would also . What is the net investment in the truck? (That is, what is the Year 0 net
increase operating costs by $5,000 per year. The computer is expected to be cash flow?) (E)
used for three years and then be sold for $25,000. The firm’s marginal tax rate a. -$50,000 d. -$62,000
is 40 percent, and the project’s cost of capital is 14 percent. (MACRS table b. -$52,600 e. -$65,000
required) c. -$55,800
201
196
. What is the net investment required at t = 0? (E) . What is the operating cash flow in Year 1? (M)
a. -$42,000 d. -$37,600 a. $17,820 d. $20,121
b. -$40,000 e. -$36,600 b. $18,254 e. $21,737
c. -$38,600 c. $19,920
202
197
. What is the operating cash flow in Year 2? (M) . What is the total value of the terminal year non-operating cash flows at the
a. $ 9,000 d. $13,453 end of Year 3? (M)
b. $10,240 e. $16,200 a. $10,000 d. $16,000
c. $11,687 b. $12,000 e. $18,000
c. $15,680
198
. What is the total value of the terminal year non-operating cash flows at the
203
end of Year 3? (M) . The truck’s cost of capital is 10 percent. What is its NPV? (M)
a. $18,120 d. $25,000 a. -$1,547 d. $ 562
b. $19,000 e. $27,000 b. -$ 562 e. $1,034
CMA EXAMINATION QUESTIONS Page 77 of 121
MANAGEMENT ADVISORY SERVICES CAPITAL BUDGETING

c. $ 0

ANSWER EXPLANATIONS

CMA EXAMINATION QUESTIONS Page 78 of 121


1
. Answer (D) is correct. Capital budgeting is the process of planning expenditures for long-
lived assets. It involves choosing among investment proposals using a ranking procedure.
Evaluations are based on various measures involving rate of return on investment.
Answer (A) is incorrect because capital budgeting involves long-term investment needs, not
immediate operating needs. Answer (B) is incorrect because strategic planning establishes
long-term goals in the context of relevant factors in the firm's environment. Answer (C) is
incorrect because cash budgeting determines operating cash flows. Capital budgeting
evaluates the rate of return on specific investment alternatives.
2
. Answer (D) is correct. Capital budgeting is a long-term planning process for investments.
This process begins with the identification of capital needs, that is, of projects required to
achieve organizational goals. The next step is to search for specific investments. The third
step is to acquire and analyze information about the potential choices. The fourth step is to
select specific investments after considering both qualitative and quantitative factors. The
fifth step is to finance the undertakings. The final step is implementation and monitoring.
Answer (A) is incorrect because analyzing capital addition proposals is a step that is
subsequent to identifying capital addition projects and other capital needs. Answer (B) is
incorrect because making expenditure decisions is a step subsequent to identifying capital
addition projects and other capital needs. Answer (C) is incorrect because analyzing and
evaluating all promising alternatives is a step that is subsequent to identifying capital
addition projects and other capital needs.
3
. Answer (C) is correct. Capital budgeting is concerned with long-range decisions, such as
whether to add a product line, to build new facilities, or to lease or buy equipment. Any
decision regarding cash inflows and outflows over a period of more than 1 year probably
needs capital budgeting analysis.
Answer (A) is incorrect because capital budgeting is useful for all long-range decision
making. Answer (B) is incorrect because capital budgeting is not useful for short-range
decisions. Answer (D) is incorrect because it is a nonsense answer.
4
. Answer (D) is correct. The capital budgeting process is a method of planning the efficient
expenditure of the firm's resources on capital projects. Such planning is essential in view of
the rising costs of scarce resources.
Answer (A) is incorrect because capital budgeting may also be used for analysis of multiple
profitable alternatives and of lease-or-buy decisions. Answer (B) is incorrect because capital
budgeting permits analysis of adding or discontinuing product lines or facilities and of lease-
or-buy decisions. Answer (C) is incorrect because the lease-or-buy decision is just one
specific example of an appropriate use of capital budgeting techniques.
5
. Answer (D) is correct. Capital budgeting is the process of planning expenditures for
investments on which the returns are expected to occur over a period of more than 1 year.
Thus, capital budgeting concerns the acquisition or disposal of long-term assets and the
financing ramifications of such decisions. The adoption of a new method of allocating
nontraceable costs to product lines has no effect on a company's cash flows, does not
concern the acquisition of long-term assets, and is not concerned with financing. Hence,
capital budgeting is irrelevant to such a decision.
Answer (A) is incorrect because a new aircraft represents a long-term investment in a capital
good. Answer (B) is incorrect because a major advertising program is a high cost investment
with long-term effects. Answer (C) is incorrect because a star quarterback is a costly asset
who is expected to have a substantial effect on the team's long-term profitability.
6
. Answer (C) is correct. The investment tax credit is of no concern because it no longer
exists. The 1986 Tax Reform Act eliminated the investment tax credit.
Answer (A) is incorrect because the availability of any necessary financing should be
considered even though the net present value method indicates that the project is
acceptable. Answer (B) is incorrect because the probability of near-term technological
changes to the manufacturing process should be considered even though the net present
value method indicates that the project is acceptable. Answer (D) is incorrect because
maintenance requirements, warranties, and availability of service arrangements should be
considered even though the net present value method indicates that the project is
acceptable.
7
. Estimating cash flows Answer: b Diff: M
8
. Answer (D) is correct. The investment in a new project includes more than the initial cost
of new capital equipment. In addition, funds must be provided for increases in receivables
and inventories. This investment in working capital is treated as an initial cost of the
investment that will be recovered in full at the end of the project's life.
Answer (A) is incorrect because the investment in working capital will be needed throughout
the life of the investment. Answer (B) is incorrect because cash will be needed to fund the
investments in receivables and inventory. Answer (C) is incorrect because the initial
investment should be treated as an initial cash outflow, but one that will be recovered at the
end of the project.
9
. Relevant cash flows Answer: b Diff: E N

Sunk costs are never included in project cash flows, so statement a is


false. Externalities are always included, so statement b is true. Since
the weighted average cost of capital includes the cost of debt, and this
is the discount rate used to evaluate project cash flows, interest expense
should not be included in project cash flows. Therefore, statement c is
false.
10
. Relevant cash flows Answer: d Diff: E N

Sunk costs should be ignored, but externalities and opportunity costs


should be included in the project evaluation. Therefore, the correct
choice is statement d.

11
. Answer (B) is correct. Tax depreciation is relevant to cash flow analysis because it affects
the amount of income taxes that must be paid. However, book depreciation is not relevant
because it does not affect the amount of cash generated by an investment.
Answer (A) is incorrect because it is a true statement relating to capital budgeting. Answer
(C) is incorrect because it is a true statement relating to capital budgeting. Answer (D) is
incorrect because it is a true statement relating to capital budgeting.
12
. Relevant cash flows Answer: c Diff: E
The correct answer is c. Sunk costs should be excluded from the analysis,
and interest expense is incorporated in the WACC and not the cash flows.
13
. Cash flows and accounting measures Answer: d Diff: M
14. Relevant cash flows Answer:
d Diff: E
Statements a and c are correct; therefore, statement d is the correct
answer. Net cash flow = Net income + depreciation; therefore, depreciation
affects operating cash flows. Sunk costs should be disregarded when making
investment decisions, while opportunity costs should be considered when
making investment decisions, as they represent the best alternative use of
an asset.

15

. Relevant cash flows Answer: c Diff: E


16
. Relevant cash flows Answer: d Diff: M
Statements b and c are correct; therefore, statement d is the correct
answer. The $3 million spent on researching the technology is a sunk cost.
17
. Relevant cash flows Answer: d Diff: M N

Statement a is a sunk cost and sunk costs are never included in the
capital budgeting analysis. Therefore, statement a is not included.
Statement b is an opportunity cost and should be included in the capital
budgeting analysis. Statement c is the cannibalization of existing
products, which will cause the company to forgo cash flows and profits in
another division. Therefore, it is included in the capital budgeting
analysis. Therefore, the correct answer is statement d.
18
. Relevant cash flows Answer: d Diff: M
Statements b and c are correct; therefore, statement d is the correct
answer. The cost of clearing the land is a sunk cost and should not be
considered in the analysis. The expected impact of the new store on the
existing store should be considered. In addition, the opportunity to lease
the land represents an opportunity cost of opening a new store on the land
and should be considered.
19

. Relevant cash flows Answer: d Diff: M


Statements a and c are correct; therefore, statement d is correct.
Externalities and opportunity costs should be considered, while sunk costs
should not be included in the analysis.
20
. Cash flow estimation Answer: d Diff: M
Statement d is true--the forgone rent is an “opportunity cost” which
should be charged to the project under consideration. Note that Statements
a and b are both false—the cash flows should not take account of interest,
because financial costs are dealt with by discounting at the WACC. If
interest were deducted to find cash flows, then this cost would be “double
counted,” and the NPV would be downward biased. Ignoring interest when
determining cash flows produces no bias in the NPV whatever. Note also
that externalities can be either positive or negative—they tend to be
negative if the new project is a substitute for existing products, but
positive if the new project is complementary to the firm’s other products.
21
. Incremental cash flows Answer: d Diff: M
22
. Incremental cash flows Answer: d Diff: M
23
. Answer (B) is correct. A tax shield is something that will protect income against taxation.
Thus, a depreciation tax shield is a reduction in income taxes due to a company's being
allowed to deduct depreciation against otherwise taxable income.
Answer (A) is incorrect because a tax shield is not a cash flow, but a means of reducing
outflows for income taxes. Answer (C) is incorrect because cash is not provided by recording
depreciation; the shield is a result of deducting depreciation from taxable revenues. Answer
(D) is incorrect because depreciation is recognized as an expense even if it has no tax
benefit.
24
. Answer (C) is correct. A depreciation deduction will reduce a firm's tax by the amount of
the deduction times the marginal tax rate. A dollar deducted is offset against the firm's last
(marginal) dollar of income.
Answer (A) is incorrect because the savings will apply at the marginal rate. Without the
deduction, income would be higher and therefore subject to the marginal tax rate. Answer
(B) is incorrect because the marginal rate is relevant to tax savings from depreciation.
Answer (D) is incorrect because one minus the firm's marginal tax rate times the
depreciation amount describes the effect on income, not taxes.
25
. Answer (C) is correct. Accelerated depreciation results in greater depreciation in the
early years of an asset's life compared with the straight-line method. Thus, accelerated
depreciation results in lower income tax expense in the early years of a project and higher
income tax expense in the later years. By effectively deferring taxes, the accelerated
method increases the present value of the depreciation tax shield.
Answer (A) is incorrect because the hurdle rate can be reached more easily as a result of the
increased present value of the depreciation tax shield. Answer (B) is incorrect because the
greater depreciation tax shield increases the NPV. Answer (D) is incorrect because greater
initial depreciation reduces the cash outflows for the taxes, but has no effect on the initial
cash outflows.
26
. Answer (D) is correct. According to microeconomic theory, a firm should produce until its
marginal revenue equals its marginal cost. In capital budgeting terms, marginal revenue is
the marginal rate of return on investment, and marginal cost is the company's marginal cost
of capital (MCC). Hence, the firm should continue to invest until the cost of the last
investment equals the return.
Answer (A) is incorrect because the firm must balance cost and return. Minimizing MCC or
average cost of capital (ACC is minimized when it equals MCC) ignores possible returns.
Answer (B) is incorrect because the firm must balance cost and return. Minimizing MCC or
average cost of capital (ACC is minimized when it equals MCC) ignores possible returns.
Answer (C) is incorrect because the rate of return on total assets is an average return.
Setting MCC equal to this rate may result in acceptance of poor investments.
27
. REQUIRED: The determinant of the optimal capital budget.
DISCUSSION: (A) In economics, a basic principle is that a firm should increase output until
marginal cost equals marginal revenue. Similarly, the optimal capital budget is determined
by calculating the point at which marginal cost of capital (which increases as capital
requirements increase) and marginal efficiency of investment (which decreases if the most
profitable projects are accepted first) intersect.
Answer (B) is incorrect because the intersection of average marginal cost with average
projected rates of return when the largest (not most profitable) projects are accepted first
offers no meaningful capital budgeting conclusion. Answer (C) is incorrect because the
optimal capital budget may exclude profitable projects as lower cost capital goes first to
projects with higher rates of return. Answer (D) is incorrect because accepting projects with
rates of return lower than the cost of capital is not rational.
28
. Find the WACCs using both John’s and Becky’s methods. (WACC = k s because
there is no debt).

John’s WACC for Division B based on overall company’s beta:


k = kRF + RPM(b)
k = 5% + 5%(1.2)
k = 5% + 6%
k = 11%.

Therefore, John would only choose Project 1, because it is the only project whose IRR
exceeds its cost of capital. Consequently, the firm’s capital budget (based on John’s WACC)
is only $400 million.

Becky’s WACC for Division B:


k = kRF + RPM(b)
k = 5% + 5%(0.9)
k = 5% + 4.5%
k = 9.5%.

Becky would choose projects 1, 2, 3, and 4 because all of these projects have an IRR that
exceeds the Division’s 9.5 percent cost of capital. Based on Becky’s WACC, the firm’s capital
budget would be $1,270 million ($400 + $300 + $250 + $320). Therefore, the firm’s capital
budget based on Becky’s WACC is $870 million ($1,270 - $400) larger than the one based on
John’s WACC.
29
. The option to abandon will increase expected cash flow and decrease risk. If a
firm has the option to abandon a project, it will choose to do so only when things
look bad (negative NPV). Thus, abandoning a project eliminates the low/negative
cash flows. Therefore, statement b is correct.
30
. By having the ability to wait and see you reduce the risk of the project. Therefore,
statement a is false. The greater the uncertainty, the more value there is in waiting for
additional information before going on with a project. Therefore, statement b is false.
Statement c is not necessarily true. By waiting to do a project you may lose strategic
advantages associated with being the first competitor to enter a new line of business, which
may alter the cash flows. Since statements a, b, and c are false, the correct choice is
statement e.
31
. Statements a, b, c, and d are all examples of different types of real options. A flexibility
option permits the firm to alter operations depending on how conditions change during the
life of the project. Typically, either inputs or outputs, or both, can be changed. Statement a
is an example of an investment timing option, while statement b is an example of an
abandonment option. Statement c is an example of a flexibility option, while statement d is
an example of a growth option. Therefore, statement c is the correct choice.
32
. By failing to consider both abandonment and growth options, the firm’s capital budget
would be too small. In both cases, the firm might reject what might otherwise be profitable
projects if these options had been considered. Therefore, the correct choice is statement a.
33
. Answer (D) is correct. The accounting rate of return (unadjusted rate of return or book
value rate of return) equals accounting net income divided by the required initial or average
investment. The accounting rate of return ignores the time value of money.
Answer (A) is incorrect because the net present value is the sum of the present values of all
the cash inflows and outflows associated with an investment. Answer (B) is incorrect
because the discounted payback method calculates the payback period by determining the
present values of the future cash flows. Answer (C) is incorrect because the internal rate of
return is the discount rate at which the NPV is zero.
34
. Answer (D) is correct. The accounting rate of return (also called the unadjusted rate of
return or book value rate of return) measures investment performance by dividing the
accounting net income by the average investment in the project. This method ignores the
time value of money.
Answer (A) is incorrect because the bail-out payback method measures the length of the
payback period when the periodic cash inflows are combined with the salvage value.
Answer (B) is incorrect because the internal rate of return method determines the rate at
which the NPV is zero. Answer (C) is incorrect because the profitability index is the ratio of
the present value of future net cash inflows to the initial cash investment.
35
. Answer (D) is correct. The accounting rate of return is calculated by dividing the annual
after-tax net income from a project by the book value of the investment in that project. The
time value of money is ignored.
Answer (A) is incorrect because the average rate of return method does not divide by the
average investment cost. Answer (B) is incorrect because the internal rate of return
incorporates the time value of money into the calculation. The IRR is the discount rate that
results in a net present value of zero. Answer (C) is incorrect because the capital asset
pricing model is a means of determining the cost of capital.
36
. Answer (B) is correct. The accounting rate of return (also called the unadjusted rate of
return or book value rate of return) is calculated by dividing the increase in accounting net
income by the required investment. Sometimes the denominator is the average investment
rather than the initial investment. This method ignores the time value of money and focuses
on income as opposed to cash flows.
Answer (A) is incorrect because the IRR is the rate at which the net present value is zero.
Thus, it incorporates time value of money concepts, whereas the accounting rate of return
does not. Answer (C) is incorrect because the accounting rate of return is similar to the
divisional performance measure of return on investment. Answer (D) is incorrect because
the accounting rate of return ignores the time value of money.
37
. Answer (D) is correct. The accounting rate of return (ARR) is based on financial
statements prepared on the accrual basis. The formula to compute the ARR is:
Expected increase in annual net income
ARR =
Initial (or average) investment
Both the revenue over life of project and depreciation expense are used in the calculation of
the ARR. Depreciation expense over the project’s life and other expenses directly
associated with the project under consideration including income tax effects are subtracted
from revenue over life of the project to determine net income over life of project. Net
income over the project’s life is then divided by the economic life to determine annual net
income, the numerator of the ARR formula. This is a weakness of the ARR method because
it does not consider actual cash flows or the time value of money.
38
. Answer (B) is correct. The accounting rate of return uses undiscounted net income (not
cash flows) to determine a rate of profitability. Annual after-tax net income is divided by the
average book value (or the initial value) of the investment in assets. Answer (C) is incorrect
because the payback period is the time required to complete the return of the original
investment. This method gives no consideration to the time value of money or to returns
after the payback period.
Answer (A) is incorrect because the internal rate of return is the rate at which NPV is zero.
The minimum desired rate of return is not used in the discounting. Answer (D) is incorrect
because the NPV method computes the discounted present value of future cash inflows to
determine whether it is greater than the initial cash outflow.
39
. Answer (B) is correct. The payback method measures the number of years required to
complete the return of the original investment. This measure is computed by dividing the
net investment by the average expected cash inflows to be generated, resulting in the
number of years required to recover the original investment. The payback method gives no
consideration to the time value of money, and there is no consideration of returns after the
payback period.
Answer (A) is incorrect because the discounted cash flow method computes a rate of return.
Answer (C) is incorrect because the net present value method is based on discounted cash
flows; the length of time to recover an investment is not the result. Answer (D) is incorrect
because the net present value method is based on discounted cash flows; the length of time
to recover an investment is not the result.
40
. Answer (B) is correct. The usual payback formula divides the initial investment by the
constant net annual cash inflow. The payback method is unsophisticated in that it ignores
the time value of money, but it is widely used because of its simplicity and emphasis on
recovery of the initial investment.
Answer (A) is incorrect because the net present value method first discounts the future cash
flows to their present value. Answer (C) is incorrect because the profitability index method
divides the present value of the future net cash inflows by the initial investment. Answer (D)
is incorrect because the accounting rate of return divides the annual net income by the
average investment in the project.
41
. Answer (B) is correct. The payback method calculates the number of years required to
complete the return of the original investment. This measure is computed by dividing the
net investment required by the average expected cash flow to be generated, resulting in the
number of years required to recover the original investment. Payback is easy to calculate
but has two principal problems: it ignores the time value of money, and it gives no
consideration to returns after the payback period. Thus, it ignores total project profitability.
Answer (A) is incorrect because the payback method does not incorporate the time value of
money. Answer (C) is incorrect because the payback method uses the net investment in the
numerator of the calculation. Answer (D) is incorrect because payback uses the net annual
cash inflows in the denominator of the calculation.
42
. Answer (A) is correct. The payback method calculates the amount of time required to
complete the return of the original investment, i.e., the time it takes for a new asset to pay
for itself. Although the payback method is easy to calculate, it has inherent problems. The
time value of money and returns after the payback period are not considered.
Answer (B) is incorrect because the payback method ignores cash flows after payback.
Answer (C) is incorrect because the payback method does not use discounted cash flow
techniques. Answer (D) is incorrect because the payback method may lead to different
decisions.
43
. Answer (D) is correct. The payback reciprocal (1 ÷ payback) has been shown to
approximate the internal rate of return (IRR) when the periodic cash flows are equal and the
life of the project is at least twice the payback period.
Answer (A) is incorrect because the payback reciprocal is not related to the profitability
index. Answer (B) is incorrect because the payback reciprocal approximates the IRR, which is
the rate at which the NPV is $0. Answer (C) is incorrect because the accounting rate of
return is based on accrual-income based figures, not on discounted cash flows.
44
. Answer (C) is correct. The payback period is computed by dividing the initial investment
by the annual net cash inflow. Depreciation expense is not subtracted from cash inflow; ony
the income taxes which are cause by the depreciation deduction are substracted. One of
the weaknesses of the payback period is that is ignores the time value of time.
45
. Answer (B) is correct. The bailout payback period is the length of time required for the
sum of the cumulative net cash inflow from an investment and its salvage value to equal the
original investment. The bailout payback method measures the risk to the investor if the
investment must be abandoned. The shorter the period, the lower the risk.
Answer (A) is incorrect because the use of the bailout payback method is not limited to firms
with federally insured loans. Answer (C) is incorrect because the payback period is
calculated by summing the net cash inflow and the salvage value. Answer (D) is incorrect
because the bailout payback method does not estimate short-term profitability.
46
. Answer (D) is correct. The payback period equals the net investment divided by the
average expected cash flow, resulting in the number of years required to recover the
original investment. The bailout payback incorporates the salvage value of the asset into the
calculation. It determines the length of the payback period when the periodic cash inflows
are combined with the salvage value. Hence, the method measures risk. The longer the
payback period, the more risky the investment.
Answer (A) is incorrect because the bailout payback method does not consider the time
value of money. Answer (B) is incorrect because the bailout payback includes salvage value
as well as cash flow from operations. Answer (C) is incorrect because the bailout payback
incorporates the disposal value in the payback calculation.
47
. Answer (A) is correct. Time value of money means that, because of the interest factor,
money held today is worth more than the same amount of money to be received in the
future. Interest is paid for the use of money, i.e., on debts, in a normal business transaction.
This payment compensates the lender for not being able to use the money for current
consumption.
Answer (B) is incorrect because present value is the value today, net of the interest factor,
of one or more payments to be made in the future. Answer (C) is incorrect because future
value is the value some time in the future of a deposit today or of a series of deposits.
Answer (D) is incorrect because an annuity is generally a series of equal payments at equal
intervals of time.
48
. Answer (A) is correct. The time value of money is concerned with two issues: (1) the
investment value of money, and (2) the risk (uncertainty) inherent in any executory
agreement. Thus, a dollar today is worth more than a dollar in the future, and the longer one
waits for a dollar, the more uncertain the receipt is. The cost of capital involves a specific
application of the time value of money principles. It is not a basic concept thereof.
Answer (B) is incorrect because risk is a basic time value of money concept. Cost of capital is
not. Answer (C) is incorrect because the interest effect is a basic time value of money
concept. Answer (D) is incorrect because the interest effect and risk are basic time value of
money concepts. Cost of capital is not.
49
. Answer (A) is correct. The present value concept may be applied both to dollars-in
(inflows) and to dollars-out (outflows). Thus, individual cash inflows and cash outflows or a
series thereof (an annuity) may be discounted to time zero (the present). Net present value
is the sum of discounted cash inflows minus any discounted cash outflows. Net present value
may be either positive or negative.
Answer (B) is incorrect because a present value may be calculated for discounted cash
outflows. Answer (C) is incorrect because a present value may be calculated for discounted
cash inflows or a series thereof (an annuity). Answer (D) is incorrect because a present value
may be calculated for discounted cash inflows or outflows.
50
. Answer (C) is correct. Discounted cash flow analysis, using either the internal rate of
return (IRR) or the net present value (NPV) method, is based on the time value of cash
inflows and outflows. All future operating cash savings are considered as well as the tax
effects on cash flows of future depreciation charges. The cash proceeds of future asset
disposals are likewise a necessary consideration. Depreciation expense is a consideration
only to the extent that it affects the cash flows for taxes. Otherwise, depreciation is excluded
from the analysis because it is a noncash expense.
Answer (A) is incorrect because future operating cash savings is a consideration in
discounted cash flow analysis. Answer (B) is incorrect because the current asset disposal
price is a consideration in discounted cash flow analysis. Answer (D) is incorrect because
the tax effects of future asset depreciation is a consideration in discounted cash flow
analysis.
51
. Answer (D) is correct. The capital budgeting methods that are generally considered the
best for long-range decision making are the internal rate of return and net present value
methods. These are both discounted cash flow methods.
Answer (A) is incorrect because the payback method gives no consideration to the time
value of money or to returns after the payback period. Answer (B) is incorrect because the
accounting rate of return does not consider the time value of money. Answer (C) is incorrect
because the unadjusted rate of return does not consider the time value of money.
52
. DISCUSSION: (A) The time value of money is concerned with two issues: (1) the
investment value of money, and (2) the risk (uncertainty) inherent in any executory
agreement. Thus, a dollar today is worth more than a dollar in the future, and the longer
one waits for a dollar, the more uncertain the receipt is.
Answers (B), (C) and (D) are incorrect because risk and interest factors are concepts
underlying the time value of money.
53
. Answer (C) is correct. Depreciation is a noncash expense that is deductible for federal
income tax purposes. Hence, it directly reduces the cash outlay for income taxes and is
explicitly incorporated in the capital budgeting model.
Answer (A) is incorrect because depreciation is not a cost of operations in the capital
budgeting model. Also, depreciation can be avoided by not making investments. Answer (B)
is incorrect because depreciation is an allocation of historical cost and as such is not a cash
inflow, but it may reduce cash outflows for taxes. Answer (D) is incorrect because periodic
depreciation is determined by spreading the depreciation base, i.e., the cost of the asset
minus salvage value, not the initial cash outflow, over the life of the investment.
54
. REQUIRED: The benchmark cost of capital.
DISCUSSION: (D) A weighted average of the costs of all financing sources should be used,
with the weights determined by the usual financing proportions. The terms of any financing
raised at the time of initiating a particular project do not represent the cost of capital for the
firm. When a firm achieves its optimal capital structure, the weighted-average cost of
capital is minimized.
Answers (A), (B), and (C) are incorrect because the cost of capital is a composite, or
weighted average, of all financing sources in their usual proportions. The cost of capital
should also be calculated on an after-tax basis.
55
. REQUIRED: To determine when the discount rate (hurdle rate) must be determined
before a capital budgeting method can be used.
Answer (C) is correct. The net present value method calculates the expected net monetary
gain or loss from a project by discounting all expected future cash inflows and outflows to
the present using some predetermined minimum desired rate of return (hurdle rate).
Answer (A) is incorrect because the payback method measures the time it will take to
recoup, in the form of cash inflows from operations, the initial dollars invested in a project.
The payback period does not consider the time value of money.
Answers (B) and (D) are incorrect. The time adjusted rate of return method is also called the
internal rate of return method. This method computes the rate of interest at which the
present value of expected cash inflows from a project equals the present value of expected
cash outflows of the project. Here, the discount rate is not determined in advance but is the
end result of the calculation.
56
. Answer (D) is correct. A hurdle rate is not necessary in calculating the accounting rate of
return. That return is calculated by dividing the net income from a project by the investment
in the project. Similarly, a company can calculate the internal rate of return (IRR) without
knowing its hurdle rate. The IRR is the discount rate at which the net present value is $0.
However, the NPV cannot be calculated without knowing the company's hurdle rate. The NPV
method requires that future cash flows be discounted using the hurdle rate.
Answer (A) is incorrect because the accounting rate of return and the IRR but not the NPV
can be calculated without knowing the hurdle rate. Answer (B) is incorrect because the
accounting rate of return and the IRR but not the NPV can be calculated without knowing the
hurdle rate. Answer (C) is incorrect because the accounting rate of return and the IRR but
not the NPV can be calculated without knowing the hurdle rate.
57
. Answer (D) is correct. Breakeven time evaluates the rapidity of new product
development. The usual calculation determines the period beginning with project approval
that is required for the discounted cumulative cash inflows to equal the discounted
cumulative cash outflows. However, it may also be calculated as the point at which
discounted cumulative cash inflows on a project equal discounted total cash outflows. The
concept is similar to the payback period, but it is more sophisticated because it incorporates
the time value of money. It also differs from the payback method because the period
covered begins at the outset of a project, not when the initial cash outflow occurs.
Answer (A) is incorrect because it is related to breakeven point, not breakeven time. Answer
(B) is incorrect because the payback period equals investment divided by annual
undiscounted net cash inflows. Answer (C) is incorrect because the payback period is the
period required for total undiscounted cash inflows to equal total undiscounted cash
outflows.
58
. Answer (C) is correct. The profitability index is another term for the excess present value
index. It measures the ratio of the present value of future net cash inflows to the original
investment. In organizations with unlimited capital funds, this index will produce no conflicts
in the decision process. If capital rationing is necessary, the index will be an insufficient
determinant. The capital available as well as the dollar amount of the net present value
must both be considered.
Answer (A) is incorrect because capital rationing is not a technique but rather a condition
that characterizes capital budgeting when insufficient capital is available to finance all
profitable investment opportunities. Answer (B) is incorrect because the average rate of
return method does not divide the future cash flows by the cost of the investment. Answer
(D) is incorrect because the accounting rate of return does not recognize the time value of
money.
59
. Answer (D) is correct. The profitability index, also known as the excess present value
index, is the ratio of the present value of future net cash inflows to the initial net cash
investment (discounted cash outflows). This tool is a variation of the NPV method that
facilitates comparison of different-sized investments.
Answer (A) is incorrect because the cash inflows are also discounted in the profitability
index. Answer (B) is incorrect because the numerator is the discounted net cash inflows.
Answer (C) is incorrect because the profitability index is based on cash flows, not profits.
60
. Answer (B) is correct. The net present value (NPV) method computes the discounted
present value of future cash inflows to determine whether they are greater than the initial
cash outflow. The discount rate (cost of capital or hurdle rate) must be known to discount
the future cash inflows. If the NPV is positive (present value of future cash inflows exceeds
initial cash outflow), the project should be accepted. If the NPV is negative, the project
should be rejected.
Answer (A) is incorrect because the accounting rate of return uses net income (not cash
flows) to determine a rate of profitability. Answer (C) is incorrect because the internal rate
of return is the rate at which NPV is zero. The minimum desired rate of return is not used.
Answer (D) is incorrect because the payback method measures the time required to
complete the return of the original investment. It gives no consideration to the time value of
money or to returns after the payback period.
61
. Answer (A) is correct. The net present value method discounts future cash flows to the
present value using some arbitrary rate of return, which is presumably the firm's cost of
capital. The initial cost of the project is then deducted from the present value. If the present
value of the future cash flows exceeds the cost, the investment is considered to be
acceptable.
Answer (B) is incorrect because the payback method does not recognize the time value of
money. Answer (C) is incorrect because the average rate of return method does not use the
firm's cost of capital as a discount rate. Answer (D) is incorrect because the accounting rate
of return method does not recognize the time value of money.
62
. REQUIRED: The effect on the value of the firm and its stock price of investment
opportunities.
DISCUSSION: (A) Investments with present values in excess of their costs (positive NPVs)
that can be identified or created by the capital budgeting activities of the firm will have a
positive impact on firm value and on the price of the common stock of the firm. Accordingly,
the more effective capital budgeting is, the higher the stock price.
Answer (B) is incorrect because positive NPV investments will increase, not decrease firm
value and share price. Answers (C) and (D) are incorrect because investments with present
values equal to their costs have a zero NPV and neither increase nor decrease firm value
and share price.
63
. Answer (A) is correct. The NPV method computes the present value of future cash inflows
to determine whether they are greater than the initial cash outflow. Future cash inflows
include any salvage value on facilities. Included in the initial investment are the cost of new
equipment and other facilities, and additional working capital needed for operations during
the term of the project. The discount rate (cost of capital or hurdle rate) must be known to
discount the future cash inflows. If the NPV is positive, the project should be accepted. The
method of funding a project is a decision separate from that of whether to invest.
Answer (B) is incorrect because the initial costs of the project, including additional working
capital needs, are necessary to determine the NPV. Answer (C) is incorrect because the
initial costs of the project, including additional working capital needs, are necessary to
determine the NPV. Answer (D) is incorrect because the project's salvage value is a future
cash inflow to be discounted.
64
. REQUIRED: The variable(s) considered in the NPV calculation.
DISCUSSION: (D)The NPV is the difference between the present value of the future cash
flows from the project discounted at an appropriate interest rate and the initial investment.
If the NPV is zero or greater, the investment may be economically rational. The method is a
technique for ranking investment proposals. Consequently, the time value of the cash flows
over the life of the project is considered.
Answers (A), (B), and (C) are incorrect because the time value of the cash flows over the life
of the project is considered.
65
. Answer (D) is correct. The effect of assuming cash flows occur at the end of the year
simply understates the present values of the future cash flows; in reality, they probably
occur on the average at mid-year.
Answer (A) is incorrect because cash flows in investment decisions do not all occur at the
end of each year. Answer (B) is incorrect because discounting cash flows approximately 6
months longer understates rather than overstates. Answer (C) is incorrect because the effect
of using the year-end assumption produces a slight conservatism in the model but does not
render the results unusable.
66
. Answer (C) is correct. The NPV method is used when the discount rate (cost of capital) is
specified. It assumes that cash flows from the investment can be reinvested at the particular
project's cost of capital (the discount rate).
Answer (A) is incorrect because the internal rate of return method assumes that cash flows
are reinvested at the internal rate of return. Answer (B) is incorrect because the NPV
method assumes that cash flows are reinvested at the NPV discount rate. Answer (D) is
incorrect because the NPV method assumes that cash flows are reinvested at the NPV
discount rate.
67
. Answer (D) is correct. The NPV method assumes that periodic cash inflows earned over
the life of an investment are reinvested at the company's cost of capital (i.e., the discount
rate used in the analysis). This is contrary to the assumption under the internal rate of
return method, which assumes that cash inflows are reinvested at the internal rate of return.
As a result of this difference, the two methods will occasionally give different rankings of
investment alternatives.
Answer (A) is incorrect because the NPV method assumes that cash inflows are reinvested at
the discount rate used in the NPV calculation. Answer (B) is incorrect because the NPV
method assumes that cash inflows are reinvested at the discount rate used in the NPV
calculation. Answer (C) is incorrect because the internal rate of return method assumes a
reinvestment rate equal to the rate of return on the project.
68
. Answer (B) is correct. The NPV method calculates the present values of estimated future
net cash inflows and compares the total with the net cost of the investment. The cost of
capital must be specified. If the NPV is positive, the project should be accepted. The IRR
method computes the interest rate at which the NPV is zero. The IRR method is relatively
easy to use when cash inflows are the same from one year to the next. However, when cash
inflows differ from year to year, the IRR can be found only through the use of trial and error.
In such cases, the NPV method is usually easier to apply. Also, the NPV method can be used
when the rate of return required for each year varies. For example, a company might want
to achieve a higher rate of return in later years when risk might be greater. Only the NPV
method can incorporate varying levels of rates of return.
Answer (A) is incorrect because the IRR method calculates a rate of return. Answer (C) is
incorrect because the IRR is the rate at which NPV is zero. Answer (D) is incorrect because
both methods discount cash flows.
69
. Answer (C) is correct. The NPV is broadly defined as the excess of the present value of
the estimated net cash inflows over the net cost of the investment. A discount rate has to be
stipulated by the person conducting the analysis. A disadvantage is that it does not provide
the true rate of return for an investment, only that the rate of return is higher than a
stipulated discount rate (which may be the cost of capital).
Answer (A) is incorrect because the ability to perform sensitivity analysis is an advantage of
the NPV method. Answer (B) is incorrect because the NPV method does not compute the
true interest rate. Answer (D) is incorrect because the IRR method, not the NPV method,
uses a trial-and-error approach when cash flows are not identical from year to year.
70
. Answer (C) is correct. The discount rate most often used in present value calculations is
the minimum desired rate of return as set by management. The NPV arrived at in this
calculation is a first step in the decision process. It indicates how the project's return
compares with the minimum desired rate of return.
Answer (A) is incorrect because the Federal Reserve rate may be considered; however, the
firm will set its minimum desired rate of return in view of its needs. Answer (B) is incorrect
because the Treasury bill rate may be considered; however, the firm will set its minimum
desired rate of return in view of its needs. Answer (D) is incorrect because the prime rate
may be considered; however, the firm will set its minimum desired rate of return in view of
its needs.
71
. Answer (A) is correct. The rate used to discount future cash flows is sometimes called the
cost of capital, the discount rate, the cutoff rate, or the hurdle rate. A risk-free rate is the
rate available on risk-free investments such as government bonds. The risk-free rate is not
equivalent to the cost of capital because the latter must incorporate a risk premium.
Answer (B) is incorrect because the rate used under the NPV method is the company's cost
of capital. Answer (C) is incorrect because the NPV method discounts future cash flows to
their present values. Answer (D) is incorrect because the cost of capital is often called a
cutoff rate. Investments yielding less than the cost of capital should not be made.
72
. Answer (D) is correct. The NPV is the excess of the present values of the estimated cash
inflows over the net cost of the investment. The discount rate used is sometimes the cost of
capital or other hurdle rate designated by management. This rate is also called the required
rate of return. The accounting rate of return is never used in NPV analysis because it ignores
the time value of money; it is computed by dividing the accounting net income by the
investment.
Answer (A) is incorrect because cost of capital is a synonym for the rate used in NPV
analysis. Answer (B) is incorrect because hurdle rate is a synonym for the rate used in NPV
analysis. Answer (C) is incorrect because discount rate is a synonym for the rate used in NPV
analysis.
73
. REQUIRED: The matter affecting a project’s net present value.
DISCUSSION: (A) To compute a project’s net present value, the initial investment is
subtracted from the present value of the after-tax cash flows. The proceeds from the sale of
the asset to be replaced reduces the initial investment.
Answer (B) is incorrect because the carrying amount of the asset to be replaced affects the
gain or loss on the sale. Answer (C) is incorrect because the amount of annual depreciation
on the asset to be replaced affects the carrying value. Answer (D) is incorrect because
annual depreciation of other assets, even if used directly, does not affect the project’s net
present value.
74
. Answer (B) is correct. In an inflationary environment, nominal future cash flows should
increase to reflect the decrease in the value of the unit of measure. Also, the investor should
increase the discount rate to reflect the increased inflation premium arising from the
additional uncertainty. Lenders will require a higher interest rate in an inflationary
environment.
Answer (A) is incorrect because future cash flows should also increase. Answer (C) is
incorrect because the discount rate should be increased to take into consideration future
uncertainty and the risk premium that lenders will require in an inflationary environment.
Answer (D) is incorrect because cash flows should increase in an inflationary environment.
75
. Answer (A) is correct. The internal rate of return (IRR) is the discount rate at which the
present value of the cash inflows equals the present values of the cash outflows (including
the original investment). Thus, the NPV of the project is zero at the IRR. The IRR is also the
maximum borrowing cost the firm can afford to pay for a specific project. The IRR is similar
to the yield rate/effective rate quoted in the business media.
Answer (B) is incorrect because the capital asset pricing model is a means of determining
the cost of capital. Answer (C) is incorrect because the profitability index is not an interest
rate. Answer (D) is incorrect because the accounting rate of return is not based on present
values.
76
. Answer (C) is correct. The IRR is the interest rate at which the present value of the
expected future cash inflows is equal to the present value of the cash outflows for a project.
Thus, the IRR is the interest rate that will produce a net present value (NPV) equal to zero.
The IRR method assumes that the cash flows will be reinvested at the internal rate of return.
Answer (A) is incorrect because the hurdle rate is a concept used to calculate the NPV of a
project; it is determined by management prior to the analysis. Answer (B) is incorrect
because the IRR is the rate of interest at which the NPV is zero. Answer (D) is incorrect
because the IRR is a means of evaluating potential investment projects.
77
. Answer (C) is correct. The IRR is a capital budgeting technique that calculates the
interest rate that yields a net present value equal to $0. It is the interest rate that will
discount the future cash flows to an amount equal to the initial cost of the project. Thus, the
higher the IRR, the more favorable the ranking of the project.
Answer (A) is incorrect because the cost of capital is not used in the calculation of the IRR.
Answer (B) is incorrect because the IRR can be determined regardless of the constancy of
the cash flows. However, it is more difficult to calculate when cash flows are not constant
because a trial-and-error approach must be used. Answer (D) is incorrect because there is
no relationship between IRR and the profitability index.
78
. REQUIRED: The true statement about internal rate of return.
DISCUSSION: (D) The internal rate of return (IRR) is the discount rate at which the present
value of the cash flows equals the original investment. Thus, the NPV of the project is zero
at the IRR. The IRR is also the maximum borrowing cost the firm could afford to pay for a
specific project. The IRR is similar to the yield rate/effective rate quoted in the business
media.
Answers (A), (B), and (C) are incorrect because the IRR is the discount rate at which the NPV
of the cash flows is zero, the breakeven borrowing rate for projects, and the yield
rate/effective rate of interest quoted on long-term debt and other instruments.
79
. Answer (D) is correct. The internal rate of return (IRR) is the discount rate at which the
present value of the cash flows equals the original investment. Thus, the NPV of the project
is zero at the IRR. The IRR is also the maximum borrowing cost the firm could afford to pay
for a specific project. The IRR is similar to the yield rate/effective rate quoted in the business
media.
Answer (A) is incorrect because the IRR is the discount rate at which the NPV of the cash
flows is zero, the breakeven borrowing rate for the project in question, the yield
rate/effective rate of interest quoted on long-term debt and other instruments, and favorable
when it exceeds the hurdle rate. Answer (B) is incorrect because the IRR is the discount rate
at which the NPV of the cash flows is zero, the breakeven borrowing rate for the project in
question, the yield rate/effective rate of interest quoted on long-term debt and other
instruments, and favorable when it exceeds the hurdle rate. Answer (C) is incorrect because
the IRR is the discount rate at which the NPV of the cash flows is zero, the breakeven
borrowing rate for the project in question, the yield rate/effective rate of interest quoted on
long-term debt and other instruments, and favorable when it exceeds the hurdle rate.
80
. Answer (D) is correct. The internal rate of return of a proposed project includes the
residual sales value of a project but not the depreciation expense. This is true because the
residual sales value represents a future cash flow whereas depreciation expense (ignoring
income tax considerations) provides no cash inflow or outflow.
Answers (A), (B), and (C) are incorrect because they contain the wrong combination of
responses.
81
. Answer (C) is correct. Under the internal rate of return (IRR) method, the interest rate is
computed that will exactly match the present value of the future net inflows with the initial
cost of the investment. The IRR method assumes that cash flows will be reinvested at the
IRR. Thus, if the project's funds are not reinvested at the internal rate of return, the ranking
calculations obtained under the IRR method may be in error. The net present value method
gives a better grasp of the problem in many decision situations because the reinvestment is
assumed to be at the cost of capital.
Answer (A) is incorrect because the IRR does calculate compounded interest rates. Answer
(B) is incorrect because both methods incorporate the time value of money. Answer (D) is
incorrect because sensitivity analysis can be used with NPV to handle multiple desired
hurdle rates.
82
. Answer (D) is correct. The IRR is the rate at which the discounted future cash flows equal
the net investment (NPV = 0). One disadvantage of the method is that inflows from the early
years are assumed to be reinvested at the IRR. This assumption may not be sound.
Investments in the future may not earn as high a rate as is currently available.
Answer (A) is incorrect because the IRR method considers the time value of money. Answer
(B) is incorrect because the IRR provides a straightforward decision criterion. Any project
with an IRR greater than the cost of capital is acceptable. Answer (C) is incorrect because
the IRR method implicitly assumes reinvestment at the IRR; the NPV method implicitly
assumes reinvestment at the cost of capital.
83
. REQUIRED: Advantage of the IRR over the accounting rate of return.
DISCUSSION: (C) The IRR is the interest rate that equalizes the present value of future cash
flows with the initial cost of the investment. The accounting rate of return is calculated by
dividing the increase in accounting net income by the required investment. However, it
ignores the time value of money and does not emphasize cash flows.
Answers (A), (B), and (D) are incorrect because both techniques recognize the project’s
salvage value.
84
. Statement c is correct; the other statements are false. MIRR and NPV can
conflict for mutually exclusive projects if the projects differ in size. NPV does not
suffer from the multiple IRR problem.
85
. Answer (D) is correct. The profitability index is the ratio of the present value of future net
cash inflows to the initial net cash investment. It is a variation of the NPV method that
facilitates comparison of different-sized investments. A profitability index greater than 1.0
indicates a profitable investment, or one that has a positive net present value.
Answer (A) is incorrect because the IRR is the discount rate at which the NPV is $0, which is
also the rate at which the profitability index is 1.0. The IRR cannot be determined solely from
the index. Answer (B) is incorrect because, if the index is 1.15 and the discount rate is the
cost of capital, the NPV is positive, and the IRR must be higher than the cost of capital.
Answer (C) is incorrect because the IRR is a discount rate, whereas the NPV is an amount.
86
. (d) The internal rate of return method determines the rate of return at which the present
value of the cash flows will exactly equal the investment outlay. It will indicate the rate of
return earned over the life of the project. The net present value method determines the
present vale of all future cash flows at a selected discount rate. If the NPV of the cash flows
is positive, the return earned b the project is higher than the selected rate. Both methods
will provide the information needed to decide if a project’s rate of return will meet Polo co.’s
requirement.
87
. Answer (D) is correct. The IRR is defined as the rate at which the NPV is zero.
Accordingly, if the NPV is positive at a cost of capital of 15%, the rate (the IRR) required to
reduce the NPV to zero must exceed 15%.
Answer (A) is incorrect because the accounting rate of return (net income ÷ investment)
ignores the time value of money. It is not determinable from the given information. Answer
(B) is incorrect because the accounting rate of return (net income ÷ investment) ignores the
time value of money. It is not determinable from the given information. Answer (C) is
incorrect because the payback period ignores the time value of money. It is not
determinable from the given information.
88
. Answer (C) is correct. The relationship between the NPV method and the IRR method
can be summarized as follows:
NPVIRRNPV > 0IRR > Discount rateNPV = 0IRR = Discount rateNPV < 0IRR < Discount
rateSince the problem states that Neu Co. has a positive net present value on the
investment, then the internal rate of return would be > 12%.
89
. Answer (A) is correct. The higher the discount rate, the lower the NPV. The IRR is the
discount rate at which the NPV is zero. Consequently, if the NPV is negative, the discount
rate used must exceed the IRR.
Answer (B) is incorrect because, if the discount rate is less than the IRR, the NPV is positive.
Answer (C) is incorrect because the NPV measures the difference between a company's
discount rate and the IRR. Answer (D) is incorrect because the relationship between the
discount rate and the risk-free rate is not a factor in investment analysis under the NPV
method.
90
. Statement a is correct; the other statements are false. If the projects are
mutually exclusive, then project B may have a higher NPV even though Project A
has a higher IRR. IRR is calculated assuming cash flows are reinvested at the IRR,
not the cost of capital.
91
. The correct answer is a; the other statements are false. The IRR is the
discount rate at which a project’s NPV is zero. If a project’s IRR exceeds the
firm’s cost of capital, then its NPV must be positive, since NPV is calculated using
the firm’s cost of capital to discount project cash flows.

92
.Statement a is true; projects with IRRs greater than the cost of capital will have a positive
NPV. Statement b is false because you know nothing about the relative magnitudes of the
projects. Statement c is false because the IRR is independent of the cost of capital.
Therefore, the correct choice is statement a.
93
. The correct statement is b; the other statements are false. Since Project A’s IRR is 15%,
at a WACC of 15% NPVA = 0; however, Project B would still have a positive NPV. Given the
information in a, we can’t conclude which project’s NPV is going to be greater. Since we are
given no details about each project’s cash flows we cannot conclude anything about
payback. Finally, IRR is independent of the discount rate, that is, IRR stays the same no
matter what the WACC is.
94
. Statement a is false. The projects could easily have different NPVs based on different
cash flows and costs of capital. Statement b is false. NPV is dependent upon the size of the
project. Think about the NPV of a $3 project versus the NPV of a $3 million project.
Statement c is false. NPV is dependent on a project’s risk. Therefore, the correct choice is
statement e.
95
. A project’s NPV increases as the cost of capital declines. A project’s IRR is
independent of its cost of capital, while a project’s MIRR is dependent on the cost
of capital since the terminal value in the MIRR equation is compounded at the cost
of capital.
96
. Statement a is the incorrect statement. NPV is positive if IRR is greater than
the cost of capital.

97
. Statement b is correct; the other statements are incorrect. Statement a is incorrect; if
the NPV > 0, then the return must be > 12%. Statement c is incorrect; if NPV > 0, then
MIRR > WACC.

98
. Statement e is correct; the other statements are incorrect. Statement a is incorrect; the
two projects’ NPV profiles could cross, consequently, a higher IRR doesn’t guarantee a
higher NPV. Statement b is incorrect; it the two projects’ NPV profiles cross, Y could have a
higher NPV. Statement c is incorrect; we don’t have enough information.
99
. Statement a is true because the IRR exceeds the WACC. Statement b is also true
because the MIRR assumes that the inflows are reinvested at the WACC, which is less than
the IRR. Statement c is false. For a normal project, the discounted payback is always longer
than the regular payback because it takes longer for the discounted cash flows to cover the
purchase price. So, statement d is the best choice.
100
. Answer (A) is correct. If taxes are ignored, depreciation is not a consideration in any of
the methods based on cash flows because it is a non-cash expense. Thus, the internal rate of
return, net present value, and payback methods would not consider depreciation because
these methods are based on cash flows. However, the accounting rate of return is based on
net income as calculated on an income statement. Because depreciation is included in the
determination of accrual accounting net income, it would affect the calculation of the
accounting rate of return.
Answer (B) is incorrect because the IRR and the payback period are based on cash flows.
Depreciation is not needed in their calculation. However, the accounting rate of return
cannot be calculated without first deducting depreciation. Answer (C) is incorrect because
the IRR and the payback period are based on cash flows. Depreciation is not needed in their
calculation. However, the accounting rate of return cannot be calculated without first
deducting depreciation. Answer (D) is incorrect because the IRR and the payback period are
based on cash flows. Depreciation is not needed in their calculation. However, the
accounting rate of return cannot be calculated without first deducting depreciation.
101
. Answer (C) is correct. The profitability index is the ratio of the present value of future net
cash inflows to the initial net cash investment. It is a variation of the net present value (NPV)
method and facilitates the comparison of different-sized investments. Because it is based on
the NPV method, the profitability index will yield the same decision as the NPV for
independent projects. However, decisions may differ for mutually exclusive projects of
different sizes.
Answer (A) is incorrect because the profitability index, like the NPV method, discounts cash
flows based on the cost of capital. Answer (B) is incorrect because the profitability index is
cash based. Answer (D) is incorrect because the NPV and the profitability index may yield
different decisions if projects are mutually exclusive and of different sizes.
102
. Answer (D) is correct. All three managers will reject the project. Manager one will
calculate a NPV of -$12,894 [-$100,000 + ($20,000 x 4.3553 PVIFA for six periods at 10%)].
Manager two will calculate a NPV of -$26,349 {- $100,000 + ($5,000 x .8772 PVIF for one
period at 14%) + [($23,000 x 3.4331 PVIFA for five periods at 14%) x .8772 PVIF for one
period at 14%]}. Manager three will calculate a net present value of -$41,640 [-$100,000 +
($135,000 x .4323 PVIF for six periods at 15%)].
Answer (A) is incorrect because all three managers will calculate a negative NPV, and none
will recommend acceptance. Answer (B) is incorrect because all three managers will
calculate a negative NPV, and none will recommend acceptance. Answer (C) is incorrect
because all three managers will calculate a negative NPV, and none will recommend
acceptance.
103
. Answer (A) is correct. The payback period is the number of periods it takes before the
cash flows from the project repay the original investment outlay. This can be expressed as
net investment divided by the average expected cash flow. Manager one expects inflows of
$20,000 per year, so it will take exactly 5 years for the project to repay the original
$100,000 invested. Manager two will calculate a payback period of more than 5 years. Only
$5,000 is expected at the end of year one, followed by inflows of $23,000 at the end of each
year in years two through six. At the end of year five, only $97,000 will have been received,
based on these expectations. Manager three will calculate a payback period of 6 years. She
estimates one inflow of $135,000 at the end of year six.
Answer (B) is incorrect because manager one will calculate a 5-year payback period, which is
shorter than the periods determined by managers two and three. Answer (C) is incorrect
because manager one will calculate a 5-year payback period, which is shorter than the
periods determined by managers two and three. Answer (D) is incorrect because all three
managers will derive a different payback period for the project.
104
. Answer (B) is correct. If the net present value (NPV) of an investment is positive, the
project should be accepted (unless projects are mutually exclusive). If the NPV is negative,
the investment should be rejected.
Answer (A) is incorrect because the present value of future net cash inflows must be
compared with the initial cash outlay to determine whether a project is acceptable. Answer
(C) is incorrect because an IRR may be greater than zero but less than a firm's cost of
capital, in which case the project would not be profitable. Answer (D) is incorrect because
the accounting rate of return is not based on cash flows and is irrelevant to a company's
hurdle rate.
105
. Answer (D) is correct. Given unlimited funds, all projects with a net present value greater
than zero should be invested in. Thus, it would be profitable to invest in any company where
the rate of return is greater than the cost of capital.
Answer (A) is incorrect because neither the accounting rate of return nor the earnings as a
percent of sales is useful in capital budgeting. The accounting rate of return is accounting
net income over the required investment; it ignores the time value of money. Earnings as a
percent of sales ignores the amount of required investment. Answer (B) is incorrect because
the payback criterion for capital budgeting is not efficient or effective. Answer (C) is
incorrect because the problem states that there are unlimited capital funds but does not
indicate what the cost of capital is. Accordingly, projects can only be invested in when the
internal rate of return is greater than cost of capital, i.e., the net present value is greater
than zero.
106
. Answer (D) is correct. A company should accept any investment proposal, unless some
are mutually exclusive, that has a positive net present value or an internal rate of return
greater than the company's cost of capital.
Answer (A) is incorrect because the mere availability of financing is not the only
consideration; more important is the cost of the financing, which must be less than the rate
of return on the proposed investment. Answer (B) is incorrect because an investment with
positive cash flows may be a bad investment due to the time value of money; cash flows in
later years are not as valuable as those in earlier years. Answer (C) is incorrect because
returns should exceed the weighted-average cost of capital, which includes the cost of
equity capital as well as the cost of debt capital.
107
. REQUIRED: The true statement about the NPV and IRR methods.
DISCUSSION: (A) The NPV criterion is that the NPV is positive, and the IRR criterion is that
the cost of capital is less than the IRR. When the cost of capital is less than the IRR, the NPV
is positive. When it exceeds the IRR, the NPV is negative. Accordingly, when two projects
are independent, the NPV and IRR criteria will always lead to the same accept or reject
decision.
Answer (B) is incorrect because, if the second project’s IRR is higher than the first project’s,
the organization would accept the second project based on the IRR criterion. Answers (C)
and (D) are incorrect because, if the projects are independent, the NPV and IRR criteria
indicates the same decision.
108
. Answer (A) is correct. Although the NPV method and the IRR method may rank projects
differently, if a project is found acceptable under the NPV approach, it will also be acceptable
under the internal rate of return approach.
Answer (B) is incorrect because the two approaches may rank projects differently (the IRR
assumes that reinvestment will be at the discount rate, which is frequently not possible).
Answer (C) is incorrect because the payback approach does not consider the time value of
money. Therefore, a project may be ranked differently than it would be under the NPV
approach or may be acceptable under the payback approach but not the NPV or IRR
approaches. Answer (D) is incorrect because the payback approach does not consider the
time value of money. Therefore, a project may be ranked differently than it would be under
the NPV approach or may be acceptable under the payback approach but not the NPV or IRR
approaches.
109
. Answer (D) is correct. The profitability (excess present value) index facilitates the
comparison of investments that have different initial costs. The profitability index equals the
present value of future net cash inflows divided by the initial cash investment. The
investment with the greater profitability index will be the preferred investment. However, if
investments are mutually exclusive, the net present value method may be the better way of
ranking projects. The excess present value index indicates the best return per dollar
invested but does not consider the alternative possibilities for unused funds. Thus, the
smaller of the mutually exclusive projects may have the higher index, but the incremental
investment in the larger project may make it the better choice. For example, an $8,000,000
project may be a better use of funds than a combination of a $6,000,000 project with a
higher index and the best alternative use of the remaining $2,000,000.
Answer (A) is incorrect because the investment generating cash flows the longest may not
have the best return. Answer (B) is incorrect because, given a net present value of zero (a
profitability index exactly equal to one), the investor would be indifferent to the project.
Answer (C) is incorrect because the accounting rate of return is not a good measure of
profitability. It ignores the time value of money.
110
. Answer (C) is correct. Investment projects may be mutually exclusive under conditions of
capital rationing (limited capital). In other words, scarcity of resources will prevent an entity
from undertaking all available profitable activities. Under the IRR method, an interest rate is
computed such that the present value of the expected future cash flows equals the cost of
the investment (NPV = 0). The IRR method assumes that the cash flows will be reinvested at
the IRR. The NPV is the excess of the present value of the estimated net cash inflows over
the net cost of the investment. The cost of capital must be specified in the NPV method. An
assumption of the NPV method is that cash flows from the investment will be reinvested at
the particular project's cost of capital. Because of the difference in the assumptions
regarding the reinvestment of cash flows, the two methods will occasionally give different
answers regarding the ranking of mutually exclusive projects. Moreover, the IRR method
may rank several small, short-lived projects ahead of a large project with a lower rate of
return but with a longer life span. However, the large project might return more dollars to
the company because of the larger amount invested and the longer time span over which
earnings will accrue. When faced with capital rationing, an investor will want to invest in
projects that generate the most dollars in relation to the limited resources available and the
size and returns from the possible investments. Thus, the NPV method should be used
because it determines the aggregate present value for each feasible combination of
projects.
Answer (A) is incorrect because the IRR is a number computed based on the characteristics
of a given project. Answer (B) is incorrect because cash flows are discounted under the IRR
method. Answer (D) is incorrect because an accelerated depreciation method will generate
larger net cash inflows in the early years of a project. To equate the present value of these
cash flows with the net investment will therefore require a higher discount rate (IRR).
111
. Answer (D) is correct. The two methods ordinarily yield the same results, but differences
can occur when the duration of the projects and the initial investments differ. The reason is
that the IRR method assumes cash inflows from the early years will be reinvested at the
internal rate of return. The NPV method assumes that early cash inflows are reinvested at
the cost of capital.
Answer (A) is incorrect because the two methods will give the same results if the lives and
required investments are the same. Answer (B) is incorrect because if the required rate of
return equals the IRR (i.e., the cost of capital is equal to the IRR), the two methods would
yield the same decision. Answer (C) is incorrect because if the required rate of return is
higher than the IRR, both methods would yield a decision not to acquire the investment.
112
. Answer (A) is correct. Project A's NPV is calculated as follows:
$1,000 x 2.2832$2,283.20 - Original cost(1,000.00) NPV$1,283.20The second project's
NPV is:
$1,500 x (3.7845 - 2.2832) $2,251.95 - Original cost(1,000.00) NPV$1,251.95Since A has
a slightly higher NPV, it should be selected.
Answer (B) is incorrect because Project A has a slightly higher NPV and IRR. Answer (C) is
incorrect because Project A has a slightly higher IRR. Answer (D) is incorrect because
Project A has a slightly higher NPV.
113
. Answer (D) is correct. The profitability index (PI) is often used to decide among
investment alternatives when more than one is acceptable. The profitability index is the
ratio of the present value of future net cash inflows to the initial net cash investment. The PI,
although a variation of the net present value method, facilitates comparison of different-
sized investments.
Answer (A) is incorrect because the accounting rate of return is a poor technique. It ignores
the time value of money. Answer (B) is incorrect because the payback method ignores the
time value of money and long-term profitability. Answer (C) is incorrect because the internal
rate of return is not effective when alternative investments have different lives.
114
. Answer (A) is correct. The profitability index is the ratio of the present value of future net
cash inflows to the initial cash investment; that is, the figures are those used to calculate the
net present value (NPV), but the numbers are divided rather than subtracted. This variation
of the NPV method facilitates comparison of different-sized investments. It provides an
optimal ranking in the absence of capital rationing.
Answer (B) is incorrect because the profitability index method is a discounted cash flow
method. Answer (C) is incorrect because the payback method gives no consideration to the
time value of money or to returns after the payback period. Answer (D) is incorrect because
the profitability index method and the NPV method are discounted cash flow methods.
However, the profitability index method is the variant that purports to calculate a return per
dollar of investment.
115
. Answer (C) is correct. The profitability index is the ratio of the present value of future net
cash inflows to the initial cash investment. This variation of the net present value method
facilitates comparison of different-sized investments. Were it not for this comparison feature,
the profitability index would be no better than the net present value method. Thus, it is the
comparison, or ranking, advantage that makes the profitability index different from the other
capital budgeting tools.
Answer (A) is incorrect because the net present value (NPV > 0) is a capital budgeting tool
that screens investments; i.e., the investment must meet a certain standard to be
acceptable. Answer (B) is incorrect because the time-adjusted rate of return is a capital
budgeting tool that screens investments; i.e., the investment must meet a certain standard
(rate of return) to be acceptable. Answer (D) is incorrect because the accounting rate of
return is a capital budgeting tool that screens investments; i.e., the investment must meet a
certain standard (rate of return) to be acceptable.
116
. Answer (B) is correct. The IRR is the discount rate at which the net present value of a
project is zero. Consequently, if the IRR exceeds the cost of capital, the NPV calculated at
the cost of capital must be positive. Projects with a positive NPV are expected to be
profitable and should be considered. Other factors being equal, projects with higher IRRs
should be accepted before those with lower IRRs.
Answer (A) is incorrect because IRRs should exceed the cost of capital, and projects should
be accepted in the descending order of their IRRs. Answer (C) is incorrect because IRRs
should exceed the cost of capital, and projects should be accepted in the descending order
of their IRRs. Answer (D) is incorrect because IRRs should exceed the cost of capital, and
projects should be accepted in the descending order of their IRRs.
117
. Answer (C) is correct. Rational investors choose projects that yield the best return given
some level of risk. If an investor desires no risk, that is, an absolutely certain rate of return,
the risk-free rate is used in calculating net present value. The risk-free rate is the return on a
risk-free investment such as government bonds. Certainty equivalent adjustments involve a
technique directly drawn from utility theory. It forces the decision maker to specify at what
point the firm is indifferent to the choice between a sum of money that is certain and the
expected value of a risky sum.
Answer (A) is incorrect because a risk-adjusted discount rate does not represent an
absolutely certain rate of return. A discount rate is adjusted upward as the investment
becomes riskier. Answer (B) is incorrect because the cost of capital has nothing to do with
certainty equivalence. Answer (D) is incorrect because the cost of equity capital does not
equate to a certainty equivalent rate.
118
. Answer (D) is correct. Under the certainty-equivalent method, expected cash flows are
multiplied by a certainty equivalent factor and discounted at the risk-free rate. Under the
risk-adjusted discount rate method, expected cash flows are discounted at the risk-adjusted
discount rate.
Answer (A) is incorrect because the certainty-equivalent method uses the risk-free rate, not
the cost of capital. Answer (B) is incorrect because the risk-adjusted discount rate discounts
expected cash flows at the risk-adjusted rate. Answer (C) is incorrect because the certainty-
equivalent method uses the risk-free rate, not the cost of capital.
119
. Answer (D) is correct. Under the certainty-equivalent approach, expected cash flows
should be multiplied by certainty-equivalent factors and discounted at the risk-free rate.
Answer (A) is incorrect because the risk-free rate should be used rather than the cost of
capital. Answer (B) is incorrect because the risk-free rate should be used rather than the
cost of capital. Answer (C) is incorrect because the risk-free rate should be used rather than
the cost of capital.
120
. Answer (C) is correct. Uncertainty can be compensated for by adjusting the desired rate
of return. If projects have relatively uncertain returns, a higher rate should be required. A
lower rate of return may be acceptable given greater certainty. The concept is that with
increased risk should come increased rewards, i.e., a higher rate of return.
Answer (A) is incorrect because preparing an analysis of probability of outcomes is not a
simple method of adjustment. Answer (B) is incorrect because accelerated depreciation
should probably be used for tax purposes in every capital project to minimize taxes in early
years. Answer (D) is incorrect because uniformly increasing the estimated cash flows and/or
ignoring salvage values introduces error into the capital budgeting analysis.
121
. Answer (D) is correct. Risk analysis attempts to measure the likelihood of the variability
of future returns from the proposed investment. Risk can be incorporated into capital
budgeting decisions in a number of ways, one of which is to use a hurdle rate higher than
the firm's cost of capital, that is, a risk-adjusted discount rate. This technique adjusts the
interest rate used for discounting upward as an investment becomes riskier. The expected
flow from the investment must be relatively larger or the increased discount rate will
generate a negative net present value, and the proposed acquisition will be rejected.
Answer (A) is incorrect because the nature of the funding may not be a sufficient reason to
use a risk-adjusted rate. The type of funding is just one factor affecting the risk of a project.
Answer (B) is incorrect because a higher hurdle will result in rejection of more projects.
Answer (C) is incorrect because a risk-adjusted high hurdle rate is used for capital
investments with greater risk.
122
. Answer (D) is correct. Risk-adjusted discount rates can be used to evaluate capital
investment options. If risks differ among various elements of the cash flows, then different
discount rates can be used for different flows.
Answer (A) is incorrect because the payback period ignores both the varying risk and the
time value of money. Answer (B) is incorrect because using the cost of capital as the
discount rate does not make any adjustment for the risk differentials among the various
cash flows. Answer (C) is incorrect because risk has to be incorporated into the company's
hurdle rate to use the internal rate of return method with risk differentials.
123
. Risk adjustment Answer: b Diff: E
124
. Answer (D) is correct. Risk analysis attempts to measure the likelihood of the variability
of future returns from the proposed investment. Risk can be incorporated into capital
budgeting decisions in a number of ways, one of which is to use a hurdle rate higher than
the firm's cost of capital, that is, a risk-adjusted discount rate. This technique adjusts the
interest rate used for discounting upward as an investment becomes riskier. The expected
flow from the investment must be relatively larger, or the increased discount rate will
generate a negative net present value, and the proposed acquisition will be rejected.
Accordingly, the IRR (the rate at which the NPV is zero) for a rejected investment may
exceed the cost of capital when the risk-adjusted rate is higher than the IRR. Conversely, the
IRR for an accepted investment may be less than the cost of capital when the risk-adjusted
rate is less than the IRR. In this case, the investment presumably has very little risk.
Furthermore, risk-adjusted rates may also reflect the differing degrees of risk, not only
among investments, but by the same investments undertaken by different organizational
subunits.
Answer (A) is incorrect because discount rates may vary with the project or with the subunit
of the organization. Answer (B) is incorrect because the company may accept some projects
with IRRs less than the cost of capital, or reject some project with IRRs greater than the cost
of capital. Answer (C) is incorrect because the company may accept some projects with IRRs
less than the cost of capital, or reject some project with IRRs greater than the cost of capital.
125
. ks = 10% + (16% - 10%)1.5 = 10% + 9% = 19%.
Expected return = 21%. 21% - Risk adjustment 1% = 20%.
Risk-adjusted return = 20% > ks = 19%.
126

. Calculate the beta of the firm, and use to calculate project beta:
ks = 0.16 = 0.10 + (0.05)bFirm. bFirm = 1.2.
bProject = (bFirm)1.5 (bProject is 50% greater than current bFirm)
bProject = (1.2)1.5 = 1.8.
Calculate required return on project, kProject, and compare to IRR.
Project: kProject = 0.10 + (0.05)1.8 = 0.19 = 19%. IRR = 0.18 = 18%.
Since the required return is one percentage point greater than the expected IRR, the firm
should not accept the new project.
127

. Calculate the beta of the firm, and use to calculate project beta:
ks = 0.16 = 0.10 + (0.05)bFirm. bFirm = 1.2.
bProject = (bFirm)1.5. (bProject is 50% greater than current bFirm)
bProject = (1.2)1.5 = 1.8.

Calculate required return on project, kProject, and compare to expected return:


Project: kProject = 0.10 + (0.05)1.8 = 0.19 = 19%. Expected return = 0.18 = 18%. Since
the required return is one percentage point greater than the expected return, the firm
should not accept the new project.
128
. Risk and project selection Answer: b Diff: E
129
. Accepting risky projects Answer: e Diff: E
130
. Risk and project selection Answer: c Diff: E N

You have to find out what the required rate of return on each project is. Projects that are of high risk must have
a higher required rate of return than projects that are of low risk. The following table shows the required return
for each project on the basis of its risk level.

Required return for


Project Expected Return Risk the risk level
A 15% High 12%
B 12 Average 10%
C 11 High 12%
D 9 Low 8%
E 6 Low 8%

The company will accept all projects whose expected return exceeds its required return. Therefore, it will
accept Projects A, B, and D.

131
. Answer (D) is correct. Risk analysis attempts to measure the likelihood of the variability
of future returns from the proposed investment. Risk can be incorporated into capital
budgeting decisions in a number of ways, one of which is to use a hurdle rate higher than
the firm's cost of capital, that is, a risk-adjusted discount rate. This technique adjusts the
interest rate used for discounting upward as an investment becomes riskier. The expected
flow from the investment must be relatively larger, or the increased discount rate will
generate a negative net present value, and the proposed acquisition will be rejected.
Accordingly, the IRR (the rate at which the NPV is zero) for a rejected investment may
exceed the cost of capital when the risk-adjusted rate is higher than the IRR. Conversely, the
IRR for an accepted investment may be less than the cost of capital when the risk-adjusted
rate is less than the IRR. In this case, the investment presumably has very little risk.
Furthermore, risk-adjusted rates may also reflect the differing degrees of risk, not only
among investments, but by the same investments undertaken by different organizational
subunits.
Answer (A) is incorrect because discount rates may vary with the project or with the subunit
of the organization. Answer (B) is incorrect because the company may accept some projects
with IRRs less than the cost of capital, or reject some project with IRRs greater than the cost
of capital. Answer (C) is incorrect because the company may accept some projects with IRRs
less than the cost of capital, or reject some project with IRRs greater than the cost of capital.
132
. By Kemp not making the risk adjustment, it is true that the company will accept more
projects in the computer division, and fewer projects in the restaurant division. However,
this will make the company riskier overall, raising its cost of equity. Investors will discount
their cash flows at a higher rate, and the company’s value will fall. In addition, some of the
computer projects might not exceed the appropriate risk-adjusted hurdle rate, and will
actually be negative NPV projects, further destroying value. Therefore, statement a is false.
Because fewer of the restaurant projects will be accepted, the restaurant division will
become a smaller part of the overall company. Therefore, statement b is false. As explained
above, statement c is true.
133
. By not risk adjusting the cost of capital, the firm will tend to reject low-risk projects since
their returns will be lower than the average cost of capital, and it will take on high-risk
projects since their returns will be higher than the average cost of capital.
134
. Risk adjustment Answer: a Diff: M
kA = 13% - 3% = 10%.
If the cash flows are cost only outflows, and the analyst wants to
correctly reflect their risk, the discount rate should be adjusted
downward (in this case by subtracting 3 percentage points) to make the
discounted flows comparatively larger.
135
. Risk analysis Answer: e Diff: E
136
. Methods of analysis Answer: a Diff: M
137
. REQUIRED: he technique used to evaluate cash flows from the purchase of a machine.
DISCUSSION: (A) Simulation is a technique used to describe the behavior of a real-world
system over time. This technique usually employs a computer program to perform the
simulation computations. Sensitivity analysis examines how outcomes change as the model
parameters change.
Answer (B) is incorrect because linear programming is a mathematical technique for
optimizing a given objective function subject to certain constraints. Answer (C) is incorrect
because correlation analysis is a statistical procedure for studying the relation between
variables. Answer (D) is incorrect because differential analysis is used for decision making
that compares differences in costs (revenues) of two or more options.
138
. Answer (C) is correct. Capital budgeting is the process of planning expenditures for
assets, the returns on which are expected to continue beyond 1 year. Simulation (Monte
Carlo simulation) as applied to capital budgeting is a technique for experimenting with
logical/mathematical models using a computer. The computer is used to generate many
examples of results based upon various assumptions.
Answer (A) is incorrect because this is a method used to rank projects for capital-budgeting
decision purposes. Answer (B) is incorrect because this is a method used to rank projects for
capital-budgeting decision purposes. Answer (D) is incorrect because this is a method used
to rank projects for capital-budgeting decision purposes.
139
. Answer (B) is correct. The term "rates of return" suggests the net present value and
internal rate of return methods in this context, but simulation analysis can also be used.
Simulation is a mathematical modeling technique for performing a what-if analysis of
estimated data. For instance, it may determine how profitable a company will be if Machine
A is purchased rather than Machine B.
Answer (A) is incorrect because regression analysis is based on past data and is often used
to determine trends or divide costs into their fixed and variable components. Answer (C) is
incorrect because Markov chain analysis is used in decision problems in which the
probability of the occurrence of a future state depends only on the current state. A
characteristic of the Markov process is that the initial state matters less and less as times
goes on, because the process will eventually reach its steady state. Answer (D) is incorrect
because Gantt charting involves drawing a bar chart showing the progress of a project.
140
. Sensitivity, scenario, and simulation analyses Answer: c Diff: E N

Statement a is false; sensitivity analysis measures a project’s stand-alone risk. Statement b is false; sensitivity
analysis doesn’t take into account probabilities, while scenario analysis does. Statement c is correct.

141
. Answer (C) is correct. After a problem has been formulated into any mathematical
model, it may be subjected to sensitivity analysis, which is a trial-and-error method used to
determine the sensitivity of the estimates used. For example, forecasts of many calculated
NPVs under various assumptions may be compared to determine how sensitive the NPV is to
changing conditions. Changing the assumptions about a certain variable or group of
variables may drastically alter the NPV, suggesting that the risk of the investment may be
excessive.
Answer (A) is incorrect because sensitivity analysis is useful when cash flows, or other
assumptions, are uncertain. Answer (B) is incorrect because sensitivity analysis can be used
with any of the capital budgeting methods. Answer (D) is incorrect because sensitivity
analysis is not a ranking technique; it calculates results under varying assumptions.
142
. Answer (A) is correct. Sensitivity analysis is a technique to evaluate a model in terms of
the effect of changing the values of the parameters. It answers "what if" questions. In capital
budgeting models, sensitivity analysis is the examination of alternative outcomes under
different assumptions.
Answer (B) is incorrect because probability (risk) analysis is used to examine the array of
possible outcomes given alternative parameters. Answer (C) is incorrect because cost
behavior (variance) analysis concerns historical costs, not predictions of future cash inflows
and outflows. Answer (D) is incorrect because ROI analysis is appropriate for determining the
profitability of a company, segment, etc.
143
. Answer (B) is correct. After a problem has been formulated into any mathematical
model, it may be subjected to sensitivity analysis, which is a trial-and-error method used to
determine the sensitivity of the estimates used. For example, forecasts of many calculated
NPVs under various assumptions may be compared to determine how sensitive the NPV is to
changing conditions. Changing the assumptions about a certain variable or group of
variables may drastically alter the NPV, suggesting that the risk of the investment may be
excessive.
Answer (A) is incorrect because sensitivity analysis is a means of making several estimates
of inputs into a capital budgeting decision to determine the effect of changes in
assumptions. Answer (C) is incorrect because sensitivity analysis is not a simulation
technique; it is simply a process of making more than one estimate of unknown variables.
Answer (D) is incorrect because sensitivity analysis would not identify the required market
share; instead, it would be used to make several estimates of market share to determine
how sensitive the decision is to changes in market share.
144
. Monte Carlo simulation Answer: e Diff: M
145
. Answer (D) is correct. An increase in the discount rate would lower the net present value,
as would a decrease in cash flows or an increase in the initial investment.
Answer (A) is incorrect because a decrease in the tax rate would decrease tax expense, thus
increasing cash flows and the NPV. Answer (B) is incorrect because a decrease in the initial
investment amount would increase the NPV. Answer (C) is incorrect because an extension of
the project life and associated cash inflows would increase the NPV.
146
12A-. NPV and depreciation Answer: c Diff: E
147
.Statements a, b, c, and d are false. Statement e is correct because you can think of a firm
as a big project. If the stock is correctly priced, i.e., the stock market is efficient, the NPV of
this project should be zero.
148
. REQUIRED: The true statement regarding the NPV profiles of two mutually exclusive
capital projects.
DISCUSSION: (A) The NPV is the excess of the present value of the estimated cash inflows
over the net cost of the investment. Thus, Project 2 has a higher internal rate of return. The
internal rate of return is the cost of capital at which the NPV is zero, that is, the cost of
capital at which the NPV profile crosses the horizontal axis. The NPV profile for Project 2
intersects the horizontal axis at a higher cost of capital percentage than does that for Project
1.
Answer (B) is incorrect because Project 1 has the lower internal rate of return. Answers (C)
and (D) are incorrect because the profiles of Projects 1 and 2 intersect. Neither project will
have a higher NPV at all cost of capital percentages. To the left of the intersection point,
Project 1 has a higher NPV. To the right of the intersection point, Project 2 has a higher NPV.
149
.You can draw the NPV profiles to get an idea of what is happening. (See the diagram below.)
Statement a is false; Project B could have a higher NPV at some WACC if the NPV profiles
cross. Statement b is false; Project B could have a negative NPV when A’s NPV is positive.
Statement c is false; the IRR is unaffected by the WACC. Statement d is the correct choice.
NPV
 $

B
A

k
0 10% IRRB IRRA

150
.
NPV
($)

Discount rate (%)


0 16% 17% 18% 30%

Draw the NPV profiles using the information given in the problem. It is clear that Project A
will have a higher NPV when the cost of capital is 12 percent. Therefore, statement a is false.
At a 17 percent cost of capital, Project B will have a higher NPV than Project A. Therefore,
statement b is true. If the cost of capital were 0, then the NPV of the projects would be the
simple sum of all the cash flows. In order for statement c to be correct, B’s NPV at a 0 cost of
capital would have to be higher than A’s. From the diagram we see that this is clearly
incorrect. So, statement c is false.
151
. NPV profiles

Answer: d Diff: M N

NPV
$

k
10% 15% 20% %

The diagram above can be drawn from the statements in this question. From the diagram
drawn, statements a, b, and c are correct; therefore, statement d is the correct choice.
152
.

NPV ($)

10% k

First, draw the NPV profiles as shown above. Make sure the profiles cross at 10 percent
because the projects have the same NPV at a cost of capital of 10 percent. When WACC is
less than 10 percent, C has a higher NPV, so C’s NPV profile is above D’s NPV profile to the
left of the crossover point (10%).

Statement a is true. IRR is always independent of the cost of capital, and from the diagram
above, we can see that D’s IRR is to the right of C’s where the two lines cross the X-axis.
Statement b is false. IRR is independent of the cost of capital, and from the diagram C’s
IRR is always lower than D’s. Statement c is true. D’s MIRR will be somewhere between
the cost of capital and the IRR. Therefore, the correct choice is statement d.
153
.
NPV

k
0 7% 12% 15%
Since both projects have an IRR greater than the cost of capital, both will have a positive
NPV. Therefore, statement a is true. At 6 percent, the cost of capital is less than the
crossover rate and Project A has a higher NPV than B. Therefore, statement b is false. If the
cost of capital is 13 percent, then the cost of capital is greater than the crossover rate and B
would have a higher NPV than A. Therefore, statement c is true. Since statements a and c
are both true, the correct choice is statement e.
154
.
NPV
 $

k
0 7% 12% 14%

Statement a is correct, because at any point to the right of the crossover point B will have a
higher NPV than A. Statement b is correct for the same reason that statement a is true; at
any point to the right of the crossover point, B will have a higher NPV than A. Statement c is
correct. If B’s cost of capital is 9 percent, when MIRR is calculated the cash flows are being
reinvested at 9 percent. When IRR is used, the IRR calculation assumes that cash flows are
being reinvested at the IRR (which is higher than the cost of capital.) Statement e is the
correct choice.
155
. REQUIRED: The true statement about the IRR.
DISCUSSION: (B) the IRR is the discount rate at which the net present value is zero.
Because the present value of a dollar is higher the sooner it is received, projects with later
cash flows will have lower net present values for any given discount rate than will projects
with earlier cash flows, if other factors are constant. Hence, projects with later cash flows
will have a lower IRR.
Answer (A) is incorrect because the present value of the cash inflows is inversely related to
the discount rate, that is, if the discount rate is higher, the present value of the cash inflows
is lower. If the investment cost is lower, a higher discount rate (the IRR) will be required to
set the net present value equal to zero. Answer (C) is incorrect because the larger the cash
inflows, the higher the IRR will be. Higher cash inflows have a higher present value at any
given discount rate. A higher discount rate will be required to set the net present value
equal to zero. Answer (D) is incorrect because projects with shorter payback periods have
higher cash inflow early in the life of the project. Projects with earlier cash inflows have the
higher IRRs.
156
. Answer (A) is correct. Investments with present values in excess of their costs (positive
NPVs) that can be identified or created by the capital budgeting activities of the firm will
have a positive impact on firm value and on the price of the common shares of the firm.
Accordingly, the more effective capital budgeting is, the higher the share price.
Answer (B) is incorrect because positive NPV investments will increase, not decrease, firm
value and share price. Answer (C) is incorrect because investments with present values
equal to their costs have a zero NPV and neither increase nor decrease firm value and share
price. Answer (D) is incorrect because investments with present values equal to their costs
have a zero NPV and neither increase nor decrease firm value and share price.
157
. Answer (A) is correct. The value of the firm is the present value of the expected cash
flows, which is given by the following expression:
n
Sigma CFt
t 1

(1 K) t
If CF is net cash flow, K is the discount rate (cost of capital), and t is time, value will rise as
CF increases.
Answer (B) is incorrect because an increase in systematic (market or undiversifiable) risk will
increase the overall cost of capital and thereby increase K, the discount rate. As a result, the
value of the firm will fall. Answer (C) is incorrect because an increase in unsystematic (or
firm-specific) risk will have no effect on the value of the firm. The total risk is the sum of
systematic and unsystematic risk. By definition, the latter is the risk that can be eliminated
by diversification. Answer (D) is incorrect because an increase in the discount rate will
reduce the value of the firm.
158
. Risk analysis Answer: c Diff: E N

Statement a is false. Stand-alone risk is measured by standard deviation. Therefore, since Y’s standard
deviation is higher than X’s, Y has higher stand-alone risk than X. Statement b is false. Corporate risk is
measured by the correlation of project cash flows with other company cash flows. Therefore, since Y’s cash
flows are highly correlated with the cash flows of existing projects, while X’s are not, Y has higher corporate
risk than X. Market risk is measured by beta. Therefore, since X’s beta is greater than Y’s, statement c is true.

159
. Statement a is correct. The IRR’s of both exceed the cost of capital. Statement
b is incorrect. We cannot determine this without knowing the NPV’s of the
projects. Statement c is correct. To see why draw the NPV profiles. Statement d
is incorrect. Therefore, statement e is the correct answer.
160
. This is the only project with either a positive NPV or an IRR which exceeds the
cost of capital.
161
. Draw out the NPV profiles of these two projects. As B’s NPV declines more
rapidly with an increase in discount rates, this implies that more of the cash flows
are coming later on. Therefore, Project A has a faster payback than Project B.
162
.
NPV
$

X
Crossover

k
10% IRRY 12% IRRX

If IRRX is greater than MIRRX, then its IRR must be higher than the cost of capital. (Remember
that the MIRR will be somewhere between the cost of capital and the IRR). Therefore,
statement a must be true. Similarly, if IRR Y is less than MIRRY, then its IRR must be lower
than the cost of capital. Therefore, statement b must be true. At a cost of capital of 10
percent they have the same NPV, so this is the crossover rate. From statements a and b we
know that IRRX must be greater than IRRY, so to the right of the crossover rate NPV X will be
larger than NPVY. Consequently, to the left of the crossover rate NPV X must be smaller than
NPVY. Therefore, statement c is also true. Since statements a, b, and c are all true, the
correct choice is statement d.
163
. This statement reflects exactly the difference between the NPV and IRR
methods.
164
. Both statements a and c are correct; therefore, statement d is the correct
choice. Due to reinvestment rate assumptions, NPV and IRR can lead to conflicts;
however, there will be no conflict between NPV and MIRR if the projects are equal
in size (which is one of the assumptions in this question).
165
. Statement a is true. To see this, sketch out a NPV profile for a normal,
independent project, which means that only one NPV profile will appear on the
graph. If WACC < IRR, then IRR says accept. But in that case, NPV > 0, so NPV will
also say accept. Statement d is false. Here is the reasoning:
1. For the NPV profiles to cross, then one project must have a higher NPV at k = 0 than the
other project, that is, their vertical axis intercepts will be different.
2. A second condition for NPV profiles to cross is that one have a higher IRR than the other.
3. The third condition necessary for profiles to cross is that the project with the higher NPV
at k = 0 will have the lower IRR.
One can sketch out two NPV profiles on a graph to see that these three conditions are
indeed required.
4. The project with the higher NPV at k = 0 must have more cash inflows, because it has
the higher NPV when cash flows are not discounted, which is the situation if k = 0.
5. If the project with more total cash inflows also had its cash flows come in earlier, it would
dominate the other project--its NPV would be higher at all discount rates, and its IRR
would also be higher, so the profiles would not cross. The only way the profiles can cross
is for the project with more total cash inflows to get a relatively high percentage of those
inflows in distant years, so that their PVs are low when discounted at high rates. Most
students either grasp this intuitively or else just guess at the question!
166
. Answer (A) is correct. If the economic results of alternative capital investments were
known with certainty or with minimal risk, the quantitative analyses would reveal the
absolute best investment options. However, if the economic results are highly uncertain,
using a decision-making process that combines rational analysis with intuition is appropriate.
Moreover, nonquantifiable variables may be involved.
Answer (B) is incorrect because the decision-making process described combines rational
quantitative analysis with intuition. In addition, research has shown that intuition can
improve decision making. Answer (C) is incorrect because knowing with certainty which
investment is the most profitable is not possible. Answer (D) is incorrect because the term
bounded rationality refers to the inability to perceive all aspects of a situation and the
tendency to simplify, not to intuitive decision making.
167
. Statement e is correct; the other statements are false. IRR can lead to
conflicting decisions with NPV even with normal cash flows if the projects are
mutually exclusive. Cash outflows are discounted at the cost of capital with the
MIRR method, while cash inflows are compounded at the cost of capital. Conflicts
between NPV and IRR arise when the cost of capital is below the crossover point.
The discounted payback method does correct the problem of ignoring the time
value of money, but it still does not account for cash flows beyond the payback
period.
168
. Statements a and c are correct; therefore, statement d is the correct choice.
The discounted payback method still ignores cash flows after the payback period.
169
. Statement a is correct; the other statements are false. Multiple IRRs can occur
only for projects with nonnormal cash flows. Mutually exclusive projects imply
that only one project should be chosen. The project with the highest NPV should
be chosen.
170
. Statement a is correct; the other statements are false. Sketch the profiles.
From the information given, D has the higher IRR. The project’s scale cannot be
determined from the information given. As C’s NPV declines more rapidly with an
increase in rates, this implies that more of the cash flows are coming later on. So
C would have a slower payback than D.
171
. Answer (D) is correct. MACRS is an accelerated method of depreciation under which
depreciation expense will be greater during the early years of an asset's life. Thus, the
outflows for income taxes will be less in the early years, but greater in the later years, and
the NPV (present value of net cash inflows - investment) will be increased. The profitability
index (present value of net cash inflows ÷ the investment) must increase if the NPV
increases.
Answer (A) is incorrect because the NPV will increase. The present value of the net inflows
will increase with no change in the investment. Answer (B) is incorrect because the IRR will
increase. Deferring expenses to later years increases the discount rate needed to reduce the
NPV to $0. Answer (C) is incorrect because the payback period will be shortened. Switching
to MACRS defers expenses and increases cash flows early in the project's life.
172
12A-. Depreciation cash flows Answer: c Diff: M
173
. Answer (D) is correct. MACRS for assets with lives of 10 years or less is based on the
200% declining-balance method of depreciation. Thus, an asset with a 3-year life would have
a straight-line rate of 33-1/3%, or a double-declining-balance rate of 66-2/3%.
Answer (A) is incorrect because the straight-line method uses the same percentage each
year during an asset's life, but MACRS uses various percentages. Answer (B) is incorrect
because MACRS is unrelated to the units-of-production method. Answer (C) is incorrect
because MACRS is unrelated to SYD depreciation.
174
. Answer (A) is correct. For tax purposes, straight-line depreciation is an alternative to the
MACRS method. Both methods will result in the same total depreciation over the life of the
asset; however, MACRS will result in greater depreciation in the early years of the asset's life
because it is an accelerated method. Given that MACRS results in larger depreciation
deductions in the early years, taxes will be lower in the early years and higher in the later
years. Because the incremental benefits will be discounted over a shorter period than the
incremental depreciation costs, MACRS is preferable to the straight-line method.
Answer (B) is incorrect because both methods will produce the same total depreciation over
the life of the asset. Answer (C) is incorrect because both methods will produce the same
total tax payments (assuming rates do not change). However, given that the tax payments
will be lower in the early years under MACRS, discounting for the time value of money makes
the straight-line alternative less advantageous. Answer (D) is incorrect because both
methods will produce the same total depreciation over the life of the asset.
175
. Taxes on gain on sale

Answer: b Diff: E N

When the machine is sold the total accumulated depreciation on it is: (0.2 + 0.32 + 0.19)  $1,000,000 =
$710,000. The book value of the equipment is: $1,000,000 - $710,000 = $290,000. The machine is sold for
$400,000, so the gain is $400,000 - $290,000 = $110,000. Taxes are calculated as $110,000  0.4 = $44,000.
176

. Risk-adjusted discount rate Answer: c Diff: E


ks = 10% + (16% - 10%)1.5 = 10% + 9% = 19%.
Original IRR = 21%. 21% - Risk adjustment 1% = 20%.
Risk adjusted IRR = 20% > ks = 19%.
177
. Risk-adjusted discount rate Answer: b Diff: M
Time lines:
Project A:
0 k = 12% 1 2 3 4 5 6

CFsA -15,000 4,000 4,000 4,000 4,000 4,000 4,000


NPVA = ? = 3,978.60 5,000 Salvage value
Terminal CF = 9,000
Project B:
0 k = ? 1 2 6 Years
  
CFsB -14,815 5,100 5,100 5,100
NPVB = NPVA = 3,978.60 0 Salvage value
Terminal CF = 5,100

Tabular solution:
Solve for the NPV of Project A, which is also the NPV of Project B at some
k = ?
NPVA = -$15,000 + $4,000(PVIFA12%,6) + $5,000(PVIF12%,6)
= -$15,000 + $4,000(4.1114) + $5,000(0.5066) = $3,978.60.
Solve for kB
NPVB = $3,978.60 = -$14,815 + $5,100(PVIFAk,6)
$18,793.60 = $5,100(PVIFAk,6)
PVIFAk,6 = 3.68502.
Look across the row for 6 years in the PVIFA table. The factor for 16
percent is 3.6847; therefore, the risk-adjusted rate for Project B is
approximately 16 percent.

Financial calculator solution:


A: Inputs: CF0 = -15,000; CF1 = 4,000; Nj = 5; CF2 = 9,000; I = 12.
Output: NPV = $3,978.78.
B: Inputs: CF0 = -18,793.78; CF1 = 5,100; Nj = 6.
Output: IRR = 15.997%  16%.
178
. Risk-adjusted discount rate Answer: e Diff: T
Time lines:
Project A:
0 k = 12% 1 2 3 4 Years

CFsA -25,000 13,000 13,000 13,000 13,000


NPVA = ? = 17,663 Terminal value = 5,000
CF4 = 18,000
Project B:
0 1 2 3 4 Years
k = ?

CFsB -25,000 15,247 15,247 15,247 15,247


NPVA = NPVB = 17,663 Terminal value = 0
CF4 = 15,247

Tabular solution:
NPVA = -$25,000 + $13,000(PVIFA12%,3) + $18,000(PVIF12%,4)
= -$25,000 + $13,000(2.4018) + $18,000(0.6355) = $17,662.40.
NPVB = $17,662.40 = -$25,000 + $15,247(PVIFAk,4)
$42,662.40 = $15,247(PVIFAk,4)
(PVIFAk,4) = 2.79808
k  16%.

Financial calculator solution:


A: Inputs: CF0 = -25,000; CF1 = 13,000; Nj = 3; CF2 = 18,000; I = 12.
Output: NPVA = 17,663.13.
B: Inputs: CF0 = -42,663.13; CF1 = 15,247; Nj = 4.
Output: IRR = 16.0% = k.
179
. New project NPVAnswer: e Diff: M
Time line:
0 k = 18% 1 2 3 Years

-3,000 1,728 1,920 1,152


NPV = ?

Project analysis worksheet:


I Initial outlay
1) Cost ($4,000)
2) Decrease in NOWC 1,000
3) Total net investment ($3,000)
II Operating flows: Year: 0 1 2 3
4) EBT and depreciation $2,000 $2,000 $2,000
5) Earnings after taxes
(line 4  0.6) 1,200 1,200 1,200
6) Depreciation (from table) 1,320 1,800 600
7) Tax savings from
depreciation (5  0.4) 528 720 240
8) Net operating flows
(line 5 + 7) $1,728 $1,920 $1,440
III Terminal year cash flows:
9) Estimated salvage value $1,000
10) Tax on salvage value
((1,000 - 280)  0.4) (288)
11) Return of NOWC (1,000)
12) Total termination CFs ($ 288)
IV Net cash flows:
13) Total net cash flows ($3,000) $1,728 $1,920 $1,152

Tabular solution:
NPV = -$3,000 + $1,728(PVIF18%,1) + $1,920(PVIF18%,2) + $1,152(PVIF18%,3)
= -$3,000 + $1,728(0.8475) + $1,920(0.7182) + $1,152(0.6086) = $544.53.

Financial calculator solution:


Inputs: CF0 = -3000; CF1 = 1728; CF2 = 1920; CF3 = 1152; I = 18.
Output: NPV = $544.46  $544.
180

. New project NPV Answer: d Diff: M


Time line:
0 1 2 3 4 5 Years
k = 12%

-45,000 7,800 10,680 7,560 5,880 -1,920


NPV = ?
Depreciation cash flows:
MACRS Depreciable Annual
Year Percent Basis Depreciation
1 0.20 $60,000 $12,000
2 0.32 60,000 19,200
3 0.19 60,000 11,400
4 0.12 60,000 7,200
5 0.11 60,000 6,600
6 0.06 60,000 3,600
$60,000

Project analysis worksheet:


I Initial outlay
1) Machine cost ($60,000)
2) Decrease in NOWC 15,000
3) Total net inv. ($45,000)
II Operating cash flows
Year: 0 1 2 3 4 5
4) Reduction in cost $ 5,000 $ 5,000 $ 5,000 $ 5,000 $ 5,000
5) After-tax dec. in cost 3,000 3,000 3,000 3,000 3,000
6) Deprec. (from table) 12,000 19,200 11,400 7,200 6,600
7) Tax savings deprec.
(line 6  0.4) 4,800 7,680 4,560 2,880 2,640
8) Net operating CFs
(line 5 + 7) $ 7,800 $10,680 $ 7,560 $ 5,880 $ 5,640
III Terminal year CFs
9) Estimated salvage value $10,000
10) Tax on salvage value
(10,000 - 3,600)(0.4) (2,560)
11) Return of NOWC (15,000)
12) Total termination CFs (7,560)
IV Net CFs
13) Total Net CFs ($45,000) $ 7,800 $10,680 $ 7,560 $ 5,880 ($ 1,920)

Tabular solution:
NPV = -$45,000 + $7,800(PVIF12%,1) + $10,680(PVIF12%,2) + $7,560(PVIF12%,3)
+ $5,880(PVIF12%,4) - $1,920(PVIF12%,5)
= -$45,000 + $7,800(0.8929) + $10,680(0.7972) + $7,560(0.7118)
+ $5,880(0.6355) - $1,920(0.5674) = -$21,492.74  -$21,493.

Financial calculator solution:


Inputs: CF0 = -45,000; CF1 = 7,800; CF2 = 10,680; CF3 = 7,560;
CF4 = 5,880; CF5 = -1,920; I = 12.
Output: NPV = -$21,493.24  $21,493.
181

. New project NPVAnswer: b Diff: M


Time line:
0 k = 9% 1 2 3 4 5 Years

-40,000 9,800 11,720 9,640 8,520 15,320 N


PV = ?

Depreciation cash flows:


MACRS Depreciable Annual
Year Percent Basis Depreciation
1 0.20 $40,000 $ 8,000
2 0.32 40,000 12,800
3 0.19 40,000 7,600
4 0.12 40,000 4,800
5 0.11 40,000 4,400
6 0.06 40,000 2,400
$40,000
Project analysis worksheet:
I Initial outlay
1) Machine cost ($40,000)
2) Decrease in NOWC --
3) Total net inv. ($40,000)
II Operating cash flows
Year: 0 1 2 3 4 5
4) Inc. in earnings
before deprec. & tax $ 6,000 $ 6,000 $ 6,000 $ 6,000 $ 6,000
5) After-tax increase in
earnings (line 4  0.6) 3,600 3,600 3,600 3,600 3,600
6) Before tax reduction
in cost 5,000 5,000 5,000 5,000 5,000
7) After tax reduction
in cost (line 6  0.4) 3,000 3,000 3,000 3,000 3,000
8) Deprec. (from table) 8,000 12,800 7,600 4,800 4,400
9) Deprec. tax savings
(line 8  0.4) 3,200 5,120 3,040 1,920 1,760
10) Net operating CFs _______ _______ _______ _______ _______
(line 5 + 7 + 9) $ 9,800 $11,720 $ 9,640 $ 8,520 $ 8,360
III Terminal year CFs
11) Estimated salvage value $10,000
12) Tax on salvage value
(10,000 - 2,400)(0.4) (3,040)
13) Return of NOWC --
14) Total termination CFs 6,960
IV Net CFs
15) Total Net CFs ($40,000) $ 9,800 $11,720 $ 9,640 $ 8,520 $15,320

Tabular solution:
NPV = -$40,000 + $9,800(PVIF9%,1) + $11,720(PVIF9%,2) + $9,640(PVIF9%,3)
+ $8,520(PVIF9%,4) + $15,320(PVIF9%,5)
= -$40,000 + $9,800(0.9174) + $11,720(0.8417) + $9,640(0.7722)
+ $8,520(0.7084) + $15,320(0.6499) = $2,291.29  $2,292.

Financial calculator solution:


Inputs: CF0 = -40,000; CF1 = 9,800; CF2 = 11,720; CF3 = 9,640;
CF4 = 8,520; CF5 = 15,320; I = 9.
Output: NPV = $2,291.90  $2,292.
182
. New project NPV Answer: a Diff: M N

0 1 2 3
Equipment purchase -$600,000
NOWC -50,000

Sales increase $2,000,000 $2,000,000 $2,000,000


Operating costs 1,400,000 1,400,000 1,400,000
Operating income $ 600,000 $ 600,000 $ 600,000
Depreciation 200,000 200,000 200,000
EBIT $ 400,000 $ 400,000 $ 400,000
Taxes (35%) 140,000 140,000 140,000
EBIT(1 - T) $ 260,000 $ 260,000 $ 260,000
+Depreciation 200,000 200,000 200,000
Operating cash flow $ 460,000 $ 460,000 $ 460,000
Recovery of NOWC 50,000
Equipment sale +100,000
Taxes on sale _________ __________ __________ -35,000
Net CF -$650,000 $ 460,000 $ 460,000 $ 575,000

NPV = -$650,000 + $460,000/1.12 + $460,000/(1.12)2 + $575,000/(1.12)3


= -$650,000 + $410,714.29 + $366,709.18 + $409,273.64
= $536,697.11  $536,697.
183

. New project NPV Answer: b Diff: M N

Year 0 1 2 3 4
Project cost -5,000,000
NOWC* -300,000

Sales $3,000,000 $4,000,000 $5,000,000 $2,000,000


Operating costs (75%) 2,250,000 3,000,000 3,750,000 1,500,000
Operating income before deprec. $ 750,000 $1,000,000 $1,250,000 $ 500,000
Depreciation 1,650,000 2,250,000 750,000 350,000
EBIT -$ 900,000 -$1,250,000 $ 500,000 $ 150,000
Taxes (40%) -360,000 -500,000 200,000 60,000
EBIT(1 - T) -$ 540,000 -$ 750,000 $ 300,000 $ 90,000
Plus: Depreciation 1,650,000 2,250,000 750,000 350,000
Operating CF $1,110,000 $1,500,000 $1,050,000 $ 440,000
Recovery of NOWC 300,000
Net CF -$5,300,000 $1,110,000 $1,500,000 $1,050,000 $ 740,000

*An increase in inventories is a use of funds for the company, and an increase in accounts payable is a source
of funds for the company. Thus, the change in net operating working capital will be $200,000 - $500,000 = -
$300,000 at time 0.

NPV = -$5,300,000 + $1,110,000/1.10 + $1,500,000/(1.10)2 + $1,050,000/(1.10)3 + $740,000/(1.10)4


NPV = -$5,300,000 + $1,009,091 + $1,239,669 + $788,881 + $505,430
NPV = -$1,756,929.

184
. New project NPV

Answer: a Diff: M N

0 1 2 3 4
Project cost ($500,000)
NOWC (40,000)

Sales $800,000 $800,000 $500,000 $500,000


Operating costs 480,000 480,000 300,000 300,000
Depreciation ________ 165,000 225,000 75,000 35,000
EBIT $155,000 $ 95,000 $125,000 $165,000
Taxes (40%) 62,000 38,000 50,000 66,000
EBIT(1 - T) $ 93,000 $ 57,000 $ 75,000 $ 99,000
Plus: Depreciation 165,000 225,000 75,000 35,000
Recovery of NOWC ________ ________ ________ ________ 40,000
Net CF ($540,000) $258,000 $282,000 $150,000 $174,000

NPV = -$540,000 + $258,000/1.10 + $282,000/(1.10)2 + $150,000/(1.10)3 + $174,000/(1.10)4


= -$540,000 + $234,545.45 + $233,057.85 + $112,697.22 + $118,844.34
= $159,144.86  $159,145.
185
. New project NPVAnswer: d Diff: T
Step 1: Calculate depreciation:
Dep 1 = 100,000(0.33) = 33,000.
Dep 2 = 100,000(0.45) = 45,000.
Dep 3 = 100,000(0.15) = 15,000.
Dep 4 = 100,000(0.07) = 7,000.

Step 2: Calculate cash flows:


CF 0 = -100,000 - 5,000 = -105,000.
CF 1 = 50,000 + 33,000 = 83,000.
CF 2 = 60,000 + 45,000 = 105,000.
CF 3 = 70,000 + 15,000 = 85,000.
CF 4 = 60,000 + 7,000 + 5,000 + 15,000 = 87,000.

Step 3: Calculate NPV:


Use CF key on calculator. Enter cash flows shown above. Enter
I/YR = 12%. Solve for NPV = $168,604.

186
. New project NPVAnswer: d Diff: T
First, find the after-tax CFs associated with the project. This is
accomplished by subtracting the depreciation expense from the raw CF,
reducing this net CF by taxes and then adding back the depreciation
expense.

For t = 1: ($45,000 - $33,000)(1 - 0.4) + $33,000 = $40,200.


Similarly, the after-tax CFs for t = 2, t = 3, and t = 4 are $45,000,
$33,000, and $29,800, respectively.

Now, enter these CFs along with the cost of the equipment to find the pre-
salvage NPV (note that the salvage value is not yet accounted for in these
CFs). The appropriate discount rate for these CFs is 11 percent. This
yields a pre-salvage NPV of $16,498.72.

Finally, the salvage value must be discounted. The PV of the salvage value
is: N = 4; I = 12; PMT = 0; FV = -10,000; and PV = $6,355.18. Adding the
PV of the salvage amount to the pre-salvage NPV yields the project NPV of
$22,853.90.
187

. New project NPVAnswer: d Diff: T


The cash flows for each of the years are as follows:
0 -100,000
1 [90,000 - 50,000 - (100,000)(0.20)](1 - 0.4) + (100,000)(0.20) = 32,000
2 [90,000 - 50,000 - (100,000)(0.32)](1 - 0.4) + (100,000)(0.32) = 36,800
3 [90,000 - 50,000 - (100,000)(0.19)](1 - 0.4) + (100,000)(0.19) = 31,600
4 [90,000 - 50,000 - (100,000)(0.12)](1 - 0.4) + (100,000)(0.12) = 28,800
5 [90,000 - 50,000 - (100,000)(0.11)](1 - 0.4) + (100,000)(0.11) = 28,400
6 [90,000 - 50,000 - (100,000)(0.06)](1 - 0.4) + (100,000)(0.06) +
(10,000)(1 - 0.4) = 32,400

Enter the cash flows and solve for the NPV = $38,839.56.
188

. New project NPVAnswer: c Diff: T


Get the depreciation using the MACRS table provided in the question.
0 1 2 3 4
Cost (500,000)
Inventory ( 50,000)
Accounts Payable 10,000
Sales 600,000 600,000 600,000 600,000
Operating Cost 400,000 400,000 400,000 400,000
Depreciation 165,000 225,000 75,000 35,000
EBIT 35,000 ( 25,000) 125,000 165,000
Taxes 14,000 ( 10,000) 50,000 66,000
EBIT(1 - T) 21,000 ( 15,000) 75,000 99,000
After-tax salvage value 30,000
Return of NOWC 40,000
+ Depreciation ________ 165,000 225,000 75,000 35,000
Net CF ($540,000) $186,000 $210,000 $150,000 $204,000

Note in year 4 your $40,000 of net operating working capital is recovered


plus the after tax salvage value of $30,000.

Enter the cash flows into the cash flow register and solve for the NPV
using the WACC of 10%. NPV = $54,676.59.
189
. Risk-adjusted NPV Answer: a Diff: M
Time lines:
Project A:
0 k = 14% 1 2 3 4 Years

CFsA -200,000 71,104 71,104 71,104 71,104


NPVA = ?

Project B:
0 k = 10% 1 2 3 4 Years

CFsB -200,000 0 0 146,411 146,411


NPVB = ?

Calculate required returns on A and B:


Project A High risk kRisk adjusted = 12% + 2% = 14%.
Project B Low risk kRisk adjusted = 12% - 2% = 10%.

Tabular solution:
NPVA = $71,104(PVIFA14%,4) - $200,000
= $71,104(2.9137) - $200,000 = $7,175.72.
NPVB = $146,411(PVIF10%,3) + $146,411(PVIF10%,4) - $200,000
= $146,411(0.7513) + $146,411(0.6830) - $200,000 = $9,997.30.
Project B has the higher NPV. Since they are mutually exclusive, select
Project B.

Financial calculator solution:


A Inputs: CF0 = -200,000; CF1 = 71,104; Nj = 4; I = 14.
Output: NPVA = $7,176.60  $7,177.
B Inputs: CF0 = -200,000; CF1 = 0; Nj = 2; CF2 = 146,411; Nj = 2; I = 10.
Output: NPVB = $10,001.43  $10,001.
Note: The difference in the NPV B between the tabular solution and
financial calculator cash flow solution of $4.13 is due to rounding of
PVIF factors to four significant decimals. Greater precision in the PVIF
factors produces identical answers.
190
. Risk-adjusted NPV Answer: e Diff: M
Time line:
0 k = 12% 1 2 10 Years
  
-50,000 6,000 6,000 6,000
NPV = ? Salvage value 10,000
CF10 = 16,000
kProject = 6% + 4%(1.5) = 12%.

Tabular solution:
NPV = -$50,000 + $6,000(PVIFA12%,10) + $10,000(PVIF12%,10)
= -$50,000 + $6,000(5.6502) + $10,000(0.3220)
= -$12,878.80  -$12,879.
Financial calculator solution:
Inputs: CF0 = -50,000; CF1 = 6,000; Nj = 9; CF10 = 16,000; I = 12%.
Output: NPV = -$12,878.93  -$12,879.
191
. Risk-adjusted NPV Answer: c Diff: M
Time lines:
Project A:
0 k = 12% 1 2 3 Periods

CFsA -5,000 2,000 2,500 2,250 N


PVA = ?

Project B:
0 k = 14% 1 2 3 Periods

CFsB -5,000 3,000 2,600 2,900 N


PVB = ?

Project A: kAverage risk = 12%.


Project B: kHigh risk = 12% + 2% = 14%.

Tabular solution:
NPVA = $2,000(PVIF12%,1) + $2,500(PVIF12%,2) + $2,250(PVIF12%,3) - $5,000
= $2,000(0.8929) + $2,500(0.7972) + $2,250(0.7118) - $5,000
= $380.35  $380.
NPVB = $3,000(PVIF14%,1) + $2,600(PVIF14%,2) + $2,900(PVIF14%,3) - $5,000
= $3,000(0.8772) + $2,600(0.7695) + $2,900(0.6750) - $5,000
= $1,589.80  $1,590.

Financial calculator solution:


A: Inputs: CF0 = -5,000; CF1 = 2,000; CF2 = 2,500; CF3 = 2,250; I% = 12.
Output: NPV = $380.20  $380.
B: Inputs: CF0 = -5,000; CF1 = 3,000; CF2 = 2,600; CF3 = 2,900; I% = 14.
Output: NPV = $1,589.61  $1,590.
192

. NPV and risk-adjusted discount rate Answer: e Diff: T N

The following table shows the cash flows:


0 1 2 3 4 5
Initial invest. outlay -$30.0
Sales $20.0 $20.0 $20.0 $20.0 $20.0
Oper. cost 12.0 12.0 12.0 12.0 12.0
Deprec. 10.0 10.0 10.0 0.0 0.0
EBIT -$ 2.0 -$ 2.0 -$ 2.0 $ 8.0 $ 8.0
Less: Taxes -0.8 -0.8 -0.8 3.2 3.2
EBIT(1 - T) -$ 1.2 -$ 1.2 -$ 1.2 $ 4.8 $ 4.8
Add back: Deprec. 10.0 10.0 10.0 0.0 0.0
NCF -$30.0 $ 8.8 $ 8.8 $ 8.8 $ 4.8 $ 4.8

Step 1: Determine the NPV of net cash flows:


NPV = -$30 + $8.8/1.10 + $8.8/(1.10)2 + $8.8/(1.10)3 + $4.8/(1.10)4 + $4.8/(1.10)5
= -$30 + $8 + $7.2727 + $6.6116 + $3.2785 + $2.9804
= -$1.8568 million.

Step 2: Determine the NPV of the project’s AT salvage value:


$1.2/(1.12)5 = $0.6809 million.

Step 3: Determine the project’s NPV:


Add the PV of the salvage value to the NPV of the cash flows to get the project’s NPV.
NPV = -$1.8568 + $0.6809 = -$1.1759 million  -$1.18 million.

Financial calculator solution:


Step 1: Determine the NPV of net cash flows:
Enter the following inputs in the calculator:
CF0 = -30, CF1-3 = 8.8, CF4-5 = 4.8, I = 10, and then solve for NPV = -$1.8568 million.

Step 2: Determine the NPV of the project’s AT salvage value:


Enter the following inputs in the calculator:
CF0 = 0, CF1-4 = 0, CF5 = 1.2, I = 12, and then solve for NPV = $0.6809 million.

Step 3: Determine the project’s NPV:


Add the PV of the salvage value to the NPV of the cash flows to get the project’s NPV.
-$1.8568 + $0.6809 = -$1.1759 million  $-1.18 million.
193

. Discounting risky outflows Answer: e Diff: M


Time line:
0 1 2 3 4
| | | | |
CFsNew Tech -1,500 -315 -315 -315 -315
NPVNew Tech = ?
CFsOld Tech -600 -600 -600 -600 -600
NPVOld Tech = ?
Recognize that (1) risk outflows must be discounted at lower rates, and
(2) since Project New Tech is risky, it must be discounted at a rate of
12% - 3% = 9%. Project Old Tech must be discounted at 12%.
Tabular solution:
PVNew Tech = -$1,500 - $315(PVIFA9%,4) = -$1,500 - $315(3.2397)
= -$2,520.51.
PVOld Tech = -$600 - $600(PVIFA12%,4) = -$600 - $600(3.0373) = -$2,422.38.
PVOld Tech is a smaller outflow than NPV New Tech, thus, Project Old Tech is the
better project.
Financial calculator solution:
Project New Tech: Inputs: CF0 = -1,500; CF1 = -315; Nj = 4; I = 9.
Output: NPV = -$2,520.51.
Project Old Tech: Inputs: CF0 = -600; CF1 = -600; Nj = 4; I = 12.
Output: NPV = -$2,422.41.
194

. Risky Projects Answer: d Diff: M


Look at the NPV, IRR, and hurdle rate for each project:
Project A B C D E
Hurdle 9.00% 9.00% 11.00% 13.00% 13.00%
NPV $13,822 $11,998
IRR 12.11% 14.04% 10.85% 16.64% 11.63%

Projects A and B are mutually exclusive, so we pick project A because it


has the largest NPV. Projects C, D, and E are independent so we pick the
ones whose IRR exceeds the cost of capital, in this case, just D.
Therefore, the projects undertaken are A and D.
195

. Scenario analysis Answer: c Diff: M


Calculate expected value of NPV:
Probability of Unit Sales Sales NPV
Outcome, Pi Volume Price (In 1000s) Pi(x)
Worst case 0.30 6,000 $3,600 -$6,000 0.3(-6,000) = -1,800
Base case 0.50 10,000 4,200 13,000 0.5(13,000) = 6,500
Best case 0.20 13,000 4,400 28,000 0.2(28,000) = 5,600
Expected NPV = $10,300

Calculate standard deviation of NPV (in thousands):


Pi(x - x )2 (x - x )2 Pi(x - x )2
_____________________________________ ____________________ _________________________

Worst case 0.3(-6 - 10.3)2 265.69 79.707


Base case 0.5(13 - 10.3)2 7.29 3.645
Best case 0.2(28 - 10.3)2 313.29 62.658
Sum 146.01
(146.01)½ = 12,083 thousand.

Calculate coefficient of variation (CV) of NPV:


CVNPV = σNPV/E(NPV) = $12,083/$10,300 = 1.17.
196
. New project investment Answer: a Diff: E
Initial investment:
Cost ($40,000)
Change in NOWC (2,000)
($42,000)
197

. Operating cash flow Answer: e Diff: M


Depreciation schedule:
Depreciable basis = $40,000.
MACRS
Depreciation Depreciable Annual
Year Rate Basis Depreciation
1 0.33 $40,000 $13,200
2 0.45 40,000 18,000
3 0.15 40,000 6,000
4 0.07 40,000 2,800
$40,000

Operating cash flows:


Year 1 2 3
1) Increase in revenues $20,000 $20,000 $20,000
2) Increase in costs (5,000) (5,000) (5,000)
3) Before-tax change in
earnings 15,000 15,000 15,000
4) After-tax change in
earnings (line 3  0.60) 9,000 9,000 9,000
5) Depreciation 13,200 18,000 6,000
6) Tax savings deprec.
(line 6  0.40) 5,280 7,200 2,400
7) Net operating CFs
(line 4 + 6) $14,280 $16,200 $11,400
198
. Non-operating cash flows Answer: a Diff: M
Additional Year 3 cash flows:
3
Salvage value $25,000
Tax on Salvage value (8,880)*
Recovery of NOWC 2,000
$18,120

*(Market value - Book value)(Tax rate)


($25,000 - $2,800)(0.40) = $8,880.
199
. New project NPVAnswer: c Diff: M
Time line:
0 k = 14% 1 2 3 Years

-42,000 14,280 16,200 11,400


TV = 18,120
29,520
Tabular solution:
NPVk = 14% = -$42,000 + $14,280(PVIF14%,1)
+ $16,200(PVIF14%,2) + $29,520(PVIF14%,3)
= -$42,000 + $14,280(0.8772) + $16,200(0.7695)
+ $29,520(0.6750) = $2,918.32.

Financial calculator solution:


Inputs: CF0 = -42,000; CF1 = 14,280; CF2 = 16,200; CF3 = 29,520; I = 14.
Output: NPV = $2,916.85  $2,917.

Note: Tabular solution differs from calculator solution due to interest


factor rounding.
200

. New project investment Answer: d Diff: E


Initial investment:
Cost ($50,000)
Modification (10,000)
Change in NOWC (2,000)
Total net investment = ($62,000)
201

. Operating cash flow Answer: c Diff: M


Depreciation schedule:
Depreciable basis = $60,000.
MACRS
Depreciation Depreciable Annual
Year Rate Basis Depreciation
1 0.33 $60,000 $19,800
2 0.45 60,000 27,000
3 0.15 60,000 9,000
4 0.07 60,000 4,200
$60,000

Operating cash flows:


Year 1 2 3
1) Before-tax cost reduction $20,000 $20,000 $20,000
2) After-tax cost reduction
(line 1  0.6) 12,000 12,000 12,000
3) Depreciation 19,800 27,000 9,000
4) Tax savings from deprec.
(line 3  0.4) 7,920 10,800 3,600
5) Net operating CFs $19,920 $22,800 $15,600
202

. Non-operating cash flows Answer: c Diff: M


Additional Year 3 cash flows:
3
Salvage value $20,000
Tax on salvage value (6,320)*
Recovery of NOWC 2,000
Total terminal year CF $15,680

*(Market value - Book value)(Tax rate)


($20,000 - $4,200)(0.40) = $6,320.
203

. New project NPVAnswer: a Diff: M


Time line:
0 k = 10% 1 2 3 Years

-62,000 19,920 22,800 15,600


TV = 15,680
31,280
Tabular solution:
NPVk = 10% = -$62,000 + $19,920(PVIF10%,1) + $22,800(PVIF10%,2)
+ $31,280(PVIF10%,3)
= -$62,000 + $19,920(0.9091) + $22,800(0.8264)
+ $31,280(0.7513) = -$1,548.14.

Financial calculator solution:


Inputs: CF0 = -62,000; CF1 = 19,920; CF2 = 22,800; CF3 = 31,280; I = 10.
Output: NPV = -$1,546.81  -$1,547.

Note: Tabular solution differs from calculator solution due to interest


factor rounding.

You might also like